Location via proxy:   [ UP ]  
[Report a bug]   [Manage cookies]                

On proving an Inequality of Ramanujan using Explicit Order Estimates of the Mertens Function

SUBHAM DE \orcidlink0009-0001-3265-4354 Department of Mathematics, Indian Institute of Technology, Delhi, India.
Abstract

This research article provides an unconditional proof of an inequality proposed by Srinivasa Ramanujan involving the Prime Counting Function π(x)𝜋𝑥\pi(x)italic_π ( italic_x ),

(π(x))2<exlogxπ(xe)superscript𝜋𝑥2𝑒𝑥𝑥𝜋𝑥𝑒\displaystyle(\pi(x))^{2}<\frac{ex}{\log x}\pi\left(\frac{x}{e}\right)( italic_π ( italic_x ) ) start_POSTSUPERSCRIPT 2 end_POSTSUPERSCRIPT < divide start_ARG italic_e italic_x end_ARG start_ARG roman_log italic_x end_ARG italic_π ( divide start_ARG italic_x end_ARG start_ARG italic_e end_ARG )

for every real xexp(1486)𝑥1486x\geq\exp(1486)italic_x ≥ roman_exp ( 1486 ), using specific order estimates of the Mertens Function, M(x)𝑀𝑥M(x)italic_M ( italic_x ). The proof primarily hinges upon investigating the underlying relation between M(x)𝑀𝑥M(x)italic_M ( italic_x ) and the Second Chebyshev Function, ψ(x)𝜓𝑥\psi(x)italic_ψ ( italic_x ), in addition to applying the meromorphic properties of the Riemann Zeta Function, ζ(s)𝜁𝑠\zeta(s)italic_ζ ( italic_s ) with an intention of deriving an improved approximation for π(x)𝜋𝑥\pi(x)italic_π ( italic_x ).

Keywords and Phrases: Riemann Zeta Function, Mertens Function, Chebyshev Function, Arithmetic Function, Error Estimates, Perron’s Formula, Mo¨¨𝑜\ddot{o}over¨ start_ARG italic_o end_ARGbius Inversion Formula, Dirichlet Partial Summation Formula.

2020 MSC: Primary 11A41, 11A25, 11N05, 11N37, 11N56 .
               Secondary 11M06, 11M26 .

1 Introduction and Motivation

The motivation for investigating the distribution of prime numbers over the real line \mathbb{R}blackboard_R first reflected in the writings of famous mathematician Ramanujan, as evident from his letters [23, pp. xxiii-xxx , 349-353] to one of the most prominent mathematiciana of 20thsuperscript20𝑡20^{th}20 start_POSTSUPERSCRIPT italic_t italic_h end_POSTSUPERSCRIPT century, G. H. Hardy during the months of Jan/Feb of 1913191319131913, which are testaments to several strong assertions about prime numbers, especaially the Prime Counting Function, π(x)𝜋𝑥\pi(x)italic_π ( italic_x ) [ref. (2.1.1)].

In the following years, Hardy himself analyzed some of thoose results [24] [25, pp. 234-238], and even wholeheartedly acknowledged about them in many of his publications, one such notable result is the Prime Number Theorem [ref. (2.1.1)].

Ramanujan provided several inequalities regarding the behaviour and the asymptotic nature of π(x)𝜋𝑥\pi(x)italic_π ( italic_x ). One of such relation can be found in the notebooks written by Ramanujan himself has the following claim.

Theorem 1.0.1.

(Ramanujan’s Inequality) For x𝑥xitalic_x sufficiently large, we shall have,

(π(x))2<exlogxπ(xe)superscript𝜋𝑥2𝑒𝑥𝑥𝜋𝑥𝑒\displaystyle(\pi(x))^{2}<\frac{ex}{\log x}\pi\left(\frac{x}{e}\right)( italic_π ( italic_x ) ) start_POSTSUPERSCRIPT 2 end_POSTSUPERSCRIPT < divide start_ARG italic_e italic_x end_ARG start_ARG roman_log italic_x end_ARG italic_π ( divide start_ARG italic_x end_ARG start_ARG italic_e end_ARG ) (1.1)

Worth mentioning that, Ramanujan indeed provided a simple, yet unique solution in support of his claim. Furthermore, it has been well established that, the result is not true for every positive real x𝑥xitalic_x. Thus, the most intriguing question that the statement of Theorem (1.0.1) poses is, is there any x0subscript𝑥0x_{0}italic_x start_POSTSUBSCRIPT 0 end_POSTSUBSCRIPT such that, Ramanujan’s Inequality will be unconditionally true for every xx0𝑥subscript𝑥0x\geq x_{0}italic_x ≥ italic_x start_POSTSUBSCRIPT 0 end_POSTSUBSCRIPT?

A brilliant effort put up by F. S. Wheeler, J. Keiper, and W. Galway in search for such x0subscript𝑥0x_{0}italic_x start_POSTSUBSCRIPT 0 end_POSTSUBSCRIPT using tools such as : Mathematica went in vain, although independently Galway successfully computed the largest prime counterexample below 1011superscript101110^{11}10 start_POSTSUPERSCRIPT 11 end_POSTSUPERSCRIPT at x=38 358 837 677𝑥38 358 837 677x=38\mbox{ }358\mbox{ }837\mbox{ }677italic_x = 38 358 837 677. However, Hassani [22, Theorem 1.2] proposed a more inspiring answer to the question in a way that, \exists such x0=138 766 146 692 471 228subscript𝑥0138 766 146 692 471 228x_{0}=138\mbox{ }766\mbox{ }146\mbox{ }692\mbox{ }471\mbox{ }228italic_x start_POSTSUBSCRIPT 0 end_POSTSUBSCRIPT = 138 766 146 692 471 228 with (1.1) being satisfied for every xx0𝑥subscript𝑥0x\geq x_{0}italic_x ≥ italic_x start_POSTSUBSCRIPT 0 end_POSTSUBSCRIPT, but one has to neccesarily assume the Riemann Hypothesis. In a recent paper by A. W. Dudek and D. J. Platt [2, Theorem 1.2], it has been established that, ramanujan’s Inequality holds true unconditionally for every xexp(9658)𝑥9658x\geq\exp(9658)italic_x ≥ roman_exp ( 9658 ). Although this can be considered as an exceptional achievement in this area, efforts of further improvements to this bound are already underway. For example, one recent result by Axler [26] suggests that, the lower bound for x𝑥xitalic_x, namely exp(9658)9658\exp(9658)roman_exp ( 9658 ) can in fact be further improved upto exp(3158.442)3158.442\exp(3158.442)roman_exp ( 3158.442 ) using similar techniques as described in [2], although modifying the error term accordingly.

This article shall provide in detail, a new proof of Ramanujan’s Inequality, using a completely different technique by introducing the notion of Mertens Function. We shall utilize one of the most significant order properties of M(x)𝑀𝑥M(x)italic_M ( italic_x ), namely, M(x)=O(x)𝑀𝑥𝑂𝑥M(x)=O(\sqrt{x})italic_M ( italic_x ) = italic_O ( square-root start_ARG italic_x end_ARG ) in order to find an improved estimate for π(x)𝜋𝑥\pi(x)italic_π ( italic_x ) [ref. Section 3.3]. Thus in turn, we shall verify the inequality in the final part of the article [ref. Section 4]. As an application to this method, we shall be able to refine the lower bound for x𝑥xitalic_x even further in order for Theorem (1.0.1) to hold true without any further assumptions.

2 Arithmetic Functions

As for definition, Arithmetic Functions are in fact complex-valued functions on the set of Natural Numbers \mathbb{N}blackboard_N.

For the convenience of the readers, let us first introduce some notations, under standard assumption that, x𝑥x\in\mathbb{R}italic_x ∈ blackboard_R.

Definition 2.0.1.

We say f(x)𝑓𝑥f(x)italic_f ( italic_x ) is asymptotic to g(x)𝑔𝑥g(x)italic_g ( italic_x ), and denote it by, f(x)g(x)similar-to𝑓𝑥𝑔𝑥f(x)\sim g(x)italic_f ( italic_x ) ∼ italic_g ( italic_x ) if, limxf(x)g(x)=1subscript𝑥𝑓𝑥𝑔𝑥1\lim\limits_{x\to\infty}\frac{f(x)}{g(x)}=1roman_lim start_POSTSUBSCRIPT italic_x → ∞ end_POSTSUBSCRIPT divide start_ARG italic_f ( italic_x ) end_ARG start_ARG italic_g ( italic_x ) end_ARG = 1.

Definition 2.0.2.

(Big O𝑂Oitalic_O Notation) Given g(x)>0 xaformulae-sequence𝑔𝑥0for-all 𝑥𝑎g(x)>0\hskip 10.0pt\forall\mbox{ }x\geq aitalic_g ( italic_x ) > 0 ∀ italic_x ≥ italic_a, the notation, f(x)=O(g(x))𝑓𝑥𝑂𝑔𝑥f(x)=O(g(x))italic_f ( italic_x ) = italic_O ( italic_g ( italic_x ) ) implies that, the quotient, f(x)g(x)𝑓𝑥𝑔𝑥\frac{f(x)}{g(x)}divide start_ARG italic_f ( italic_x ) end_ARG start_ARG italic_g ( italic_x ) end_ARG is bounded for all xa𝑥𝑎x\geq aitalic_x ≥ italic_a; i.e., \exists a constant M>0𝑀0M>0italic_M > 0 such that,

|f(x)|M.g(x) xaformulae-sequence𝑓𝑥𝑀𝑔𝑥for-all 𝑥𝑎\arrowvert f(x)\arrowvert\leq M.g(x)\hskip 10.0pt\mbox{, }\forall\mbox{ }x\geq a| italic_f ( italic_x ) | ≤ italic_M . italic_g ( italic_x ) , ∀ italic_x ≥ italic_a .

In this section, we shall discuss about a few specific important such type of arithmetic functions pertaining to the context of the paper and the proof of the original result.

2.1 Prime Counting Function

Definition 2.1.1.

For each x0x0\textit{x}\geq 0x ≥ 0 ,we define,

π(x):=assign𝜋𝑥absent\pi(x):=italic_π ( italic_x ) :=The number of primes xabsentx\leq\textit{x}≤ x.

The most important contribution of π(x)𝜋𝑥\pi(x)italic_π ( italic_x ) is undoubtedly to the Prime Number Theorem, which can be stated as follows.

Theorem 2.1.1.

For every real x0𝑥0x\geq 0italic_x ≥ 0, the following estimate is valid.

π(x)xlogxsimilar-to𝜋𝑥𝑥𝑥\displaystyle\pi(x)\sim\frac{x}{\log x}italic_π ( italic_x ) ∼ divide start_ARG italic_x end_ARG start_ARG roman_log italic_x end_ARG (2.1)

Equivalently,

limxπ(x)logxx=1subscript𝑥𝜋𝑥𝑥𝑥1\displaystyle\lim\limits_{x\to\infty}\frac{\pi(x)\log x}{x}=1roman_lim start_POSTSUBSCRIPT italic_x → ∞ end_POSTSUBSCRIPT divide start_ARG italic_π ( italic_x ) roman_log italic_x end_ARG start_ARG italic_x end_ARG = 1 (2.2)

.

For an elementary proof of above, readers can refer to [20].

2.2 Chebyshev Function

Chebyshev ψ𝜓\psiitalic_ψ Function has the following definition.

Definition 2.2.1.

For each x0x0\textit{x}\geq 0x ≥ 0 , we define,

ψ(x):=nxΛ(n)assign𝜓𝑥subscript𝑛𝑥Λ𝑛\psi(x):=\sum\limits_{n\leq x}\Lambda(n)italic_ψ ( italic_x ) := ∑ start_POSTSUBSCRIPT italic_n ≤ italic_x end_POSTSUBSCRIPT roman_Λ ( italic_n ) ,

Where ,

Λ(n):={logp , if n=pm, pmx, m0 , otherwise .assignΛ𝑛cases𝑝 formulae-sequence if 𝑛superscript𝑝𝑚formulae-sequence superscript𝑝𝑚𝑥 𝑚0  otherwise \displaystyle\Lambda(n):=\left\{\begin{array}[]{cc}\log p\mbox{ },&\mbox{ if }% n=p^{m},\mbox{ }p^{m}\leq x,\mbox{ }m\in\mathbb{N}\\ 0\mbox{ },&\mbox{ otherwise }.\end{array}\right.roman_Λ ( italic_n ) := { start_ARRAY start_ROW start_CELL roman_log italic_p , end_CELL start_CELL if italic_n = italic_p start_POSTSUPERSCRIPT italic_m end_POSTSUPERSCRIPT , italic_p start_POSTSUPERSCRIPT italic_m end_POSTSUPERSCRIPT ≤ italic_x , italic_m ∈ blackboard_N end_CELL end_ROW start_ROW start_CELL 0 , end_CELL start_CELL otherwise . end_CELL end_ROW end_ARRAY (2.5)

Λ(n)Λ𝑛\Lambda(n)roman_Λ ( italic_n ) is said to be the ”Mangoldt Function” .

An important observation is,

ψ(x)=nxΛ(n)=m=1p,pmxΛ(pm)=m=1px1mlogp𝜓𝑥subscript𝑛𝑥Λ𝑛superscriptsubscript𝑚1subscript𝑝superscript𝑝𝑚𝑥Λsuperscript𝑝𝑚superscriptsubscript𝑚1subscript𝑝superscript𝑥1𝑚𝑝\displaystyle\psi(x)=\sum\limits_{n\leq x}\Lambda(n)=\sum\limits_{m=1}^{\infty% }\sum\limits_{p,p^{m}\leq x}\Lambda(p^{m})=\sum\limits_{m=1}^{\infty}\sum% \limits_{p\leq x^{\frac{1}{m}}}\log pitalic_ψ ( italic_x ) = ∑ start_POSTSUBSCRIPT italic_n ≤ italic_x end_POSTSUBSCRIPT roman_Λ ( italic_n ) = ∑ start_POSTSUBSCRIPT italic_m = 1 end_POSTSUBSCRIPT start_POSTSUPERSCRIPT ∞ end_POSTSUPERSCRIPT ∑ start_POSTSUBSCRIPT italic_p , italic_p start_POSTSUPERSCRIPT italic_m end_POSTSUPERSCRIPT ≤ italic_x end_POSTSUBSCRIPT roman_Λ ( italic_p start_POSTSUPERSCRIPT italic_m end_POSTSUPERSCRIPT ) = ∑ start_POSTSUBSCRIPT italic_m = 1 end_POSTSUBSCRIPT start_POSTSUPERSCRIPT ∞ end_POSTSUPERSCRIPT ∑ start_POSTSUBSCRIPT italic_p ≤ italic_x start_POSTSUPERSCRIPT divide start_ARG 1 end_ARG start_ARG italic_m end_ARG end_POSTSUPERSCRIPT end_POSTSUBSCRIPT roman_log italic_p (2.6)

In fact, one can use the ψ𝜓\psiitalic_ψ function in order tosimplify the statement of the Prime Number Theorem (2.1.1). In other words, one can deduce that, proving the theorem is equivalent to proving the following statement,

ψ(x)x as x.similar-to𝜓𝑥𝑥 as 𝑥\displaystyle\psi(x)\sim x\mbox{\hskip 20.0pt as \hskip 10.0pt}x\rightarrow\infty.italic_ψ ( italic_x ) ∼ italic_x as italic_x → ∞ . (2.7)

2.3 Mo¨¨𝑜\ddot{o}over¨ start_ARG italic_o end_ARGbius Function

We start with the formal definition.

Definition 2.3.1.

(Mo¨¨𝑜\ddot{o}over¨ start_ARG italic_o end_ARGbius Function) μ:{0,±1}:𝜇0plus-or-minus1\mu:\mathbb{N}\rightarrow\{0,\pm 1\}italic_μ : blackboard_N → { 0 , ± 1 } is defined as follows:

μ(n):={(1)k if n=i=1kpiai such that, gcd(pi,pj)=1ij1 if, n=10 otherwise.assign𝜇𝑛casessuperscript1𝑘formulae-sequence if 𝑛superscriptsubscriptproduct𝑖1𝑘superscriptsubscript𝑝𝑖subscript𝑎𝑖 such that, 𝑔𝑐𝑑subscript𝑝𝑖subscript𝑝𝑗1for-all𝑖𝑗missing-subexpressionmissing-subexpressionmissing-subexpressionmissing-subexpression1 if, 𝑛1missing-subexpressionmissing-subexpressionmissing-subexpressionmissing-subexpression0 otherwise.missing-subexpression\mu(n):=\left\{\begin{array}[]{lll}{(-1)}^{k}&\mbox{ if }n=\prod\limits_{i=1}^% {k}{{p_{i}}^{a_{i}}}\mbox{ such that, }gcd(p_{i},p_{j})=1\hskip 20.0pt\forall% \hskip 10.0pti\neq j\\ \\ 1&\mbox{ if, }n=1\\ \\ 0&\mbox{ otherwise.}\end{array}\right.italic_μ ( italic_n ) := { start_ARRAY start_ROW start_CELL ( - 1 ) start_POSTSUPERSCRIPT italic_k end_POSTSUPERSCRIPT end_CELL start_CELL if italic_n = ∏ start_POSTSUBSCRIPT italic_i = 1 end_POSTSUBSCRIPT start_POSTSUPERSCRIPT italic_k end_POSTSUPERSCRIPT italic_p start_POSTSUBSCRIPT italic_i end_POSTSUBSCRIPT start_POSTSUPERSCRIPT italic_a start_POSTSUBSCRIPT italic_i end_POSTSUBSCRIPT end_POSTSUPERSCRIPT such that, italic_g italic_c italic_d ( italic_p start_POSTSUBSCRIPT italic_i end_POSTSUBSCRIPT , italic_p start_POSTSUBSCRIPT italic_j end_POSTSUBSCRIPT ) = 1 ∀ italic_i ≠ italic_j end_CELL start_CELL end_CELL end_ROW start_ROW start_CELL end_CELL start_CELL end_CELL start_CELL end_CELL end_ROW start_ROW start_CELL 1 end_CELL start_CELL if, italic_n = 1 end_CELL start_CELL end_CELL end_ROW start_ROW start_CELL end_CELL start_CELL end_CELL start_CELL end_CELL end_ROW start_ROW start_CELL 0 end_CELL start_CELL otherwise. end_CELL start_CELL end_CELL end_ROW end_ARRAY

One can in fact use definition (2.3.1) to deduce the following property regarding the Mo¨¨𝑜\ddot{o}over¨ start_ARG italic_o end_ARGbius Function.

Proposition 2.3.1.

[17, Theorem 2.1 , pp. 25]

d|nμ(d)=1n={1 if, n=10 otherwise. subscriptconditional𝑑𝑛𝜇𝑑1𝑛cases1 if, 𝑛1missing-subexpressionmissing-subexpression0 otherwise. \sum\limits_{d|n}{\mu(d)}=\left\lfloor\frac{1}{n}\right\rfloor=\left\{\begin{% array}[]{ll}1&\mbox{ if, }n=1\\ \\ 0&\mbox{ otherwise. }\end{array}\right.∑ start_POSTSUBSCRIPT italic_d | italic_n end_POSTSUBSCRIPT italic_μ ( italic_d ) = ⌊ divide start_ARG 1 end_ARG start_ARG italic_n end_ARG ⌋ = { start_ARRAY start_ROW start_CELL 1 end_CELL start_CELL if, italic_n = 1 end_CELL end_ROW start_ROW start_CELL end_CELL start_CELL end_CELL end_ROW start_ROW start_CELL 0 end_CELL start_CELL otherwise. end_CELL end_ROW end_ARRAY (2.8)

2.4 Mertens Function

Definition 2.4.1.

The Mertens Function M::𝑀M:\mathbb{N}\rightarrow\mathbb{Z}italic_M : blackboard_N → blackboard_Z has the representation,

M(n):=d=1nμ(d)assign𝑀𝑛superscriptsubscript𝑑1𝑛𝜇𝑑M(n):=\sum\limits_{d=1}^{n}{\mu(d)}italic_M ( italic_n ) := ∑ start_POSTSUBSCRIPT italic_d = 1 end_POSTSUBSCRIPT start_POSTSUPERSCRIPT italic_n end_POSTSUPERSCRIPT italic_μ ( italic_d ) (2.9)
Remark 2.4.1.

In general, there’s a notion of the Extended Mertens Function,

M(x):=1nxμ(n)assign𝑀𝑥subscript1𝑛𝑥𝜇𝑛M(x):=\sum\limits_{1\leq n\leq x}{\mu(n)}italic_M ( italic_x ) := ∑ start_POSTSUBSCRIPT 1 ≤ italic_n ≤ italic_x end_POSTSUBSCRIPT italic_μ ( italic_n ) ,       for-all\forall x𝑥x\in\mathbb{R}italic_x ∈ blackboard_R

In his paper [4], Mertens conjectured that, for all M(n)𝑀𝑛M(n)italic_M ( italic_n ) with 1n1041𝑛superscript1041\leq n\leq 10^{4}1 ≤ italic_n ≤ 10 start_POSTSUPERSCRIPT 4 end_POSTSUPERSCRIPT, we shall have,

|M(n)|<n𝑀𝑛𝑛\displaystyle|M(n)|<\sqrt{n}| italic_M ( italic_n ) | < square-root start_ARG italic_n end_ARG (2.10)

This is also known as the Mertens Hypothesis. [ Interested readers can refer to [3, Theorem 14.28, pp. 374] ]

Extending Mertens’ results further upto n=5×106𝑛5superscript106n=5\times 10^{6}italic_n = 5 × 10 start_POSTSUPERSCRIPT 6 end_POSTSUPERSCRIPT, Sterneck [5] conjectured that,

|M(n)|<12n𝑀𝑛12𝑛|M(n)|<\frac{1}{2}\sqrt{n}| italic_M ( italic_n ) | < divide start_ARG 1 end_ARG start_ARG 2 end_ARG square-root start_ARG italic_n end_ARG ,        for-all\forall n>200𝑛200n>200italic_n > 200 .

The primary objective for Mertens behind introducing the function M(x)𝑀𝑥M(x)italic_M ( italic_x ) (As defined in (2.4.1)) was its underlying relation to the location of the zeros of the Riemann Zeta Function ζ(s)𝜁𝑠\zeta(s)italic_ζ ( italic_s ), the reason being largely due to it’s consequences for the distribution of the primes, also hailed as one of the most important unsolved problems in Analytic Number Theory. We shall be working with a particular order estimate of M(x)𝑀𝑥M(x)italic_M ( italic_x ) in later sections of the text, although readers are encouraged to consult [9], [11], [15] and [16] for further details.

2.5 Some Necessary Derivations

Proposition 2.5.1.

The Dirichlet Series Representation [3, Theorem 3.13 , pp. 62] for μ(n)𝜇𝑛\mu(n)italic_μ ( italic_n ) is given by:

n=1μ(n)ns=1ζ(s) , (s)>1.formulae-sequencesuperscriptsubscript𝑛1𝜇𝑛superscript𝑛𝑠1𝜁𝑠 , 𝑠1\displaystyle\sum_{n=1}^{\infty}\frac{\mu(n)}{n^{s}}=\frac{1}{\zeta(s)}\hskip 2% 0.0pt\mbox{ , }\Re(s)>1.∑ start_POSTSUBSCRIPT italic_n = 1 end_POSTSUBSCRIPT start_POSTSUPERSCRIPT ∞ end_POSTSUPERSCRIPT divide start_ARG italic_μ ( italic_n ) end_ARG start_ARG italic_n start_POSTSUPERSCRIPT italic_s end_POSTSUPERSCRIPT end_ARG = divide start_ARG 1 end_ARG start_ARG italic_ζ ( italic_s ) end_ARG , roman_ℜ ( italic_s ) > 1 . (2.11)

ζ(s)𝜁𝑠\zeta(s)italic_ζ ( italic_s ) denoting the Riemann Zeta Function.

Proposition 2.5.2.

The following order estimates hold true:

  1. (i)𝑖(i)( italic_i )

    dxμ(d)d=O(1logx)subscript𝑑𝑥𝜇𝑑𝑑𝑂1𝑥\sum\limits_{d\leq x}\frac{\mu(d)}{d}=O\left(\frac{1}{\log x}\right)∑ start_POSTSUBSCRIPT italic_d ≤ italic_x end_POSTSUBSCRIPT divide start_ARG italic_μ ( italic_d ) end_ARG start_ARG italic_d end_ARG = italic_O ( divide start_ARG 1 end_ARG start_ARG roman_log italic_x end_ARG )

  2. (ii)𝑖𝑖(ii)( italic_i italic_i )

    dxμ(d)log(d)d=O(1)subscript𝑑𝑥𝜇𝑑𝑑𝑑𝑂1\sum\limits_{d\leq x}\frac{\mu(d)\log(d)}{d}=O(1)∑ start_POSTSUBSCRIPT italic_d ≤ italic_x end_POSTSUBSCRIPT divide start_ARG italic_μ ( italic_d ) roman_log ( italic_d ) end_ARG start_ARG italic_d end_ARG = italic_O ( 1 )

  3. (iii)𝑖𝑖𝑖(iii)( italic_i italic_i italic_i )

    dxμ(d)=M(x)=O(x)subscript𝑑𝑥𝜇𝑑𝑀𝑥𝑂𝑥\sum\limits_{d\leq x}\mu(d)=M(x)=O(\sqrt{x})∑ start_POSTSUBSCRIPT italic_d ≤ italic_x end_POSTSUBSCRIPT italic_μ ( italic_d ) = italic_M ( italic_x ) = italic_O ( square-root start_ARG italic_x end_ARG )

Proof.
  1. (i)𝑖(i)( italic_i )

    Near s=1𝑠1s=1italic_s = 1, we have the expansion for ζ(s)𝜁𝑠\zeta(s)italic_ζ ( italic_s ):

    ζ(s)=1s1+γ+O(s1)𝜁𝑠1𝑠1𝛾𝑂𝑠1\zeta(s)=\frac{1}{s-1}+\gamma+O(s-1)italic_ζ ( italic_s ) = divide start_ARG 1 end_ARG start_ARG italic_s - 1 end_ARG + italic_γ + italic_O ( italic_s - 1 )

    Where, γ𝛾\gammaitalic_γ denotes the Euler Constant. Thus,

    1ζ(s)=(s1)γ(s1)2+O((s1)3)1𝜁𝑠𝑠1𝛾superscript𝑠12𝑂superscript𝑠13\frac{1}{\zeta(s)}=(s-1)-\gamma(s-1)^{2}+O((s-1)^{3})divide start_ARG 1 end_ARG start_ARG italic_ζ ( italic_s ) end_ARG = ( italic_s - 1 ) - italic_γ ( italic_s - 1 ) start_POSTSUPERSCRIPT 2 end_POSTSUPERSCRIPT + italic_O ( ( italic_s - 1 ) start_POSTSUPERSCRIPT 3 end_POSTSUPERSCRIPT )

    Using Perron’s Formula, we have:

    nxμ(n)n=12πiciTc+iT1ζ(s)xss𝑑s+O(xlog2xT)subscript𝑛𝑥𝜇𝑛𝑛12𝜋𝑖superscriptsubscript𝑐𝑖𝑇𝑐𝑖𝑇1𝜁𝑠superscript𝑥𝑠𝑠differential-d𝑠𝑂𝑥superscript2𝑥𝑇\displaystyle\sum_{n\leq x}\frac{\mu(n)}{n}=\frac{1}{2\pi i}\int\limits_{c-iT}% ^{c+iT}\frac{1}{\zeta(s)}\frac{x^{s}}{s}\,ds+O\left(\frac{x\log^{2}x}{T}\right)∑ start_POSTSUBSCRIPT italic_n ≤ italic_x end_POSTSUBSCRIPT divide start_ARG italic_μ ( italic_n ) end_ARG start_ARG italic_n end_ARG = divide start_ARG 1 end_ARG start_ARG 2 italic_π italic_i end_ARG ∫ start_POSTSUBSCRIPT italic_c - italic_i italic_T end_POSTSUBSCRIPT start_POSTSUPERSCRIPT italic_c + italic_i italic_T end_POSTSUPERSCRIPT divide start_ARG 1 end_ARG start_ARG italic_ζ ( italic_s ) end_ARG divide start_ARG italic_x start_POSTSUPERSCRIPT italic_s end_POSTSUPERSCRIPT end_ARG start_ARG italic_s end_ARG italic_d italic_s + italic_O ( divide start_ARG italic_x roman_log start_POSTSUPERSCRIPT 2 end_POSTSUPERSCRIPT italic_x end_ARG start_ARG italic_T end_ARG ) (2.12)

    where c>1𝑐1c>1italic_c > 1 and T𝑇Titalic_T is a parameter to be chosen later.

    We evaluate the integral using the following steps.

    Step 1: Integral around s=1𝑠1s=1italic_s = 1

    Consider a small semicircle ΓΓ\Gammaroman_Γ (say) of radius ϵitalic-ϵ\epsilonitalic_ϵ around s=1𝑠1s=1italic_s = 1 having the following parametrization, s=1+ϵeiθ𝑠1italic-ϵsuperscript𝑒𝑖𝜃s=1+\epsilon e^{i\theta}italic_s = 1 + italic_ϵ italic_e start_POSTSUPERSCRIPT italic_i italic_θ end_POSTSUPERSCRIPT, π/2θπ/2𝜋2𝜃𝜋2-\pi/2\leq\theta\leq\pi/2- italic_π / 2 ≤ italic_θ ≤ italic_π / 2 .

    In this region,

    1ζ(s)=ϵeiθγϵ2e2iθ+O(ϵ3)1𝜁𝑠italic-ϵsuperscript𝑒𝑖𝜃𝛾superscriptitalic-ϵ2superscript𝑒2𝑖𝜃𝑂superscriptitalic-ϵ3\displaystyle\frac{1}{\zeta(s)}=\epsilon e^{i\theta}-\gamma\epsilon^{2}e^{2i% \theta}+O(\epsilon^{3})divide start_ARG 1 end_ARG start_ARG italic_ζ ( italic_s ) end_ARG = italic_ϵ italic_e start_POSTSUPERSCRIPT italic_i italic_θ end_POSTSUPERSCRIPT - italic_γ italic_ϵ start_POSTSUPERSCRIPT 2 end_POSTSUPERSCRIPT italic_e start_POSTSUPERSCRIPT 2 italic_i italic_θ end_POSTSUPERSCRIPT + italic_O ( italic_ϵ start_POSTSUPERSCRIPT 3 end_POSTSUPERSCRIPT )

    and,

    xss=x1+ϵeiθ1+ϵeiθ=xxϵeiθ1+ϵeiθ=xeϵeiθlogx(1ϵeiθ+O(ϵ2))superscript𝑥𝑠𝑠superscript𝑥1italic-ϵsuperscript𝑒𝑖𝜃1italic-ϵsuperscript𝑒𝑖𝜃𝑥superscript𝑥italic-ϵsuperscript𝑒𝑖𝜃1italic-ϵsuperscript𝑒𝑖𝜃𝑥superscript𝑒italic-ϵsuperscript𝑒𝑖𝜃𝑥1italic-ϵsuperscript𝑒𝑖𝜃𝑂superscriptitalic-ϵ2\displaystyle\frac{x^{s}}{s}=\frac{x^{1+\epsilon e^{i\theta}}}{1+\epsilon e^{i% \theta}}=x\cdot\frac{x^{\epsilon e^{i\theta}}}{1+\epsilon e^{i\theta}}=x\cdot e% ^{\epsilon e^{i\theta}\log x}\left(1-\epsilon e^{i\theta}+O(\epsilon^{2})\right)divide start_ARG italic_x start_POSTSUPERSCRIPT italic_s end_POSTSUPERSCRIPT end_ARG start_ARG italic_s end_ARG = divide start_ARG italic_x start_POSTSUPERSCRIPT 1 + italic_ϵ italic_e start_POSTSUPERSCRIPT italic_i italic_θ end_POSTSUPERSCRIPT end_POSTSUPERSCRIPT end_ARG start_ARG 1 + italic_ϵ italic_e start_POSTSUPERSCRIPT italic_i italic_θ end_POSTSUPERSCRIPT end_ARG = italic_x ⋅ divide start_ARG italic_x start_POSTSUPERSCRIPT italic_ϵ italic_e start_POSTSUPERSCRIPT italic_i italic_θ end_POSTSUPERSCRIPT end_POSTSUPERSCRIPT end_ARG start_ARG 1 + italic_ϵ italic_e start_POSTSUPERSCRIPT italic_i italic_θ end_POSTSUPERSCRIPT end_ARG = italic_x ⋅ italic_e start_POSTSUPERSCRIPT italic_ϵ italic_e start_POSTSUPERSCRIPT italic_i italic_θ end_POSTSUPERSCRIPT roman_log italic_x end_POSTSUPERSCRIPT ( 1 - italic_ϵ italic_e start_POSTSUPERSCRIPT italic_i italic_θ end_POSTSUPERSCRIPT + italic_O ( italic_ϵ start_POSTSUPERSCRIPT 2 end_POSTSUPERSCRIPT ) )
    =x(1+ϵeiθlogx+O(ϵ2))(1ϵeiθ+O(ϵ2))absent𝑥1italic-ϵsuperscript𝑒𝑖𝜃𝑥𝑂superscriptitalic-ϵ21italic-ϵsuperscript𝑒𝑖𝜃𝑂superscriptitalic-ϵ2\displaystyle=x\left(1+\epsilon e^{i\theta}\log x+O(\epsilon^{2})\right)\left(% 1-\epsilon e^{i\theta}+O(\epsilon^{2})\right)= italic_x ( 1 + italic_ϵ italic_e start_POSTSUPERSCRIPT italic_i italic_θ end_POSTSUPERSCRIPT roman_log italic_x + italic_O ( italic_ϵ start_POSTSUPERSCRIPT 2 end_POSTSUPERSCRIPT ) ) ( 1 - italic_ϵ italic_e start_POSTSUPERSCRIPT italic_i italic_θ end_POSTSUPERSCRIPT + italic_O ( italic_ϵ start_POSTSUPERSCRIPT 2 end_POSTSUPERSCRIPT ) )
    =x(1+ϵeiθ(logx1)+O(ϵ2))absent𝑥1italic-ϵsuperscript𝑒𝑖𝜃𝑥1𝑂superscriptitalic-ϵ2\displaystyle=x\left(1+\epsilon e^{i\theta}(\log x-1)+O(\epsilon^{2})\right)= italic_x ( 1 + italic_ϵ italic_e start_POSTSUPERSCRIPT italic_i italic_θ end_POSTSUPERSCRIPT ( roman_log italic_x - 1 ) + italic_O ( italic_ϵ start_POSTSUPERSCRIPT 2 end_POSTSUPERSCRIPT ) )

    Thus, the integrand becomes:

    1ζ(s)xss=x(ϵeiθγϵ2e2iθ+O(ϵ3))(1+ϵeiθ(logx1)+O(ϵ2))1𝜁𝑠superscript𝑥𝑠𝑠𝑥italic-ϵsuperscript𝑒𝑖𝜃𝛾superscriptitalic-ϵ2superscript𝑒2𝑖𝜃𝑂superscriptitalic-ϵ31italic-ϵsuperscript𝑒𝑖𝜃𝑥1𝑂superscriptitalic-ϵ2\displaystyle\frac{1}{\zeta(s)}\frac{x^{s}}{s}=x\left(\epsilon e^{i\theta}-% \gamma\epsilon^{2}e^{2i\theta}+O(\epsilon^{3})\right)\left(1+\epsilon e^{i% \theta}(\log x-1)+O(\epsilon^{2})\right)divide start_ARG 1 end_ARG start_ARG italic_ζ ( italic_s ) end_ARG divide start_ARG italic_x start_POSTSUPERSCRIPT italic_s end_POSTSUPERSCRIPT end_ARG start_ARG italic_s end_ARG = italic_x ( italic_ϵ italic_e start_POSTSUPERSCRIPT italic_i italic_θ end_POSTSUPERSCRIPT - italic_γ italic_ϵ start_POSTSUPERSCRIPT 2 end_POSTSUPERSCRIPT italic_e start_POSTSUPERSCRIPT 2 italic_i italic_θ end_POSTSUPERSCRIPT + italic_O ( italic_ϵ start_POSTSUPERSCRIPT 3 end_POSTSUPERSCRIPT ) ) ( 1 + italic_ϵ italic_e start_POSTSUPERSCRIPT italic_i italic_θ end_POSTSUPERSCRIPT ( roman_log italic_x - 1 ) + italic_O ( italic_ϵ start_POSTSUPERSCRIPT 2 end_POSTSUPERSCRIPT ) )
    =x(ϵeiθ+ϵ2e2iθ(logx1γ)+O(ϵ3))absent𝑥italic-ϵsuperscript𝑒𝑖𝜃superscriptitalic-ϵ2superscript𝑒2𝑖𝜃𝑥1𝛾𝑂superscriptitalic-ϵ3\displaystyle\hskip 50.0pt=x\left(\epsilon e^{i\theta}+\epsilon^{2}e^{2i\theta% }(\log x-1-\gamma)+O(\epsilon^{3})\right)= italic_x ( italic_ϵ italic_e start_POSTSUPERSCRIPT italic_i italic_θ end_POSTSUPERSCRIPT + italic_ϵ start_POSTSUPERSCRIPT 2 end_POSTSUPERSCRIPT italic_e start_POSTSUPERSCRIPT 2 italic_i italic_θ end_POSTSUPERSCRIPT ( roman_log italic_x - 1 - italic_γ ) + italic_O ( italic_ϵ start_POSTSUPERSCRIPT 3 end_POSTSUPERSCRIPT ) )

    Therefore,

    Γ1ζ(s)xss=π/2π/2x(ϵeiθ+ϵ2e2iθ(logx1γ)+O(ϵ3))iϵeiθ𝑑θsubscriptΓ1𝜁𝑠superscript𝑥𝑠𝑠superscriptsubscript𝜋2𝜋2𝑥italic-ϵsuperscript𝑒𝑖𝜃superscriptitalic-ϵ2superscript𝑒2𝑖𝜃𝑥1𝛾𝑂superscriptitalic-ϵ3𝑖italic-ϵsuperscript𝑒𝑖𝜃differential-d𝜃\displaystyle\int\limits_{\Gamma}\frac{1}{\zeta(s)}\frac{x^{s}}{s}=\int\limits% _{-\pi/2}^{\pi/2}x\left(\epsilon e^{i\theta}+\epsilon^{2}e^{2i\theta}(\log x-1% -\gamma)+O(\epsilon^{3})\right)i\epsilon e^{i\theta}\,d\theta∫ start_POSTSUBSCRIPT roman_Γ end_POSTSUBSCRIPT divide start_ARG 1 end_ARG start_ARG italic_ζ ( italic_s ) end_ARG divide start_ARG italic_x start_POSTSUPERSCRIPT italic_s end_POSTSUPERSCRIPT end_ARG start_ARG italic_s end_ARG = ∫ start_POSTSUBSCRIPT - italic_π / 2 end_POSTSUBSCRIPT start_POSTSUPERSCRIPT italic_π / 2 end_POSTSUPERSCRIPT italic_x ( italic_ϵ italic_e start_POSTSUPERSCRIPT italic_i italic_θ end_POSTSUPERSCRIPT + italic_ϵ start_POSTSUPERSCRIPT 2 end_POSTSUPERSCRIPT italic_e start_POSTSUPERSCRIPT 2 italic_i italic_θ end_POSTSUPERSCRIPT ( roman_log italic_x - 1 - italic_γ ) + italic_O ( italic_ϵ start_POSTSUPERSCRIPT 3 end_POSTSUPERSCRIPT ) ) italic_i italic_ϵ italic_e start_POSTSUPERSCRIPT italic_i italic_θ end_POSTSUPERSCRIPT italic_d italic_θ
    =π/2π/2x(ϵ2e2iθ(logx1γ)+O(ϵ3))ieiθdθ[π/2π/2e2iθdθ=0]\displaystyle=\int\limits_{-\pi/2}^{\pi/2}x\left(\epsilon^{2}e^{2i\theta}(\log x% -1-\gamma)+O(\epsilon^{3})\right)ie^{i\theta}\,d\theta\hskip 20.0pt\left[% \because\int\limits_{-\pi/2}^{\pi/2}e^{2i\theta}\,d\theta=0\right]= ∫ start_POSTSUBSCRIPT - italic_π / 2 end_POSTSUBSCRIPT start_POSTSUPERSCRIPT italic_π / 2 end_POSTSUPERSCRIPT italic_x ( italic_ϵ start_POSTSUPERSCRIPT 2 end_POSTSUPERSCRIPT italic_e start_POSTSUPERSCRIPT 2 italic_i italic_θ end_POSTSUPERSCRIPT ( roman_log italic_x - 1 - italic_γ ) + italic_O ( italic_ϵ start_POSTSUPERSCRIPT 3 end_POSTSUPERSCRIPT ) ) italic_i italic_e start_POSTSUPERSCRIPT italic_i italic_θ end_POSTSUPERSCRIPT italic_d italic_θ [ ∵ ∫ start_POSTSUBSCRIPT - italic_π / 2 end_POSTSUBSCRIPT start_POSTSUPERSCRIPT italic_π / 2 end_POSTSUPERSCRIPT italic_e start_POSTSUPERSCRIPT 2 italic_i italic_θ end_POSTSUPERSCRIPT italic_d italic_θ = 0 ]
    =ϵ2x(logx1γ)π/2π/2ie3iθ𝑑θ+O(ϵ3)absentsuperscriptitalic-ϵ2𝑥𝑥1𝛾superscriptsubscript𝜋2𝜋2𝑖superscript𝑒3𝑖𝜃differential-d𝜃𝑂superscriptitalic-ϵ3\displaystyle=\epsilon^{2}x(\log x-1-\gamma)\int\limits_{-\pi/2}^{\pi/2}ie^{3i% \theta}\,d\theta+O(\epsilon^{3})= italic_ϵ start_POSTSUPERSCRIPT 2 end_POSTSUPERSCRIPT italic_x ( roman_log italic_x - 1 - italic_γ ) ∫ start_POSTSUBSCRIPT - italic_π / 2 end_POSTSUBSCRIPT start_POSTSUPERSCRIPT italic_π / 2 end_POSTSUPERSCRIPT italic_i italic_e start_POSTSUPERSCRIPT 3 italic_i italic_θ end_POSTSUPERSCRIPT italic_d italic_θ + italic_O ( italic_ϵ start_POSTSUPERSCRIPT 3 end_POSTSUPERSCRIPT )

    Again, the integral of e3iθsuperscript𝑒3𝑖𝜃e^{3i\theta}italic_e start_POSTSUPERSCRIPT 3 italic_i italic_θ end_POSTSUPERSCRIPT over a symmetric interval around zero is zero. Therefore, the integral around the small semicircle contributes a negligible amount of O(ϵ3x)𝑂superscriptitalic-ϵ3𝑥O(\epsilon^{3}x)italic_O ( italic_ϵ start_POSTSUPERSCRIPT 3 end_POSTSUPERSCRIPT italic_x ).

    Step 2: Integral along the vertical line s=c+it𝑠𝑐𝑖𝑡s=c+ititalic_s = italic_c + italic_i italic_t

    For the part of the integral along the vertical line, say, L𝐿Litalic_L: s=c+it𝑠𝑐𝑖𝑡s=c+ititalic_s = italic_c + italic_i italic_t, TtT𝑇𝑡𝑇-T\leq t\leq T- italic_T ≤ italic_t ≤ italic_T, where c>1𝑐1c>1italic_c > 1, i.e.,

    ciTc+iT1ζ(s)xss𝑑ssuperscriptsubscript𝑐𝑖𝑇𝑐𝑖𝑇1𝜁𝑠superscript𝑥𝑠𝑠differential-d𝑠\displaystyle\int\limits_{c-iT}^{c+iT}\frac{1}{\zeta(s)}\frac{x^{s}}{s}\,ds∫ start_POSTSUBSCRIPT italic_c - italic_i italic_T end_POSTSUBSCRIPT start_POSTSUPERSCRIPT italic_c + italic_i italic_T end_POSTSUPERSCRIPT divide start_ARG 1 end_ARG start_ARG italic_ζ ( italic_s ) end_ARG divide start_ARG italic_x start_POSTSUPERSCRIPT italic_s end_POSTSUPERSCRIPT end_ARG start_ARG italic_s end_ARG italic_d italic_s

    It can indeed be verified that, 1ζ(s)1𝜁𝑠\frac{1}{\zeta(s)}divide start_ARG 1 end_ARG start_ARG italic_ζ ( italic_s ) end_ARG is bounded on L𝐿Litalic_L as, ζ(s)𝜁𝑠\zeta(s)italic_ζ ( italic_s ) does not have any pole for Re(s)>1Re𝑠1\operatorname{Re}(s)>1roman_Re ( italic_s ) > 1. Specifically, for s=c+it𝑠𝑐𝑖𝑡s=c+ititalic_s = italic_c + italic_i italic_t with c>1𝑐1c>1italic_c > 1, ζ(s)𝜁𝑠\zeta(s)italic_ζ ( italic_s ) is bounded away from zero, so 1ζ(s)1𝜁𝑠\frac{1}{\zeta(s)}divide start_ARG 1 end_ARG start_ARG italic_ζ ( italic_s ) end_ARG is bounded.

    An appropriate choice of T=x𝑇𝑥T=\sqrt{x}italic_T = square-root start_ARG italic_x end_ARG gives us a bound on the integral:

    | ciTc+iT1ζ(s)xc+itc+it𝑑s|TT|1ζ(c+it)xcxitc+it|𝑑tKxcTT1c2+t2𝑑tKxc2Tc superscriptsubscript𝑐𝑖𝑇𝑐𝑖𝑇1𝜁𝑠superscript𝑥𝑐𝑖𝑡𝑐𝑖𝑡differential-d𝑠superscriptsubscript𝑇𝑇1𝜁𝑐𝑖𝑡superscript𝑥𝑐superscript𝑥𝑖𝑡𝑐𝑖𝑡differential-d𝑡𝐾superscript𝑥𝑐superscriptsubscript𝑇𝑇1superscript𝑐2superscript𝑡2differential-d𝑡𝐾superscript𝑥𝑐2𝑇𝑐\displaystyle\left|\mbox{ }\int\limits_{c-iT}^{c+iT}\frac{1}{\zeta(s)}\frac{x^% {c+it}}{c+it}\,ds\right|\leq\int\limits_{-T}^{T}\left|\frac{1}{\zeta(c+it)}% \frac{x^{c}x^{it}}{c+it}\right|dt\leq Kx^{c}\int\limits_{-T}^{T}\frac{1}{\sqrt% {c^{2}+t^{2}}}\,dt\leq Kx^{c}\cdot\frac{2T}{c}| ∫ start_POSTSUBSCRIPT italic_c - italic_i italic_T end_POSTSUBSCRIPT start_POSTSUPERSCRIPT italic_c + italic_i italic_T end_POSTSUPERSCRIPT divide start_ARG 1 end_ARG start_ARG italic_ζ ( italic_s ) end_ARG divide start_ARG italic_x start_POSTSUPERSCRIPT italic_c + italic_i italic_t end_POSTSUPERSCRIPT end_ARG start_ARG italic_c + italic_i italic_t end_ARG italic_d italic_s | ≤ ∫ start_POSTSUBSCRIPT - italic_T end_POSTSUBSCRIPT start_POSTSUPERSCRIPT italic_T end_POSTSUPERSCRIPT | divide start_ARG 1 end_ARG start_ARG italic_ζ ( italic_c + italic_i italic_t ) end_ARG divide start_ARG italic_x start_POSTSUPERSCRIPT italic_c end_POSTSUPERSCRIPT italic_x start_POSTSUPERSCRIPT italic_i italic_t end_POSTSUPERSCRIPT end_ARG start_ARG italic_c + italic_i italic_t end_ARG | italic_d italic_t ≤ italic_K italic_x start_POSTSUPERSCRIPT italic_c end_POSTSUPERSCRIPT ∫ start_POSTSUBSCRIPT - italic_T end_POSTSUBSCRIPT start_POSTSUPERSCRIPT italic_T end_POSTSUPERSCRIPT divide start_ARG 1 end_ARG start_ARG square-root start_ARG italic_c start_POSTSUPERSCRIPT 2 end_POSTSUPERSCRIPT + italic_t start_POSTSUPERSCRIPT 2 end_POSTSUPERSCRIPT end_ARG end_ARG italic_d italic_t ≤ italic_K italic_x start_POSTSUPERSCRIPT italic_c end_POSTSUPERSCRIPT ⋅ divide start_ARG 2 italic_T end_ARG start_ARG italic_c end_ARG

    Since 1ζ(c+it)1𝜁𝑐𝑖𝑡\frac{1}{\zeta(c+it)}divide start_ARG 1 end_ARG start_ARG italic_ζ ( italic_c + italic_i italic_t ) end_ARG is bounded by some constant K𝐾Kitalic_K and |xit|=1superscript𝑥𝑖𝑡1\left|x^{it}\right|=1| italic_x start_POSTSUPERSCRIPT italic_i italic_t end_POSTSUPERSCRIPT | = 1. Subsequently,

    |L1ζ(s)xss𝑑s|=| ciTc+iT1ζ(s)xss𝑑s|Kxc2xc=O(xc12)subscript𝐿1𝜁𝑠superscript𝑥𝑠𝑠differential-d𝑠 superscriptsubscript𝑐𝑖𝑇𝑐𝑖𝑇1𝜁𝑠superscript𝑥𝑠𝑠differential-d𝑠𝐾superscript𝑥𝑐2𝑥𝑐𝑂superscript𝑥𝑐12\displaystyle\left|\int\limits_{L}\frac{1}{\zeta(s)}\frac{x^{s}}{s}\,ds\right|% =\left|\mbox{ }\int\limits_{c-iT}^{c+iT}\frac{1}{\zeta(s)}\frac{x^{s}}{s}\,ds% \right|\leq Kx^{c}\cdot\frac{2\sqrt{x}}{c}=O\left(x^{c-\frac{1}{2}}\right)| ∫ start_POSTSUBSCRIPT italic_L end_POSTSUBSCRIPT divide start_ARG 1 end_ARG start_ARG italic_ζ ( italic_s ) end_ARG divide start_ARG italic_x start_POSTSUPERSCRIPT italic_s end_POSTSUPERSCRIPT end_ARG start_ARG italic_s end_ARG italic_d italic_s | = | ∫ start_POSTSUBSCRIPT italic_c - italic_i italic_T end_POSTSUBSCRIPT start_POSTSUPERSCRIPT italic_c + italic_i italic_T end_POSTSUPERSCRIPT divide start_ARG 1 end_ARG start_ARG italic_ζ ( italic_s ) end_ARG divide start_ARG italic_x start_POSTSUPERSCRIPT italic_s end_POSTSUPERSCRIPT end_ARG start_ARG italic_s end_ARG italic_d italic_s | ≤ italic_K italic_x start_POSTSUPERSCRIPT italic_c end_POSTSUPERSCRIPT ⋅ divide start_ARG 2 square-root start_ARG italic_x end_ARG end_ARG start_ARG italic_c end_ARG = italic_O ( italic_x start_POSTSUPERSCRIPT italic_c - divide start_ARG 1 end_ARG start_ARG 2 end_ARG end_POSTSUPERSCRIPT )

    Since c>1𝑐1c>1italic_c > 1, c12>12𝑐1212c-\frac{1}{2}>\frac{1}{2}italic_c - divide start_ARG 1 end_ARG start_ARG 2 end_ARG > divide start_ARG 1 end_ARG start_ARG 2 end_ARG, and for large x𝑥xitalic_x, this term is small.

    Step 3: Error term from the integral

    The total error term combining both parts is:

    O(xlog2xT)=O(xlog2xx)=O(x1/2log2x)𝑂𝑥superscript2𝑥𝑇𝑂𝑥superscript2𝑥𝑥𝑂superscript𝑥12superscript2𝑥\displaystyle O\left(\frac{x\log^{2}x}{T}\right)=O\left(\frac{x\log^{2}x}{% \sqrt{x}}\right)=O(x^{1/2}\log^{2}x)italic_O ( divide start_ARG italic_x roman_log start_POSTSUPERSCRIPT 2 end_POSTSUPERSCRIPT italic_x end_ARG start_ARG italic_T end_ARG ) = italic_O ( divide start_ARG italic_x roman_log start_POSTSUPERSCRIPT 2 end_POSTSUPERSCRIPT italic_x end_ARG start_ARG square-root start_ARG italic_x end_ARG end_ARG ) = italic_O ( italic_x start_POSTSUPERSCRIPT 1 / 2 end_POSTSUPERSCRIPT roman_log start_POSTSUPERSCRIPT 2 end_POSTSUPERSCRIPT italic_x )

    It can be observed that, the residue term around s=1𝑠1s=1italic_s = 1 contributes to x𝑥xitalic_x, and the error terms contribute to =O(ϵ3x)+O(x1/2log2x)absent𝑂superscriptitalic-ϵ3𝑥𝑂superscript𝑥12superscript2𝑥=O(\epsilon^{3}x)+O(x^{1/2}\log^{2}x)= italic_O ( italic_ϵ start_POSTSUPERSCRIPT 3 end_POSTSUPERSCRIPT italic_x ) + italic_O ( italic_x start_POSTSUPERSCRIPT 1 / 2 end_POSTSUPERSCRIPT roman_log start_POSTSUPERSCRIPT 2 end_POSTSUPERSCRIPT italic_x ).

    Choosing ϵitalic-ϵ\epsilonitalic_ϵ small enough (such as ϵ=x1/6italic-ϵsuperscript𝑥16\epsilon=x^{-1/6}italic_ϵ = italic_x start_POSTSUPERSCRIPT - 1 / 6 end_POSTSUPERSCRIPT),

    O(ϵ3x)=O(x11/2)=O(x1/2)𝑂superscriptitalic-ϵ3𝑥𝑂superscript𝑥112𝑂superscript𝑥12\displaystyle O(\epsilon^{3}x)=O(x^{1-1/2})=O(x^{1/2})italic_O ( italic_ϵ start_POSTSUPERSCRIPT 3 end_POSTSUPERSCRIPT italic_x ) = italic_O ( italic_x start_POSTSUPERSCRIPT 1 - 1 / 2 end_POSTSUPERSCRIPT ) = italic_O ( italic_x start_POSTSUPERSCRIPT 1 / 2 end_POSTSUPERSCRIPT )

    Thus, combining all terms and dividing by x𝑥xitalic_x to normalize, we obtain.

    dxμ(d)d=x+O(x1/2log2x)=O(1logx)subscript𝑑𝑥𝜇𝑑𝑑𝑥𝑂superscript𝑥12superscript2𝑥𝑂1𝑥\displaystyle\sum_{d\leq x}\frac{\mu(d)}{d}=x+O(x^{1/2}\log^{2}x)=O\left(\frac% {1}{\log x}\right)∑ start_POSTSUBSCRIPT italic_d ≤ italic_x end_POSTSUBSCRIPT divide start_ARG italic_μ ( italic_d ) end_ARG start_ARG italic_d end_ARG = italic_x + italic_O ( italic_x start_POSTSUPERSCRIPT 1 / 2 end_POSTSUPERSCRIPT roman_log start_POSTSUPERSCRIPT 2 end_POSTSUPERSCRIPT italic_x ) = italic_O ( divide start_ARG 1 end_ARG start_ARG roman_log italic_x end_ARG )
  2. (ii)𝑖𝑖(ii)( italic_i italic_i )

    As for the proof, we can use Perron’s formula for any arithmetic function a(n)𝑎𝑛a(n)italic_a ( italic_n ),

    nxa(n)=12πiciTc+iT(n=1a(n)ns)xss𝑑s+O(n=1|a(n)|ncmin(1,xT|s|))subscript𝑛𝑥𝑎𝑛12𝜋𝑖superscriptsubscript𝑐𝑖𝑇𝑐𝑖𝑇superscriptsubscript𝑛1𝑎𝑛superscript𝑛𝑠superscript𝑥𝑠𝑠differential-d𝑠𝑂superscriptsubscript𝑛1𝑎𝑛superscript𝑛𝑐1𝑥𝑇𝑠\displaystyle\sum_{n\leq x}a(n)=\frac{1}{2\pi i}\int\limits_{c-iT}^{c+iT}\left% (\sum_{n=1}^{\infty}\frac{a(n)}{n^{s}}\right)\frac{x^{s}}{s}\,ds+O\left(\sum_{% n=1}^{\infty}\frac{|a(n)|}{n^{c}}\min\left(1,\frac{x}{T|s|}\right)\right)∑ start_POSTSUBSCRIPT italic_n ≤ italic_x end_POSTSUBSCRIPT italic_a ( italic_n ) = divide start_ARG 1 end_ARG start_ARG 2 italic_π italic_i end_ARG ∫ start_POSTSUBSCRIPT italic_c - italic_i italic_T end_POSTSUBSCRIPT start_POSTSUPERSCRIPT italic_c + italic_i italic_T end_POSTSUPERSCRIPT ( ∑ start_POSTSUBSCRIPT italic_n = 1 end_POSTSUBSCRIPT start_POSTSUPERSCRIPT ∞ end_POSTSUPERSCRIPT divide start_ARG italic_a ( italic_n ) end_ARG start_ARG italic_n start_POSTSUPERSCRIPT italic_s end_POSTSUPERSCRIPT end_ARG ) divide start_ARG italic_x start_POSTSUPERSCRIPT italic_s end_POSTSUPERSCRIPT end_ARG start_ARG italic_s end_ARG italic_d italic_s + italic_O ( ∑ start_POSTSUBSCRIPT italic_n = 1 end_POSTSUBSCRIPT start_POSTSUPERSCRIPT ∞ end_POSTSUPERSCRIPT divide start_ARG | italic_a ( italic_n ) | end_ARG start_ARG italic_n start_POSTSUPERSCRIPT italic_c end_POSTSUPERSCRIPT end_ARG roman_min ( 1 , divide start_ARG italic_x end_ARG start_ARG italic_T | italic_s | end_ARG ) ) (2.13)

    where c>1𝑐1c>1italic_c > 1 and T𝑇Titalic_T be a suitably chosen parameter.

    Consider the Dirichlet series involving μ(d)logd𝜇𝑑𝑑\mu(d)\log ditalic_μ ( italic_d ) roman_log italic_d:

    ζ(s)ζ2(s)=n=1μ(n)lognnssuperscript𝜁𝑠superscript𝜁2𝑠superscriptsubscript𝑛1𝜇𝑛𝑛superscript𝑛𝑠\displaystyle-\frac{\zeta^{\prime}(s)}{\zeta^{2}(s)}=\sum_{n=1}^{\infty}\frac{% \mu(n)\log n}{n^{s}}- divide start_ARG italic_ζ start_POSTSUPERSCRIPT ′ end_POSTSUPERSCRIPT ( italic_s ) end_ARG start_ARG italic_ζ start_POSTSUPERSCRIPT 2 end_POSTSUPERSCRIPT ( italic_s ) end_ARG = ∑ start_POSTSUBSCRIPT italic_n = 1 end_POSTSUBSCRIPT start_POSTSUPERSCRIPT ∞ end_POSTSUPERSCRIPT divide start_ARG italic_μ ( italic_n ) roman_log italic_n end_ARG start_ARG italic_n start_POSTSUPERSCRIPT italic_s end_POSTSUPERSCRIPT end_ARG

    Applying this to (2.13):

    dxμ(d)logdd=12πiciTc+iTζ(s)ζ2(s)xss𝑑s+O(d=1|μ(d)logd|dcmin(1,xT|s|))subscript𝑑𝑥𝜇𝑑𝑑𝑑12𝜋𝑖superscriptsubscript𝑐𝑖𝑇𝑐𝑖𝑇superscript𝜁𝑠superscript𝜁2𝑠superscript𝑥𝑠𝑠differential-d𝑠𝑂superscriptsubscript𝑑1𝜇𝑑𝑑superscript𝑑𝑐1𝑥𝑇𝑠\displaystyle\sum_{d\leq x}\frac{\mu(d)\log d}{d}=-\frac{1}{2\pi i}\int\limits% _{c-iT}^{c+iT}\frac{\zeta^{\prime}(s)}{\zeta^{2}(s)}\frac{x^{s}}{s}\,ds+O\left% (\sum_{d=1}^{\infty}\frac{|\mu(d)\log d|}{d^{c}}\min\left(1,\frac{x}{T|s|}% \right)\right)∑ start_POSTSUBSCRIPT italic_d ≤ italic_x end_POSTSUBSCRIPT divide start_ARG italic_μ ( italic_d ) roman_log italic_d end_ARG start_ARG italic_d end_ARG = - divide start_ARG 1 end_ARG start_ARG 2 italic_π italic_i end_ARG ∫ start_POSTSUBSCRIPT italic_c - italic_i italic_T end_POSTSUBSCRIPT start_POSTSUPERSCRIPT italic_c + italic_i italic_T end_POSTSUPERSCRIPT divide start_ARG italic_ζ start_POSTSUPERSCRIPT ′ end_POSTSUPERSCRIPT ( italic_s ) end_ARG start_ARG italic_ζ start_POSTSUPERSCRIPT 2 end_POSTSUPERSCRIPT ( italic_s ) end_ARG divide start_ARG italic_x start_POSTSUPERSCRIPT italic_s end_POSTSUPERSCRIPT end_ARG start_ARG italic_s end_ARG italic_d italic_s + italic_O ( ∑ start_POSTSUBSCRIPT italic_d = 1 end_POSTSUBSCRIPT start_POSTSUPERSCRIPT ∞ end_POSTSUPERSCRIPT divide start_ARG | italic_μ ( italic_d ) roman_log italic_d | end_ARG start_ARG italic_d start_POSTSUPERSCRIPT italic_c end_POSTSUPERSCRIPT end_ARG roman_min ( 1 , divide start_ARG italic_x end_ARG start_ARG italic_T | italic_s | end_ARG ) )

    We thus study the following integral,

    12πiciTc+iTζ(s)ζ2(s)xss𝑑s12𝜋𝑖superscriptsubscript𝑐𝑖𝑇𝑐𝑖𝑇superscript𝜁𝑠superscript𝜁2𝑠superscript𝑥𝑠𝑠differential-d𝑠\displaystyle-\frac{1}{2\pi i}\int\limits_{c-iT}^{c+iT}\frac{\zeta^{\prime}(s)% }{\zeta^{2}(s)}\frac{x^{s}}{s}\,ds- divide start_ARG 1 end_ARG start_ARG 2 italic_π italic_i end_ARG ∫ start_POSTSUBSCRIPT italic_c - italic_i italic_T end_POSTSUBSCRIPT start_POSTSUPERSCRIPT italic_c + italic_i italic_T end_POSTSUPERSCRIPT divide start_ARG italic_ζ start_POSTSUPERSCRIPT ′ end_POSTSUPERSCRIPT ( italic_s ) end_ARG start_ARG italic_ζ start_POSTSUPERSCRIPT 2 end_POSTSUPERSCRIPT ( italic_s ) end_ARG divide start_ARG italic_x start_POSTSUPERSCRIPT italic_s end_POSTSUPERSCRIPT end_ARG start_ARG italic_s end_ARG italic_d italic_s

    using following steps.

    Step 1: Integral around s=1𝑠1s=1italic_s = 1

    Consider a small semicircle ΓΓ\Gammaroman_Γ (say) of radius ϵitalic-ϵ\epsilonitalic_ϵ around s=1𝑠1s=1italic_s = 1 having the following parametrization, s=1+ϵeiθ𝑠1italic-ϵsuperscript𝑒𝑖𝜃s=1+\epsilon e^{i\theta}italic_s = 1 + italic_ϵ italic_e start_POSTSUPERSCRIPT italic_i italic_θ end_POSTSUPERSCRIPT, π/2θπ/2𝜋2𝜃𝜋2-\pi/2\leq\theta\leq\pi/2- italic_π / 2 ≤ italic_θ ≤ italic_π / 2, and a priori from the fact that, near s=1𝑠1s=1italic_s = 1,

    ζ(s)=1s1+γ+O(s1)𝜁𝑠1𝑠1𝛾𝑂𝑠1\displaystyle\zeta(s)=\frac{1}{s-1}+\gamma+O(s-1)italic_ζ ( italic_s ) = divide start_ARG 1 end_ARG start_ARG italic_s - 1 end_ARG + italic_γ + italic_O ( italic_s - 1 )

    and,

    ζ(s)=1(s1)2+O(1)superscript𝜁𝑠1superscript𝑠12𝑂1\displaystyle\zeta^{\prime}(s)=-\frac{1}{(s-1)^{2}}+O(1)italic_ζ start_POSTSUPERSCRIPT ′ end_POSTSUPERSCRIPT ( italic_s ) = - divide start_ARG 1 end_ARG start_ARG ( italic_s - 1 ) start_POSTSUPERSCRIPT 2 end_POSTSUPERSCRIPT end_ARG + italic_O ( 1 )

    Thus, on ΓΓ\Gammaroman_Γ, we have,

    ζ(s)ζ2(s)=1(ϵeiθ)2+O(1)(1ϵeiθ+γ+O(ϵ))2=ϵ1eiθ+O(ϵ)superscript𝜁𝑠superscript𝜁2𝑠1superscriptitalic-ϵsuperscript𝑒𝑖𝜃2𝑂1superscript1italic-ϵsuperscript𝑒𝑖𝜃𝛾𝑂italic-ϵ2superscriptitalic-ϵ1superscript𝑒𝑖𝜃𝑂italic-ϵ\displaystyle\frac{\zeta^{\prime}(s)}{\zeta^{2}(s)}=\frac{-\frac{1}{(\epsilon e% ^{i\theta})^{2}}+O(1)}{\left(\frac{1}{\epsilon e^{i\theta}}+\gamma+O(\epsilon)% \right)^{2}}=-\epsilon^{-1}e^{-i\theta}+O(\epsilon)divide start_ARG italic_ζ start_POSTSUPERSCRIPT ′ end_POSTSUPERSCRIPT ( italic_s ) end_ARG start_ARG italic_ζ start_POSTSUPERSCRIPT 2 end_POSTSUPERSCRIPT ( italic_s ) end_ARG = divide start_ARG - divide start_ARG 1 end_ARG start_ARG ( italic_ϵ italic_e start_POSTSUPERSCRIPT italic_i italic_θ end_POSTSUPERSCRIPT ) start_POSTSUPERSCRIPT 2 end_POSTSUPERSCRIPT end_ARG + italic_O ( 1 ) end_ARG start_ARG ( divide start_ARG 1 end_ARG start_ARG italic_ϵ italic_e start_POSTSUPERSCRIPT italic_i italic_θ end_POSTSUPERSCRIPT end_ARG + italic_γ + italic_O ( italic_ϵ ) ) start_POSTSUPERSCRIPT 2 end_POSTSUPERSCRIPT end_ARG = - italic_ϵ start_POSTSUPERSCRIPT - 1 end_POSTSUPERSCRIPT italic_e start_POSTSUPERSCRIPT - italic_i italic_θ end_POSTSUPERSCRIPT + italic_O ( italic_ϵ )

    Hence, the integrand becomes,

    12πiπ/2π/2(ϵ1eiθ+O(ϵ))x1+ϵeiθ(1+ϵeiθ)iϵeiθ𝑑θ12𝜋𝑖superscriptsubscript𝜋2𝜋2superscriptitalic-ϵ1superscript𝑒𝑖𝜃𝑂italic-ϵsuperscript𝑥1italic-ϵsuperscript𝑒𝑖𝜃1italic-ϵsuperscript𝑒𝑖𝜃𝑖italic-ϵsuperscript𝑒𝑖𝜃differential-d𝜃\displaystyle-\frac{1}{2\pi i}\int\limits_{-\pi/2}^{\pi/2}\left(-\epsilon^{-1}% e^{-i\theta}+O(\epsilon)\right)\frac{x^{1+\epsilon e^{i\theta}}}{(1+\epsilon e% ^{i\theta})}\,i\epsilon e^{i\theta}d\theta- divide start_ARG 1 end_ARG start_ARG 2 italic_π italic_i end_ARG ∫ start_POSTSUBSCRIPT - italic_π / 2 end_POSTSUBSCRIPT start_POSTSUPERSCRIPT italic_π / 2 end_POSTSUPERSCRIPT ( - italic_ϵ start_POSTSUPERSCRIPT - 1 end_POSTSUPERSCRIPT italic_e start_POSTSUPERSCRIPT - italic_i italic_θ end_POSTSUPERSCRIPT + italic_O ( italic_ϵ ) ) divide start_ARG italic_x start_POSTSUPERSCRIPT 1 + italic_ϵ italic_e start_POSTSUPERSCRIPT italic_i italic_θ end_POSTSUPERSCRIPT end_POSTSUPERSCRIPT end_ARG start_ARG ( 1 + italic_ϵ italic_e start_POSTSUPERSCRIPT italic_i italic_θ end_POSTSUPERSCRIPT ) end_ARG italic_i italic_ϵ italic_e start_POSTSUPERSCRIPT italic_i italic_θ end_POSTSUPERSCRIPT italic_d italic_θ
    =12πiπ/2π/2(ϵ1eiθ+O(ϵ)) . x(1+ϵeiθlogx+O(ϵ2))(1ϵeiθ+O(ϵ2))iϵeiθ𝑑θabsent12𝜋𝑖superscriptsubscript𝜋2𝜋2superscriptitalic-ϵ1superscript𝑒𝑖𝜃𝑂italic-ϵ . 𝑥1italic-ϵsuperscript𝑒𝑖𝜃𝑥𝑂superscriptitalic-ϵ21italic-ϵsuperscript𝑒𝑖𝜃𝑂superscriptitalic-ϵ2𝑖italic-ϵsuperscript𝑒𝑖𝜃differential-d𝜃\displaystyle=-\frac{1}{2\pi i}\int\limits_{-\pi/2}^{\pi/2}\left(-\epsilon^{-1% }e^{-i\theta}+O(\epsilon)\right)\mbox{ . }x\left(1+\epsilon e^{i\theta}\log x+% O(\epsilon^{2})\right)\left(1-\epsilon e^{i\theta}+O(\epsilon^{2})\right)i% \epsilon e^{i\theta}d\theta= - divide start_ARG 1 end_ARG start_ARG 2 italic_π italic_i end_ARG ∫ start_POSTSUBSCRIPT - italic_π / 2 end_POSTSUBSCRIPT start_POSTSUPERSCRIPT italic_π / 2 end_POSTSUPERSCRIPT ( - italic_ϵ start_POSTSUPERSCRIPT - 1 end_POSTSUPERSCRIPT italic_e start_POSTSUPERSCRIPT - italic_i italic_θ end_POSTSUPERSCRIPT + italic_O ( italic_ϵ ) ) . italic_x ( 1 + italic_ϵ italic_e start_POSTSUPERSCRIPT italic_i italic_θ end_POSTSUPERSCRIPT roman_log italic_x + italic_O ( italic_ϵ start_POSTSUPERSCRIPT 2 end_POSTSUPERSCRIPT ) ) ( 1 - italic_ϵ italic_e start_POSTSUPERSCRIPT italic_i italic_θ end_POSTSUPERSCRIPT + italic_O ( italic_ϵ start_POSTSUPERSCRIPT 2 end_POSTSUPERSCRIPT ) ) italic_i italic_ϵ italic_e start_POSTSUPERSCRIPT italic_i italic_θ end_POSTSUPERSCRIPT italic_d italic_θ
    =12ππ/2π/2(ϵ1eiθ+O(ϵ))(xϵeiθ(logx1)+O(ϵ2))iϵeiθ𝑑θabsent12𝜋superscriptsubscript𝜋2𝜋2superscriptitalic-ϵ1superscript𝑒𝑖𝜃𝑂italic-ϵ𝑥italic-ϵsuperscript𝑒𝑖𝜃𝑥1𝑂superscriptitalic-ϵ2𝑖italic-ϵsuperscript𝑒𝑖𝜃differential-d𝜃\displaystyle=-\frac{1}{2\pi}\int\limits_{-\pi/2}^{\pi/2}\left(-\epsilon^{-1}e% ^{-i\theta}+O(\epsilon)\right)\left(x\epsilon e^{i\theta}(\log x-1)+O(\epsilon% ^{2})\right)i\epsilon e^{i\theta}d\theta= - divide start_ARG 1 end_ARG start_ARG 2 italic_π end_ARG ∫ start_POSTSUBSCRIPT - italic_π / 2 end_POSTSUBSCRIPT start_POSTSUPERSCRIPT italic_π / 2 end_POSTSUPERSCRIPT ( - italic_ϵ start_POSTSUPERSCRIPT - 1 end_POSTSUPERSCRIPT italic_e start_POSTSUPERSCRIPT - italic_i italic_θ end_POSTSUPERSCRIPT + italic_O ( italic_ϵ ) ) ( italic_x italic_ϵ italic_e start_POSTSUPERSCRIPT italic_i italic_θ end_POSTSUPERSCRIPT ( roman_log italic_x - 1 ) + italic_O ( italic_ϵ start_POSTSUPERSCRIPT 2 end_POSTSUPERSCRIPT ) ) italic_i italic_ϵ italic_e start_POSTSUPERSCRIPT italic_i italic_θ end_POSTSUPERSCRIPT italic_d italic_θ
    =O(ϵ2x(logx1))absent𝑂superscriptitalic-ϵ2𝑥𝑥1\displaystyle=O(\epsilon^{2}x(\log x-1))= italic_O ( italic_ϵ start_POSTSUPERSCRIPT 2 end_POSTSUPERSCRIPT italic_x ( roman_log italic_x - 1 ) )

    ( Expanding and simplifying, the leading term integrates to zero due to symmetry of the integrand around zero ).

    Step 2: Integral along the vertical line s=c+it𝑠𝑐𝑖𝑡s=c+ititalic_s = italic_c + italic_i italic_t

    For the part of the integral along the vertical line, say, L𝐿Litalic_L: s=c+it𝑠𝑐𝑖𝑡s=c+ititalic_s = italic_c + italic_i italic_t, TtT𝑇𝑡𝑇-T\leq t\leq T- italic_T ≤ italic_t ≤ italic_T, where c>1𝑐1c>1italic_c > 1, i.e.,

    ciTc+iTζ(s)ζ2(s)xss𝑑ssuperscriptsubscript𝑐𝑖𝑇𝑐𝑖𝑇superscript𝜁𝑠superscript𝜁2𝑠superscript𝑥𝑠𝑠differential-d𝑠\displaystyle\int_{c-iT}^{c+iT}\frac{\zeta^{\prime}(s)}{\zeta^{2}(s)}\frac{x^{% s}}{s}\,ds∫ start_POSTSUBSCRIPT italic_c - italic_i italic_T end_POSTSUBSCRIPT start_POSTSUPERSCRIPT italic_c + italic_i italic_T end_POSTSUPERSCRIPT divide start_ARG italic_ζ start_POSTSUPERSCRIPT ′ end_POSTSUPERSCRIPT ( italic_s ) end_ARG start_ARG italic_ζ start_POSTSUPERSCRIPT 2 end_POSTSUPERSCRIPT ( italic_s ) end_ARG divide start_ARG italic_x start_POSTSUPERSCRIPT italic_s end_POSTSUPERSCRIPT end_ARG start_ARG italic_s end_ARG italic_d italic_s

    It can be checked that, ζ(s)ζ2(s)superscript𝜁𝑠superscript𝜁2𝑠\frac{\zeta^{\prime}(s)}{\zeta^{2}(s)}divide start_ARG italic_ζ start_POSTSUPERSCRIPT ′ end_POSTSUPERSCRIPT ( italic_s ) end_ARG start_ARG italic_ζ start_POSTSUPERSCRIPT 2 end_POSTSUPERSCRIPT ( italic_s ) end_ARG is bounded on L𝐿Litalic_L. Specifically, for s=c+it𝑠𝑐𝑖𝑡s=c+ititalic_s = italic_c + italic_i italic_t with c>1𝑐1c>1italic_c > 1, both ζ(s)𝜁𝑠\zeta(s)italic_ζ ( italic_s ) and ζ(s)superscript𝜁𝑠\zeta^{\prime}(s)italic_ζ start_POSTSUPERSCRIPT ′ end_POSTSUPERSCRIPT ( italic_s ) are bounded, so ζ(s)ζ2(s)superscript𝜁𝑠superscript𝜁2𝑠\frac{\zeta^{\prime}(s)}{\zeta^{2}(s)}divide start_ARG italic_ζ start_POSTSUPERSCRIPT ′ end_POSTSUPERSCRIPT ( italic_s ) end_ARG start_ARG italic_ζ start_POSTSUPERSCRIPT 2 end_POSTSUPERSCRIPT ( italic_s ) end_ARG is bounded by some constant K𝐾Kitalic_K. Hence,

    | ciTc+iTζ(s)ζ2(s)xc+itc+it𝑑s|KTTxcc2+t2𝑑tK . 2Tc superscriptsubscript𝑐𝑖𝑇𝑐𝑖𝑇superscript𝜁𝑠superscript𝜁2𝑠superscript𝑥𝑐𝑖𝑡𝑐𝑖𝑡differential-d𝑠𝐾superscriptsubscript𝑇𝑇superscript𝑥𝑐superscript𝑐2superscript𝑡2differential-d𝑡𝐾 . 2𝑇𝑐\displaystyle\left|\mbox{ }\int\limits_{c-iT}^{c+iT}\frac{\zeta^{\prime}(s)}{% \zeta^{2}(s)}\frac{x^{c+it}}{c+it}\,ds\right|\leq K\int_{-T}^{T}\frac{x^{c}}{% \sqrt{c^{2}+t^{2}}}\,dt\leq K\mbox{ . }\frac{2T}{c}| ∫ start_POSTSUBSCRIPT italic_c - italic_i italic_T end_POSTSUBSCRIPT start_POSTSUPERSCRIPT italic_c + italic_i italic_T end_POSTSUPERSCRIPT divide start_ARG italic_ζ start_POSTSUPERSCRIPT ′ end_POSTSUPERSCRIPT ( italic_s ) end_ARG start_ARG italic_ζ start_POSTSUPERSCRIPT 2 end_POSTSUPERSCRIPT ( italic_s ) end_ARG divide start_ARG italic_x start_POSTSUPERSCRIPT italic_c + italic_i italic_t end_POSTSUPERSCRIPT end_ARG start_ARG italic_c + italic_i italic_t end_ARG italic_d italic_s | ≤ italic_K ∫ start_POSTSUBSCRIPT - italic_T end_POSTSUBSCRIPT start_POSTSUPERSCRIPT italic_T end_POSTSUPERSCRIPT divide start_ARG italic_x start_POSTSUPERSCRIPT italic_c end_POSTSUPERSCRIPT end_ARG start_ARG square-root start_ARG italic_c start_POSTSUPERSCRIPT 2 end_POSTSUPERSCRIPT + italic_t start_POSTSUPERSCRIPT 2 end_POSTSUPERSCRIPT end_ARG end_ARG italic_d italic_t ≤ italic_K . divide start_ARG 2 italic_T end_ARG start_ARG italic_c end_ARG

    Choosing T=x𝑇𝑥T=\sqrt{x}italic_T = square-root start_ARG italic_x end_ARG,

    | ciTc+iTζ(s)ζ2(s)xss𝑑s|Kxc2xc=O(xc12) superscriptsubscript𝑐𝑖𝑇𝑐𝑖𝑇superscript𝜁𝑠superscript𝜁2𝑠superscript𝑥𝑠𝑠differential-d𝑠𝐾superscript𝑥𝑐2𝑥𝑐𝑂superscript𝑥𝑐12\displaystyle\left|\mbox{ }\int\limits_{c-iT}^{c+iT}\frac{\zeta^{\prime}(s)}{% \zeta^{2}(s)}\frac{x^{s}}{s}\,ds\right|\leq Kx^{c}\cdot\frac{2\sqrt{x}}{c}=O(x% ^{c-\frac{1}{2}})| ∫ start_POSTSUBSCRIPT italic_c - italic_i italic_T end_POSTSUBSCRIPT start_POSTSUPERSCRIPT italic_c + italic_i italic_T end_POSTSUPERSCRIPT divide start_ARG italic_ζ start_POSTSUPERSCRIPT ′ end_POSTSUPERSCRIPT ( italic_s ) end_ARG start_ARG italic_ζ start_POSTSUPERSCRIPT 2 end_POSTSUPERSCRIPT ( italic_s ) end_ARG divide start_ARG italic_x start_POSTSUPERSCRIPT italic_s end_POSTSUPERSCRIPT end_ARG start_ARG italic_s end_ARG italic_d italic_s | ≤ italic_K italic_x start_POSTSUPERSCRIPT italic_c end_POSTSUPERSCRIPT ⋅ divide start_ARG 2 square-root start_ARG italic_x end_ARG end_ARG start_ARG italic_c end_ARG = italic_O ( italic_x start_POSTSUPERSCRIPT italic_c - divide start_ARG 1 end_ARG start_ARG 2 end_ARG end_POSTSUPERSCRIPT )

    Since c>1𝑐1c>1italic_c > 1, c12>12𝑐1212c-\frac{1}{2}>\frac{1}{2}italic_c - divide start_ARG 1 end_ARG start_ARG 2 end_ARG > divide start_ARG 1 end_ARG start_ARG 2 end_ARG, and for large x𝑥xitalic_x, this term is small.

    Step 3: Error term from the integral

    The total error term combining both parts is:

    O(xlog2xT)=O(xlog2xx)=O(x1/2log2x)𝑂𝑥superscript2𝑥𝑇𝑂𝑥superscript2𝑥𝑥𝑂superscript𝑥12superscript2𝑥\displaystyle O\left(\frac{x\log^{2}x}{T}\right)=O\left(\frac{x\log^{2}x}{% \sqrt{x}}\right)=O(x^{1/2}\log^{2}x)italic_O ( divide start_ARG italic_x roman_log start_POSTSUPERSCRIPT 2 end_POSTSUPERSCRIPT italic_x end_ARG start_ARG italic_T end_ARG ) = italic_O ( divide start_ARG italic_x roman_log start_POSTSUPERSCRIPT 2 end_POSTSUPERSCRIPT italic_x end_ARG start_ARG square-root start_ARG italic_x end_ARG end_ARG ) = italic_O ( italic_x start_POSTSUPERSCRIPT 1 / 2 end_POSTSUPERSCRIPT roman_log start_POSTSUPERSCRIPT 2 end_POSTSUPERSCRIPT italic_x )

    Important to note that the residue term around s=1𝑠1s=1italic_s = 1 contributes negligibly as =O(ϵ2x(logx1))absent𝑂superscriptitalic-ϵ2𝑥𝑥1=O(\epsilon^{2}x(\log x-1))= italic_O ( italic_ϵ start_POSTSUPERSCRIPT 2 end_POSTSUPERSCRIPT italic_x ( roman_log italic_x - 1 ) ), and the error terms contribute upto O(x1/2log2x)=O(x1/3(logx1))𝑂superscript𝑥12superscript2𝑥𝑂superscript𝑥13𝑥1O(x^{1/2}\log^{2}x)=O(x^{1/3}(\log x-1))italic_O ( italic_x start_POSTSUPERSCRIPT 1 / 2 end_POSTSUPERSCRIPT roman_log start_POSTSUPERSCRIPT 2 end_POSTSUPERSCRIPT italic_x ) = italic_O ( italic_x start_POSTSUPERSCRIPT 1 / 3 end_POSTSUPERSCRIPT ( roman_log italic_x - 1 ) ). (This can be achieved by considering ϵitalic-ϵ\epsilonitalic_ϵ small enough (such as ϵ=x1/6italic-ϵsuperscript𝑥16\epsilon=x^{-1/6}italic_ϵ = italic_x start_POSTSUPERSCRIPT - 1 / 6 end_POSTSUPERSCRIPT). Therefore, we conclude that,

    dxμ(d)logdd=O(1)subscript𝑑𝑥𝜇𝑑𝑑𝑑𝑂1\displaystyle\sum_{d\leq x}\frac{\mu(d)\log d}{d}=O(1)∑ start_POSTSUBSCRIPT italic_d ≤ italic_x end_POSTSUBSCRIPT divide start_ARG italic_μ ( italic_d ) roman_log italic_d end_ARG start_ARG italic_d end_ARG = italic_O ( 1 )
  3. (iii)𝑖𝑖𝑖(iii)( italic_i italic_i italic_i )

    A priori from the definition of M(x)𝑀𝑥M(x)italic_M ( italic_x ) and applying Perron’s Formula for a(n)=μ(n)𝑎𝑛𝜇𝑛a(n)=\mu(n)italic_a ( italic_n ) = italic_μ ( italic_n ) yields,

    M(x)=12πiciTc+iT1ζ(s)xss𝑑s+O(n=11ncmin(1,xT|s|))𝑀𝑥12𝜋𝑖superscriptsubscript𝑐𝑖𝑇𝑐𝑖𝑇1𝜁𝑠superscript𝑥𝑠𝑠differential-d𝑠𝑂superscriptsubscript𝑛11superscript𝑛𝑐1𝑥𝑇𝑠\displaystyle M(x)=\frac{1}{2\pi i}\int\limits_{c-iT}^{c+iT}\frac{1}{\zeta(s)}% \frac{x^{s}}{s}\,ds+O\left(\sum_{n=1}^{\infty}\frac{1}{n^{c}}\min\left(1,\frac% {x}{T|s|}\right)\right)italic_M ( italic_x ) = divide start_ARG 1 end_ARG start_ARG 2 italic_π italic_i end_ARG ∫ start_POSTSUBSCRIPT italic_c - italic_i italic_T end_POSTSUBSCRIPT start_POSTSUPERSCRIPT italic_c + italic_i italic_T end_POSTSUPERSCRIPT divide start_ARG 1 end_ARG start_ARG italic_ζ ( italic_s ) end_ARG divide start_ARG italic_x start_POSTSUPERSCRIPT italic_s end_POSTSUPERSCRIPT end_ARG start_ARG italic_s end_ARG italic_d italic_s + italic_O ( ∑ start_POSTSUBSCRIPT italic_n = 1 end_POSTSUBSCRIPT start_POSTSUPERSCRIPT ∞ end_POSTSUPERSCRIPT divide start_ARG 1 end_ARG start_ARG italic_n start_POSTSUPERSCRIPT italic_c end_POSTSUPERSCRIPT end_ARG roman_min ( 1 , divide start_ARG italic_x end_ARG start_ARG italic_T | italic_s | end_ARG ) ) (2.14)

    As for computing the integral in (2.14) over the vertical line L𝐿Litalic_L: s=c+it𝑠𝑐𝑖𝑡s=c+ititalic_s = italic_c + italic_i italic_t, TtT𝑇𝑡𝑇-T\leq t\leq T- italic_T ≤ italic_t ≤ italic_T, where c>1𝑐1c>1italic_c > 1 , our aim is to try shifting the contour of integration to a vertical line closer to the critical strip. For our convenience, we choose c=1+ϵ𝑐1italic-ϵc=1+\epsilonitalic_c = 1 + italic_ϵ where, ϵ>0italic-ϵ0\epsilon>0italic_ϵ > 0. Using the fact that ζ(s)𝜁𝑠\zeta(s)italic_ζ ( italic_s ) has no zeros for (s)>1𝑠1\Re(s)>1roman_ℜ ( italic_s ) > 1, we intend on obtaining a suitable bound for 1ζ(s)1𝜁𝑠\frac{1}{\zeta(s)}divide start_ARG 1 end_ARG start_ARG italic_ζ ( italic_s ) end_ARG in this region.

    Observe that for (s)=1+ϵ𝑠1italic-ϵ\Re(s)=1+\epsilonroman_ℜ ( italic_s ) = 1 + italic_ϵ, ζ(s)𝜁𝑠\zeta(s)italic_ζ ( italic_s ) is bounded away from zero, implying 1ζ(s)1𝜁𝑠\frac{1}{\zeta(s)}divide start_ARG 1 end_ARG start_ARG italic_ζ ( italic_s ) end_ARG is also bounded. Specifically, for s=1+ϵ+it𝑠1italic-ϵ𝑖𝑡s=1+\epsilon+ititalic_s = 1 + italic_ϵ + italic_i italic_t,

    |1ζ(s)|A1𝜁𝑠𝐴\displaystyle\left|\frac{1}{\zeta(s)}\right|\leq A| divide start_ARG 1 end_ARG start_ARG italic_ζ ( italic_s ) end_ARG | ≤ italic_A

    for some constant A𝐴Aitalic_A. On the other hand,

    | 1+ϵiT1+ϵ+iT1ζ(s)xss𝑑s|=| TTx1+ϵ+it1+ϵ+it1ζ(1+ϵ+it)𝑑t|ATTx1+ϵ(1+ϵ)2+t2𝑑t superscriptsubscript1italic-ϵ𝑖𝑇1italic-ϵ𝑖𝑇1𝜁𝑠superscript𝑥𝑠𝑠differential-d𝑠 superscriptsubscript𝑇𝑇superscript𝑥1italic-ϵ𝑖𝑡1italic-ϵ𝑖𝑡1𝜁1italic-ϵ𝑖𝑡differential-d𝑡𝐴superscriptsubscript𝑇𝑇superscript𝑥1italic-ϵsuperscript1italic-ϵ2superscript𝑡2differential-d𝑡\displaystyle\left|\mbox{ }\int\limits_{1+\epsilon-iT}^{1+\epsilon+iT}\frac{1}% {\zeta(s)}\frac{x^{s}}{s}\,ds\right|=\left|\mbox{ }\int\limits_{-T}^{T}\frac{x% ^{1+\epsilon+it}}{1+\epsilon+it}\cdot\frac{1}{\zeta(1+\epsilon+it)}\,dt\right|% \leq A\cdot\int\limits_{-T}^{T}\frac{x^{1+\epsilon}}{\sqrt{(1+\epsilon)^{2}+t^% {2}}}\,dt| ∫ start_POSTSUBSCRIPT 1 + italic_ϵ - italic_i italic_T end_POSTSUBSCRIPT start_POSTSUPERSCRIPT 1 + italic_ϵ + italic_i italic_T end_POSTSUPERSCRIPT divide start_ARG 1 end_ARG start_ARG italic_ζ ( italic_s ) end_ARG divide start_ARG italic_x start_POSTSUPERSCRIPT italic_s end_POSTSUPERSCRIPT end_ARG start_ARG italic_s end_ARG italic_d italic_s | = | ∫ start_POSTSUBSCRIPT - italic_T end_POSTSUBSCRIPT start_POSTSUPERSCRIPT italic_T end_POSTSUPERSCRIPT divide start_ARG italic_x start_POSTSUPERSCRIPT 1 + italic_ϵ + italic_i italic_t end_POSTSUPERSCRIPT end_ARG start_ARG 1 + italic_ϵ + italic_i italic_t end_ARG ⋅ divide start_ARG 1 end_ARG start_ARG italic_ζ ( 1 + italic_ϵ + italic_i italic_t ) end_ARG italic_d italic_t | ≤ italic_A ⋅ ∫ start_POSTSUBSCRIPT - italic_T end_POSTSUBSCRIPT start_POSTSUPERSCRIPT italic_T end_POSTSUPERSCRIPT divide start_ARG italic_x start_POSTSUPERSCRIPT 1 + italic_ϵ end_POSTSUPERSCRIPT end_ARG start_ARG square-root start_ARG ( 1 + italic_ϵ ) start_POSTSUPERSCRIPT 2 end_POSTSUPERSCRIPT + italic_t start_POSTSUPERSCRIPT 2 end_POSTSUPERSCRIPT end_ARG end_ARG italic_d italic_t
    Ax1+ϵ1(1+ϵ)2+t2𝑑t=Ax1+ϵπ1+ϵ=O(x1+ϵ)absent𝐴superscript𝑥1italic-ϵsuperscriptsubscript1superscript1italic-ϵ2superscript𝑡2differential-d𝑡𝐴superscript𝑥1italic-ϵ𝜋1italic-ϵ𝑂superscript𝑥1italic-ϵ\displaystyle\leq A\cdot x^{1+\epsilon}\int\limits_{-\infty}^{\infty}\frac{1}{% \sqrt{(1+\epsilon)^{2}+t^{2}}}\,dt=A\cdot x^{1+\epsilon}\frac{\pi}{1+\epsilon}% =O(x^{1+\epsilon})≤ italic_A ⋅ italic_x start_POSTSUPERSCRIPT 1 + italic_ϵ end_POSTSUPERSCRIPT ∫ start_POSTSUBSCRIPT - ∞ end_POSTSUBSCRIPT start_POSTSUPERSCRIPT ∞ end_POSTSUPERSCRIPT divide start_ARG 1 end_ARG start_ARG square-root start_ARG ( 1 + italic_ϵ ) start_POSTSUPERSCRIPT 2 end_POSTSUPERSCRIPT + italic_t start_POSTSUPERSCRIPT 2 end_POSTSUPERSCRIPT end_ARG end_ARG italic_d italic_t = italic_A ⋅ italic_x start_POSTSUPERSCRIPT 1 + italic_ϵ end_POSTSUPERSCRIPT divide start_ARG italic_π end_ARG start_ARG 1 + italic_ϵ end_ARG = italic_O ( italic_x start_POSTSUPERSCRIPT 1 + italic_ϵ end_POSTSUPERSCRIPT )

    Subsequently, the error term from the vertical line integral can be estimated as, O(x1+ϵ)𝑂superscript𝑥1italic-ϵO(x^{1+\epsilon})italic_O ( italic_x start_POSTSUPERSCRIPT 1 + italic_ϵ end_POSTSUPERSCRIPT ), for any small ϵ>0italic-ϵ0\epsilon>0italic_ϵ > 0.

    Hence, we need to choose T𝑇Titalic_T appropriately to control the error term in Perron’s formula. Using the Cauchy Residue Theorem [21, Chapt. 5.1 , pp. 120] and estimating the integral, we set T=x𝑇𝑥T=\sqrt{x}italic_T = square-root start_ARG italic_x end_ARG and consider the main term and error terms:

    1+ϵix1+ϵ+ix1ζ(s)xss𝑑s=O(x1/2+ϵ)superscriptsubscript1italic-ϵ𝑖𝑥1italic-ϵ𝑖𝑥1𝜁𝑠superscript𝑥𝑠𝑠differential-d𝑠𝑂superscript𝑥12italic-ϵ\displaystyle\int\limits_{1+\epsilon-i\sqrt{x}}^{1+\epsilon+i\sqrt{x}}\frac{1}% {\zeta(s)}\frac{x^{s}}{s}\,ds=O(x^{1/2+\epsilon})∫ start_POSTSUBSCRIPT 1 + italic_ϵ - italic_i square-root start_ARG italic_x end_ARG end_POSTSUBSCRIPT start_POSTSUPERSCRIPT 1 + italic_ϵ + italic_i square-root start_ARG italic_x end_ARG end_POSTSUPERSCRIPT divide start_ARG 1 end_ARG start_ARG italic_ζ ( italic_s ) end_ARG divide start_ARG italic_x start_POSTSUPERSCRIPT italic_s end_POSTSUPERSCRIPT end_ARG start_ARG italic_s end_ARG italic_d italic_s = italic_O ( italic_x start_POSTSUPERSCRIPT 1 / 2 + italic_ϵ end_POSTSUPERSCRIPT )

    This ensures that the main contribution comes from the vertical integral and the error terms are bounded appropriately. Accordingly,

    M(x)=O(x1/2+ϵ)𝑀𝑥𝑂superscript𝑥12italic-ϵ\displaystyle M(x)=O(x^{1/2+\epsilon})italic_M ( italic_x ) = italic_O ( italic_x start_POSTSUPERSCRIPT 1 / 2 + italic_ϵ end_POSTSUPERSCRIPT )

    By choosing ϵ>0italic-ϵ0\epsilon>0italic_ϵ > 0 sufficiently small, we can make the bound as close to O(x)𝑂𝑥O(\sqrt{x})italic_O ( square-root start_ARG italic_x end_ARG ) as desired.

3 Order Estimates involving M(x)𝑀𝑥M(x)italic_M ( italic_x )

In this section, we shall apply the definitions of Chebyshev ψ𝜓\psiitalic_ψ Function, ψ(x)𝜓𝑥\psi(x)italic_ψ ( italic_x ) in order to come up with a suitable estimate for π(x)𝜋𝑥\pi(x)italic_π ( italic_x ) in terms of the Mertens Function, M(x)𝑀𝑥M(x)italic_M ( italic_x ).

3.1 Relation between ψ(x)𝜓𝑥\psi(x)italic_ψ ( italic_x ) and M(x)𝑀𝑥M(x)italic_M ( italic_x )

Theorem 3.1.1.

The following holds true for the Chebyshev ψ𝜓\psiitalic_ψ function, ψ(x)𝜓𝑥\psi(x)italic_ψ ( italic_x ):

ψ(x)=xnxM(xn)+O(x)𝜓𝑥𝑥subscript𝑛𝑥𝑀𝑥𝑛𝑂𝑥\displaystyle\psi(x)=x-\sum_{n\leq x}M\left(\frac{x}{n}\right)+O(\sqrt{x})italic_ψ ( italic_x ) = italic_x - ∑ start_POSTSUBSCRIPT italic_n ≤ italic_x end_POSTSUBSCRIPT italic_M ( divide start_ARG italic_x end_ARG start_ARG italic_n end_ARG ) + italic_O ( square-root start_ARG italic_x end_ARG ) (3.1)
Proof.

A proiori from the definition of ψ(x)𝜓𝑥\psi(x)italic_ψ ( italic_x ) involving the Von Mangoldt Function, Λ(n)Λ𝑛\Lambda(n)roman_Λ ( italic_n ), we apply the Mo¨¨𝑜\ddot{o}over¨ start_ARG italic_o end_ARGbius Inversion Formula [17, Section 14.1 , pp. 30] on Λ(n)Λ𝑛\Lambda(n)roman_Λ ( italic_n ) to obtain,

Λ(n)=dnμ(d)log(nd)=nxdnμ(d)log(nd)Λ𝑛subscriptconditional𝑑𝑛𝜇𝑑𝑛𝑑subscript𝑛𝑥subscriptconditional𝑑𝑛𝜇𝑑𝑛𝑑\displaystyle\Lambda(n)=\sum_{d\mid n}\mu(d)\log\left(\frac{n}{d}\right)=\sum_% {n\leq x}\sum_{d\mid n}\mu(d)\log\left(\frac{n}{d}\right)roman_Λ ( italic_n ) = ∑ start_POSTSUBSCRIPT italic_d ∣ italic_n end_POSTSUBSCRIPT italic_μ ( italic_d ) roman_log ( divide start_ARG italic_n end_ARG start_ARG italic_d end_ARG ) = ∑ start_POSTSUBSCRIPT italic_n ≤ italic_x end_POSTSUBSCRIPT ∑ start_POSTSUBSCRIPT italic_d ∣ italic_n end_POSTSUBSCRIPT italic_μ ( italic_d ) roman_log ( divide start_ARG italic_n end_ARG start_ARG italic_d end_ARG ) (3.2)
=dxμ(d)kxdlog(k)[ Interchanging order of summation ]absentsubscript𝑑𝑥𝜇𝑑subscript𝑘𝑥𝑑𝑘delimited-[] Interchanging order of summation \displaystyle=\sum_{d\leq x}\mu(d)\sum_{k\leq\frac{x}{d}}\log(k)\hskip 20.0pt[% \mbox{ Interchanging order of summation }]= ∑ start_POSTSUBSCRIPT italic_d ≤ italic_x end_POSTSUBSCRIPT italic_μ ( italic_d ) ∑ start_POSTSUBSCRIPT italic_k ≤ divide start_ARG italic_x end_ARG start_ARG italic_d end_ARG end_POSTSUBSCRIPT roman_log ( italic_k ) [ Interchanging order of summation ] (3.3)
Remark 3.1.2.

Here, we set n=dk𝑛𝑑𝑘n=dkitalic_n = italic_d italic_k, so that the inner sum is over k𝑘kitalic_k with kxd𝑘𝑥𝑑k\leq\frac{x}{d}italic_k ≤ divide start_ARG italic_x end_ARG start_ARG italic_d end_ARG.

We approximate the sum of log(k)𝑘\log(k)roman_log ( italic_k ) by integrating the logarithm function from 1 to xd𝑥𝑑\frac{x}{d}divide start_ARG italic_x end_ARG start_ARG italic_d end_ARG, and then applying Integration by Parts.

kxdlog(k)=1xdlog(t)𝑑t+O(log(xd))=[tlog(t)t]1xd+O(log(xd))subscript𝑘𝑥𝑑𝑘superscriptsubscript1𝑥𝑑𝑡differential-d𝑡𝑂𝑥𝑑superscriptsubscriptdelimited-[]𝑡𝑡𝑡1𝑥𝑑𝑂𝑥𝑑\displaystyle\sum_{k\leq\frac{x}{d}}\log(k)=\int_{1}^{\frac{x}{d}}\log(t)\,dt+% O(\log(\frac{x}{d}))=\left[t\log(t)-t\right]_{1}^{\frac{x}{d}}+O(\log(\frac{x}% {d}))∑ start_POSTSUBSCRIPT italic_k ≤ divide start_ARG italic_x end_ARG start_ARG italic_d end_ARG end_POSTSUBSCRIPT roman_log ( italic_k ) = ∫ start_POSTSUBSCRIPT 1 end_POSTSUBSCRIPT start_POSTSUPERSCRIPT divide start_ARG italic_x end_ARG start_ARG italic_d end_ARG end_POSTSUPERSCRIPT roman_log ( italic_t ) italic_d italic_t + italic_O ( roman_log ( divide start_ARG italic_x end_ARG start_ARG italic_d end_ARG ) ) = [ italic_t roman_log ( italic_t ) - italic_t ] start_POSTSUBSCRIPT 1 end_POSTSUBSCRIPT start_POSTSUPERSCRIPT divide start_ARG italic_x end_ARG start_ARG italic_d end_ARG end_POSTSUPERSCRIPT + italic_O ( roman_log ( divide start_ARG italic_x end_ARG start_ARG italic_d end_ARG ) )
=xdlog(xd)xd+1+O(log(xd))=xdlog(xd)xd+1+O(log(x))absent𝑥𝑑𝑥𝑑𝑥𝑑1𝑂𝑥𝑑𝑥𝑑𝑥𝑑𝑥𝑑1𝑂𝑥\displaystyle=\frac{x}{d}\log\left(\frac{x}{d}\right)-\frac{x}{d}+1+O(\log(% \frac{x}{d}))=\frac{x}{d}\log\left(\frac{x}{d}\right)-\frac{x}{d}+1+O(\log(x))= divide start_ARG italic_x end_ARG start_ARG italic_d end_ARG roman_log ( divide start_ARG italic_x end_ARG start_ARG italic_d end_ARG ) - divide start_ARG italic_x end_ARG start_ARG italic_d end_ARG + 1 + italic_O ( roman_log ( divide start_ARG italic_x end_ARG start_ARG italic_d end_ARG ) ) = divide start_ARG italic_x end_ARG start_ARG italic_d end_ARG roman_log ( divide start_ARG italic_x end_ARG start_ARG italic_d end_ARG ) - divide start_ARG italic_x end_ARG start_ARG italic_d end_ARG + 1 + italic_O ( roman_log ( italic_x ) )
[ Since log(xd) is of the same order as log(x) for large x ]delimited-[] Since log(xd) is of the same order as log(x) for large x \displaystyle[\mbox{ Since $\log(\frac{x}{d})$ is of the same order as $\log(x% )$ for large $x$ }][ Since roman_log ( divide start_ARG italic_x end_ARG start_ARG italic_d end_ARG ) is of the same order as roman_log ( italic_x ) for large italic_x ]

Subsequently, from (3.3) we get,

ψ(x)=dxμ(d)(xdlog(xd)xd+1+O(log(x)))𝜓𝑥subscript𝑑𝑥𝜇𝑑𝑥𝑑𝑥𝑑𝑥𝑑1𝑂𝑥\displaystyle\psi(x)=\sum_{d\leq x}\mu(d)\left(\frac{x}{d}\log\left(\frac{x}{d% }\right)-\frac{x}{d}+1+O(\log(x))\right)italic_ψ ( italic_x ) = ∑ start_POSTSUBSCRIPT italic_d ≤ italic_x end_POSTSUBSCRIPT italic_μ ( italic_d ) ( divide start_ARG italic_x end_ARG start_ARG italic_d end_ARG roman_log ( divide start_ARG italic_x end_ARG start_ARG italic_d end_ARG ) - divide start_ARG italic_x end_ARG start_ARG italic_d end_ARG + 1 + italic_O ( roman_log ( italic_x ) ) )

Distributing μ(d)𝜇𝑑\mu(d)italic_μ ( italic_d ) and separating the main terms from the error term,

ψ(x)=dxμ(d)(xdlog(xd))dxμ(d)(xd)+dxμ(d)1+dxμ(d)O(log(x))𝜓𝑥subscript𝑑𝑥𝜇𝑑𝑥𝑑𝑥𝑑subscript𝑑𝑥𝜇𝑑𝑥𝑑subscript𝑑𝑥𝜇𝑑1subscript𝑑𝑥𝜇𝑑𝑂𝑥\displaystyle\psi(x)=\sum_{d\leq x}\mu(d)\left(\frac{x}{d}\log\left(\frac{x}{d% }\right)\right)-\sum_{d\leq x}\mu(d)\left(\frac{x}{d}\right)+\sum_{d\leq x}\mu% (d)\cdot 1+\sum_{d\leq x}\mu(d)O(\log(x))italic_ψ ( italic_x ) = ∑ start_POSTSUBSCRIPT italic_d ≤ italic_x end_POSTSUBSCRIPT italic_μ ( italic_d ) ( divide start_ARG italic_x end_ARG start_ARG italic_d end_ARG roman_log ( divide start_ARG italic_x end_ARG start_ARG italic_d end_ARG ) ) - ∑ start_POSTSUBSCRIPT italic_d ≤ italic_x end_POSTSUBSCRIPT italic_μ ( italic_d ) ( divide start_ARG italic_x end_ARG start_ARG italic_d end_ARG ) + ∑ start_POSTSUBSCRIPT italic_d ≤ italic_x end_POSTSUBSCRIPT italic_μ ( italic_d ) ⋅ 1 + ∑ start_POSTSUBSCRIPT italic_d ≤ italic_x end_POSTSUBSCRIPT italic_μ ( italic_d ) italic_O ( roman_log ( italic_x ) )
=xdxμ(d)dlog(xd)xdxμ(d)d+dxμ(d)+O(xlog(x))dxμ(d)absent𝑥subscript𝑑𝑥𝜇𝑑𝑑𝑥𝑑𝑥subscript𝑑𝑥𝜇𝑑𝑑subscript𝑑𝑥𝜇𝑑𝑂𝑥𝑥subscript𝑑𝑥𝜇𝑑\displaystyle=x\sum_{d\leq x}\frac{\mu(d)}{d}\log\left(\frac{x}{d}\right)-x% \sum_{d\leq x}\frac{\mu(d)}{d}+\sum_{d\leq x}\mu(d)+O(x\log(x))\sum_{d\leq x}% \mu(d)= italic_x ∑ start_POSTSUBSCRIPT italic_d ≤ italic_x end_POSTSUBSCRIPT divide start_ARG italic_μ ( italic_d ) end_ARG start_ARG italic_d end_ARG roman_log ( divide start_ARG italic_x end_ARG start_ARG italic_d end_ARG ) - italic_x ∑ start_POSTSUBSCRIPT italic_d ≤ italic_x end_POSTSUBSCRIPT divide start_ARG italic_μ ( italic_d ) end_ARG start_ARG italic_d end_ARG + ∑ start_POSTSUBSCRIPT italic_d ≤ italic_x end_POSTSUBSCRIPT italic_μ ( italic_d ) + italic_O ( italic_x roman_log ( italic_x ) ) ∑ start_POSTSUBSCRIPT italic_d ≤ italic_x end_POSTSUBSCRIPT italic_μ ( italic_d )

A priori using the results obtained in proposition (2.5.2),

xdxμ(d)dlog(xd)=xlog(x)dxμ(d)dxdxμ(d)log(d)d𝑥subscript𝑑𝑥𝜇𝑑𝑑𝑥𝑑𝑥𝑥subscript𝑑𝑥𝜇𝑑𝑑𝑥subscript𝑑𝑥𝜇𝑑𝑑𝑑\displaystyle x\sum_{d\leq x}\frac{\mu(d)}{d}\log\left(\frac{x}{d}\right)=x% \log(x)\sum_{d\leq x}\frac{\mu(d)}{d}-x\sum_{d\leq x}\frac{\mu(d)\log(d)}{d}italic_x ∑ start_POSTSUBSCRIPT italic_d ≤ italic_x end_POSTSUBSCRIPT divide start_ARG italic_μ ( italic_d ) end_ARG start_ARG italic_d end_ARG roman_log ( divide start_ARG italic_x end_ARG start_ARG italic_d end_ARG ) = italic_x roman_log ( italic_x ) ∑ start_POSTSUBSCRIPT italic_d ≤ italic_x end_POSTSUBSCRIPT divide start_ARG italic_μ ( italic_d ) end_ARG start_ARG italic_d end_ARG - italic_x ∑ start_POSTSUBSCRIPT italic_d ≤ italic_x end_POSTSUBSCRIPT divide start_ARG italic_μ ( italic_d ) roman_log ( italic_d ) end_ARG start_ARG italic_d end_ARG (3.4)

Furthermore,

xlog(x)O(1logx)xO(1)=O(x)x=O(x)𝑥𝑥𝑂1𝑥𝑥𝑂1𝑂𝑥𝑥𝑂𝑥\displaystyle x\log(x)\cdot O\left(\frac{1}{\log x}\right)-x\cdot O(1)=O(x)-x=% O(x)italic_x roman_log ( italic_x ) ⋅ italic_O ( divide start_ARG 1 end_ARG start_ARG roman_log italic_x end_ARG ) - italic_x ⋅ italic_O ( 1 ) = italic_O ( italic_x ) - italic_x = italic_O ( italic_x )

Hence,

O(xlogx)dxμ(d)=O(xlogx)O(x)=O(x3/2logx)𝑂𝑥𝑥subscript𝑑𝑥𝜇𝑑𝑂𝑥𝑥𝑂𝑥𝑂superscript𝑥32𝑥\displaystyle O(x\log x)\sum_{d\leq x}\mu(d)=O(x\log x)\cdot O(\sqrt{x})=O(x^{% 3/2}\log x)italic_O ( italic_x roman_log italic_x ) ∑ start_POSTSUBSCRIPT italic_d ≤ italic_x end_POSTSUBSCRIPT italic_μ ( italic_d ) = italic_O ( italic_x roman_log italic_x ) ⋅ italic_O ( square-root start_ARG italic_x end_ARG ) = italic_O ( italic_x start_POSTSUPERSCRIPT 3 / 2 end_POSTSUPERSCRIPT roman_log italic_x ) (3.5)

Combining (3.4) and (3.5) yields,

ψ(x)=xnxM(xn)+O(x)+O(x3/2logx)𝜓𝑥𝑥subscript𝑛𝑥𝑀𝑥𝑛𝑂𝑥𝑂superscript𝑥32𝑥\displaystyle\psi(x)=x-\sum_{n\leq x}M\left(\frac{x}{n}\right)+O(x)+O(x^{3/2}% \log x)italic_ψ ( italic_x ) = italic_x - ∑ start_POSTSUBSCRIPT italic_n ≤ italic_x end_POSTSUBSCRIPT italic_M ( divide start_ARG italic_x end_ARG start_ARG italic_n end_ARG ) + italic_O ( italic_x ) + italic_O ( italic_x start_POSTSUPERSCRIPT 3 / 2 end_POSTSUPERSCRIPT roman_log italic_x )

Since O(x3/2logx)𝑂superscript𝑥32𝑥O(x^{3/2}\log x)italic_O ( italic_x start_POSTSUPERSCRIPT 3 / 2 end_POSTSUPERSCRIPT roman_log italic_x ) dominates O(x)𝑂𝑥O(x)italic_O ( italic_x ), we conclude that,

ψ(x)=xnxM(xn)+O(x)𝜓𝑥𝑥subscript𝑛𝑥𝑀𝑥𝑛𝑂𝑥\displaystyle\psi(x)=x-\sum_{n\leq x}M\left(\frac{x}{n}\right)+O(\sqrt{x})italic_ψ ( italic_x ) = italic_x - ∑ start_POSTSUBSCRIPT italic_n ≤ italic_x end_POSTSUBSCRIPT italic_M ( divide start_ARG italic_x end_ARG start_ARG italic_n end_ARG ) + italic_O ( square-root start_ARG italic_x end_ARG )

And the proof is thus complete. ∎

3.2 An important approximation for ψ(x)𝜓𝑥\psi(x)italic_ψ ( italic_x )

A tricky application of the famous Prime Number Theorem (2.1.1) yields the following estimate,

ψ(x)=x+O(xeclogx)𝜓𝑥𝑥𝑂𝑥superscript𝑒𝑐𝑥\displaystyle\psi(x)=x+O\left(xe^{-c\sqrt{\log x}}\right)italic_ψ ( italic_x ) = italic_x + italic_O ( italic_x italic_e start_POSTSUPERSCRIPT - italic_c square-root start_ARG roman_log italic_x end_ARG end_POSTSUPERSCRIPT ) (3.6)

or some constant c>0𝑐0c>0italic_c > 0. However, it is indeed possible to obtain a simpler, and more effective bound for the Chebyshev ψ𝜓\psiitalic_ψ Function.

Lemma 3.2.1.

We have,

ψ(x)=x+O(x1/2log2x)𝜓𝑥𝑥𝑂superscript𝑥12superscript2𝑥\displaystyle\psi(x)=x+O\left(x^{1/2}\log^{2}x\right)italic_ψ ( italic_x ) = italic_x + italic_O ( italic_x start_POSTSUPERSCRIPT 1 / 2 end_POSTSUPERSCRIPT roman_log start_POSTSUPERSCRIPT 2 end_POSTSUPERSCRIPT italic_x ) (3.7)
Proof.

This proof thouroughly utilizes results from Analytic Number Theory, specifically the properties of the Chebyshev function ψ(x)𝜓𝑥\psi(x)italic_ψ ( italic_x ) and the distribution of primes. We shall also leverage results from the analytic properties of the Riemann zeta function ζ(s)𝜁𝑠\zeta(s)italic_ζ ( italic_s ).

Important to note that, the proof relies on properties of the Riemann zeta function ζ(s)𝜁𝑠\zeta(s)italic_ζ ( italic_s ) and its non-trivial zeros. However, we do not assume the Riemann Hypothesis (RH) here explicitly.

The explicit formula for ψ(x)𝜓𝑥\psi(x)italic_ψ ( italic_x ) involves the zeros of ζ(s)𝜁𝑠\zeta(s)italic_ζ ( italic_s ):

ψ(x)=xρxρρlog(2π),𝜓𝑥𝑥subscript𝜌superscript𝑥𝜌𝜌2𝜋\psi(x)=x-\sum_{\rho}\frac{x^{\rho}}{\rho}-\log(2\pi),italic_ψ ( italic_x ) = italic_x - ∑ start_POSTSUBSCRIPT italic_ρ end_POSTSUBSCRIPT divide start_ARG italic_x start_POSTSUPERSCRIPT italic_ρ end_POSTSUPERSCRIPT end_ARG start_ARG italic_ρ end_ARG - roman_log ( 2 italic_π ) ,

where the sum is over the non-trivial zeros ρ𝜌\rhoitalic_ρ of ζ(s)𝜁𝑠\zeta(s)italic_ζ ( italic_s ), and the term log(2π)2𝜋\log(2\pi)roman_log ( 2 italic_π ) arises due to the existence of pole at s=1𝑠1s=1italic_s = 1.

The critical line (s)=12𝑠12\Re(s)=\frac{1}{2}roman_ℜ ( italic_s ) = divide start_ARG 1 end_ARG start_ARG 2 end_ARG plays a key role. Let ρ=β+iγ𝜌𝛽𝑖𝛾\rho=\beta+i\gammaitalic_ρ = italic_β + italic_i italic_γ be a non-trivial zero of ζ(s)𝜁𝑠\zeta(s)italic_ζ ( italic_s ) with β=(ρ)𝛽𝜌\beta=\Re(\rho)italic_β = roman_ℜ ( italic_ρ ) and γ=(ρ)𝛾𝜌\gamma=\Im(\rho)italic_γ = roman_ℑ ( italic_ρ ).

The zeros are symmetric about the real axis, so we consider only the upper half-plane. For each zero ρ𝜌\rhoitalic_ρ of ζ(s)𝜁𝑠\zeta(s)italic_ζ ( italic_s ), the term xρρsuperscript𝑥𝜌𝜌\frac{x^{\rho}}{\rho}divide start_ARG italic_x start_POSTSUPERSCRIPT italic_ρ end_POSTSUPERSCRIPT end_ARG start_ARG italic_ρ end_ARG contributes to ψ(x)𝜓𝑥\psi(x)italic_ψ ( italic_x ). To estimate the error term, consider the sum over the non-trivial zeros ρ𝜌\rhoitalic_ρ:

ρxρρsubscript𝜌superscript𝑥𝜌𝜌\displaystyle\sum_{\rho}\frac{x^{\rho}}{\rho}∑ start_POSTSUBSCRIPT italic_ρ end_POSTSUBSCRIPT divide start_ARG italic_x start_POSTSUPERSCRIPT italic_ρ end_POSTSUPERSCRIPT end_ARG start_ARG italic_ρ end_ARG

In addition to above, we use the fact that the zeros ρ𝜌\rhoitalic_ρ have β1𝛽1\beta\leq 1italic_β ≤ 1. The contribution of each such ρ𝜌\rhoitalic_ρ can be bounded by,

|xρρ|xβ|ρ|1superscript𝑥𝜌𝜌superscript𝑥𝛽superscript𝜌1\displaystyle\left|\frac{x^{\rho}}{\rho}\right|\leq x^{\beta}|\rho|^{-1}| divide start_ARG italic_x start_POSTSUPERSCRIPT italic_ρ end_POSTSUPERSCRIPT end_ARG start_ARG italic_ρ end_ARG | ≤ italic_x start_POSTSUPERSCRIPT italic_β end_POSTSUPERSCRIPT | italic_ρ | start_POSTSUPERSCRIPT - 1 end_POSTSUPERSCRIPT

For zeros with β=12𝛽12\beta=\frac{1}{2}italic_β = divide start_ARG 1 end_ARG start_ARG 2 end_ARG, xβ=x1/2.superscript𝑥𝛽superscript𝑥12x^{\beta}=x^{1/2}.italic_x start_POSTSUPERSCRIPT italic_β end_POSTSUPERSCRIPT = italic_x start_POSTSUPERSCRIPT 1 / 2 end_POSTSUPERSCRIPT .. Hence, the number of zeros with |γ|T𝛾𝑇|\gamma|\leq T| italic_γ | ≤ italic_T is O(TlogT)𝑂𝑇𝑇O(T\log T)italic_O ( italic_T roman_log italic_T ). We choose T=x𝑇𝑥T=xitalic_T = italic_x to cover the relevant range of zeros.

|γ|x|ρ|1=|γ|x((12)2+γ2)1/2|γ|x1|γ|=O(logx)subscript𝛾𝑥superscript𝜌1subscript𝛾𝑥superscriptsuperscript122superscript𝛾212subscript𝛾𝑥1𝛾𝑂𝑥\displaystyle\sum_{|\gamma|\leq x}|\rho|^{-1}=\sum_{|\gamma|\leq x}\left(\left% (\frac{1}{2}\right)^{2}+\gamma^{2}\right)^{-1/2}\leq\sum_{|\gamma|\leq x}\frac% {1}{|\gamma|}=O(\log x)∑ start_POSTSUBSCRIPT | italic_γ | ≤ italic_x end_POSTSUBSCRIPT | italic_ρ | start_POSTSUPERSCRIPT - 1 end_POSTSUPERSCRIPT = ∑ start_POSTSUBSCRIPT | italic_γ | ≤ italic_x end_POSTSUBSCRIPT ( ( divide start_ARG 1 end_ARG start_ARG 2 end_ARG ) start_POSTSUPERSCRIPT 2 end_POSTSUPERSCRIPT + italic_γ start_POSTSUPERSCRIPT 2 end_POSTSUPERSCRIPT ) start_POSTSUPERSCRIPT - 1 / 2 end_POSTSUPERSCRIPT ≤ ∑ start_POSTSUBSCRIPT | italic_γ | ≤ italic_x end_POSTSUBSCRIPT divide start_ARG 1 end_ARG start_ARG | italic_γ | end_ARG = italic_O ( roman_log italic_x )

Combining these estimates, we obtain,

ρxρρ=O(x1/2logx)subscript𝜌superscript𝑥𝜌𝜌𝑂superscript𝑥12𝑥\displaystyle\sum_{\rho}\frac{x^{\rho}}{\rho}=O(x^{1/2}\log x)∑ start_POSTSUBSCRIPT italic_ρ end_POSTSUBSCRIPT divide start_ARG italic_x start_POSTSUPERSCRIPT italic_ρ end_POSTSUPERSCRIPT end_ARG start_ARG italic_ρ end_ARG = italic_O ( italic_x start_POSTSUPERSCRIPT 1 / 2 end_POSTSUPERSCRIPT roman_log italic_x ) (3.8)

Including the logarithmic term from the pole at s=1𝑠1s=1italic_s = 1, the error term in ψ(x)𝜓𝑥\psi(x)italic_ψ ( italic_x ) becomes,

ψ(x)=x+O(x1/2logx)𝜓𝑥𝑥𝑂superscript𝑥12𝑥\displaystyle\psi(x)=x+O(x^{1/2}\log x)italic_ψ ( italic_x ) = italic_x + italic_O ( italic_x start_POSTSUPERSCRIPT 1 / 2 end_POSTSUPERSCRIPT roman_log italic_x )

Since we know that the error term actually involves log2xsuperscript2𝑥\log^{2}xroman_log start_POSTSUPERSCRIPT 2 end_POSTSUPERSCRIPT italic_x due to the density of the zeros of ζ(s)𝜁𝑠\zeta(s)italic_ζ ( italic_s ) and more refined estimates, thus, we can further improve our estimate to,

ψ(x)=x+O(x1/2log2x)𝜓𝑥𝑥𝑂superscript𝑥12superscript2𝑥\displaystyle\psi(x)=x+O(x^{1/2}\log^{2}x)italic_ψ ( italic_x ) = italic_x + italic_O ( italic_x start_POSTSUPERSCRIPT 1 / 2 end_POSTSUPERSCRIPT roman_log start_POSTSUPERSCRIPT 2 end_POSTSUPERSCRIPT italic_x )

As the title of this section suggests, we shall now proceed towards understanding how we can approximate π(x)𝜋𝑥\pi(x)italic_π ( italic_x ) using properties of ψ(x)𝜓𝑥\psi(x)italic_ψ ( italic_x ).

Theorem 3.2.2.

The following holds for the Prime counting function, π(x)𝜋𝑥\pi(x)italic_π ( italic_x ):

π(x)=ψ(x)logx+O(x(logx)2)𝜋𝑥𝜓𝑥𝑥𝑂𝑥superscript𝑥2\displaystyle\pi(x)=\frac{\psi(x)}{\log x}+O\left(\frac{x}{(\log x)^{2}}\right)italic_π ( italic_x ) = divide start_ARG italic_ψ ( italic_x ) end_ARG start_ARG roman_log italic_x end_ARG + italic_O ( divide start_ARG italic_x end_ARG start_ARG ( roman_log italic_x ) start_POSTSUPERSCRIPT 2 end_POSTSUPERSCRIPT end_ARG ) (3.9)
Proof.

Before we delve into the proof, notice that Λ(n)Λ𝑛\Lambda(n)roman_Λ ( italic_n ) is zero except when n𝑛nitalic_n is a power of a prime, specifically n=pk𝑛superscript𝑝𝑘n=p^{k}italic_n = italic_p start_POSTSUPERSCRIPT italic_k end_POSTSUPERSCRIPT. We can hence rewrite ψ(x)𝜓𝑥\psi(x)italic_ψ ( italic_x ) as,

ψ(x)=pxlogp(1+logxlogp)𝜓𝑥subscript𝑝𝑥𝑝1𝑥𝑝\displaystyle\psi(x)=\sum_{p\leq x}\log p\cdot\left(1+\left\lfloor\frac{\log x% }{\log p}\right\rfloor\right)italic_ψ ( italic_x ) = ∑ start_POSTSUBSCRIPT italic_p ≤ italic_x end_POSTSUBSCRIPT roman_log italic_p ⋅ ( 1 + ⌊ divide start_ARG roman_log italic_x end_ARG start_ARG roman_log italic_p end_ARG ⌋ )

Further simplification yields,

ψ(x)=pkxlogp=pxlogp+p2xlogp+p3xlogp+=pxlogp+k=2px1/klogp𝜓𝑥subscriptsuperscript𝑝𝑘𝑥𝑝subscript𝑝𝑥𝑝subscriptsuperscript𝑝2𝑥𝑝subscriptsuperscript𝑝3𝑥𝑝subscript𝑝𝑥𝑝superscriptsubscript𝑘2subscript𝑝superscript𝑥1𝑘𝑝\displaystyle\psi(x)=\sum_{p^{k}\leq x}\log p=\sum_{p\leq x}\log p+\sum_{p^{2}% \leq x}\log p+\sum_{p^{3}\leq x}\log p+\cdots=\sum_{p\leq x}\log p+\sum_{k=2}^% {\infty}\sum_{p\leq x^{1/k}}\log pitalic_ψ ( italic_x ) = ∑ start_POSTSUBSCRIPT italic_p start_POSTSUPERSCRIPT italic_k end_POSTSUPERSCRIPT ≤ italic_x end_POSTSUBSCRIPT roman_log italic_p = ∑ start_POSTSUBSCRIPT italic_p ≤ italic_x end_POSTSUBSCRIPT roman_log italic_p + ∑ start_POSTSUBSCRIPT italic_p start_POSTSUPERSCRIPT 2 end_POSTSUPERSCRIPT ≤ italic_x end_POSTSUBSCRIPT roman_log italic_p + ∑ start_POSTSUBSCRIPT italic_p start_POSTSUPERSCRIPT 3 end_POSTSUPERSCRIPT ≤ italic_x end_POSTSUBSCRIPT roman_log italic_p + ⋯ = ∑ start_POSTSUBSCRIPT italic_p ≤ italic_x end_POSTSUBSCRIPT roman_log italic_p + ∑ start_POSTSUBSCRIPT italic_k = 2 end_POSTSUBSCRIPT start_POSTSUPERSCRIPT ∞ end_POSTSUPERSCRIPT ∑ start_POSTSUBSCRIPT italic_p ≤ italic_x start_POSTSUPERSCRIPT 1 / italic_k end_POSTSUPERSCRIPT end_POSTSUBSCRIPT roman_log italic_p (3.10)

The error term in (3.10) comes from the higher powers of primes,

k=2px1/klogpsuperscriptsubscript𝑘2subscript𝑝superscript𝑥1𝑘𝑝\displaystyle\sum_{k=2}^{\infty}\sum_{p\leq x^{1/k}}\log p∑ start_POSTSUBSCRIPT italic_k = 2 end_POSTSUBSCRIPT start_POSTSUPERSCRIPT ∞ end_POSTSUPERSCRIPT ∑ start_POSTSUBSCRIPT italic_p ≤ italic_x start_POSTSUPERSCRIPT 1 / italic_k end_POSTSUPERSCRIPT end_POSTSUBSCRIPT roman_log italic_p

For k=2𝑘2k=2italic_k = 2, the sum is,

pxlogp12xlogxsubscript𝑝𝑥𝑝12𝑥𝑥\displaystyle\sum_{p\leq\sqrt{x}}\log p\approx\frac{1}{2}\sqrt{x}\log x∑ start_POSTSUBSCRIPT italic_p ≤ square-root start_ARG italic_x end_ARG end_POSTSUBSCRIPT roman_log italic_p ≈ divide start_ARG 1 end_ARG start_ARG 2 end_ARG square-root start_ARG italic_x end_ARG roman_log italic_x

Whereas, for higher values of k𝑘kitalic_k, the sums are even smaller,

k=3px1/klogpk=3x3logx=O(x3logx)superscriptsubscript𝑘3subscript𝑝superscript𝑥1𝑘𝑝superscriptsubscript𝑘33𝑥𝑥𝑂3𝑥𝑥\displaystyle\sum_{k=3}^{\infty}\sum_{p\leq x^{1/k}}\log p\leq\sum_{k=3}^{% \infty}\sqrt[3]{x}\log x=O(\sqrt[3]{x}\log x)∑ start_POSTSUBSCRIPT italic_k = 3 end_POSTSUBSCRIPT start_POSTSUPERSCRIPT ∞ end_POSTSUPERSCRIPT ∑ start_POSTSUBSCRIPT italic_p ≤ italic_x start_POSTSUPERSCRIPT 1 / italic_k end_POSTSUPERSCRIPT end_POSTSUBSCRIPT roman_log italic_p ≤ ∑ start_POSTSUBSCRIPT italic_k = 3 end_POSTSUBSCRIPT start_POSTSUPERSCRIPT ∞ end_POSTSUPERSCRIPT nth-root start_ARG 3 end_ARG start_ARG italic_x end_ARG roman_log italic_x = italic_O ( nth-root start_ARG 3 end_ARG start_ARG italic_x end_ARG roman_log italic_x )

Therefore, the error term is dominated by the term corresponding to k=2𝑘2k=2italic_k = 2, in other words,

k=2px1/klogp=O(xlogx).superscriptsubscript𝑘2subscript𝑝superscript𝑥1𝑘𝑝𝑂𝑥𝑥\displaystyle\sum_{k=2}^{\infty}\sum_{p\leq x^{1/k}}\log p=O(\sqrt{x}\log x).∑ start_POSTSUBSCRIPT italic_k = 2 end_POSTSUBSCRIPT start_POSTSUPERSCRIPT ∞ end_POSTSUPERSCRIPT ∑ start_POSTSUBSCRIPT italic_p ≤ italic_x start_POSTSUPERSCRIPT 1 / italic_k end_POSTSUPERSCRIPT end_POSTSUBSCRIPT roman_log italic_p = italic_O ( square-root start_ARG italic_x end_ARG roman_log italic_x ) .

Combining the main term and the error terms,

ψ(x)=pxlogp+O(xlogx)𝜓𝑥subscript𝑝𝑥𝑝𝑂𝑥𝑥\displaystyle\psi(x)=\sum_{p\leq x}\log p+O(\sqrt{x}\log x)italic_ψ ( italic_x ) = ∑ start_POSTSUBSCRIPT italic_p ≤ italic_x end_POSTSUBSCRIPT roman_log italic_p + italic_O ( square-root start_ARG italic_x end_ARG roman_log italic_x ) (3.11)

On the other hand, applying the statement of Prime Number Theorem, we can have the following approximation,

pxlogpπ(x)logxsimilar-tosubscript𝑝𝑥𝑝𝜋𝑥𝑥\displaystyle\sum_{p\leq x}\log p\sim\pi(x)\log x∑ start_POSTSUBSCRIPT italic_p ≤ italic_x end_POSTSUBSCRIPT roman_log italic_p ∼ italic_π ( italic_x ) roman_log italic_x

More precisely,

pxlogp=π(x)logx+ϵ(x)subscript𝑝𝑥𝑝𝜋𝑥𝑥italic-ϵ𝑥\displaystyle\sum_{p\leq x}\log p=\pi(x)\log x+\epsilon(x)∑ start_POSTSUBSCRIPT italic_p ≤ italic_x end_POSTSUBSCRIPT roman_log italic_p = italic_π ( italic_x ) roman_log italic_x + italic_ϵ ( italic_x ) (3.12)

Given our earlier discussion, the error term, ϵ(x)italic-ϵ𝑥\epsilon(x)italic_ϵ ( italic_x ) can be bounded by,

ϵ(x)=O(xlogx).italic-ϵ𝑥𝑂𝑥𝑥\displaystyle\epsilon(x)=O(\sqrt{x}\log x).italic_ϵ ( italic_x ) = italic_O ( square-root start_ARG italic_x end_ARG roman_log italic_x ) .

To isolate π(x)𝜋𝑥\pi(x)italic_π ( italic_x ) from (3.12), we divide both sides by logx𝑥\log xroman_log italic_x,

π(x)=ψ(x)logxϵ(x)logx𝜋𝑥𝜓𝑥𝑥italic-ϵ𝑥𝑥\displaystyle\pi(x)=\frac{\psi(x)}{\log x}-\frac{\epsilon(x)}{\log x}italic_π ( italic_x ) = divide start_ARG italic_ψ ( italic_x ) end_ARG start_ARG roman_log italic_x end_ARG - divide start_ARG italic_ϵ ( italic_x ) end_ARG start_ARG roman_log italic_x end_ARG

Substituting the error term bound:

π(x)=ψ(x)logx+O(xlogxlogx)=ψ(x)logx+O(x1)𝜋𝑥𝜓𝑥𝑥𝑂𝑥𝑥𝑥𝜓𝑥𝑥𝑂𝑥1\displaystyle\pi(x)=\frac{\psi(x)}{\log x}+O\left(\frac{\sqrt{x}\log x}{\log x% }\right)=\frac{\psi(x)}{\log x}+O\left(\frac{\sqrt{x}}{1}\right)italic_π ( italic_x ) = divide start_ARG italic_ψ ( italic_x ) end_ARG start_ARG roman_log italic_x end_ARG + italic_O ( divide start_ARG square-root start_ARG italic_x end_ARG roman_log italic_x end_ARG start_ARG roman_log italic_x end_ARG ) = divide start_ARG italic_ψ ( italic_x ) end_ARG start_ARG roman_log italic_x end_ARG + italic_O ( divide start_ARG square-root start_ARG italic_x end_ARG end_ARG start_ARG 1 end_ARG )

From (3.7) in Lemma (3.2.1), we can further refine,

ψ(x)logx=xlogx+O(xlog2xlogx)=xlogx+O(xlogx)𝜓𝑥𝑥𝑥𝑥𝑂𝑥superscript2𝑥𝑥𝑥𝑥𝑂𝑥𝑥\displaystyle\frac{\psi(x)}{\log x}=\frac{x}{\log x}+O\left(\frac{\sqrt{x}\log% ^{2}x}{\log x}\right)=\frac{x}{\log x}+O(\sqrt{x}\log x)divide start_ARG italic_ψ ( italic_x ) end_ARG start_ARG roman_log italic_x end_ARG = divide start_ARG italic_x end_ARG start_ARG roman_log italic_x end_ARG + italic_O ( divide start_ARG square-root start_ARG italic_x end_ARG roman_log start_POSTSUPERSCRIPT 2 end_POSTSUPERSCRIPT italic_x end_ARG start_ARG roman_log italic_x end_ARG ) = divide start_ARG italic_x end_ARG start_ARG roman_log italic_x end_ARG + italic_O ( square-root start_ARG italic_x end_ARG roman_log italic_x )

Combining the error terms, and applying Theorem (2.1.1) again enables us to assert that,

π(x)=ψ(x)logx+O(x(logx)2).𝜋𝑥𝜓𝑥𝑥𝑂𝑥superscript𝑥2\displaystyle\pi(x)=\frac{\psi(x)}{\log x}+O\left(\frac{x}{(\log x)^{2}}\right).italic_π ( italic_x ) = divide start_ARG italic_ψ ( italic_x ) end_ARG start_ARG roman_log italic_x end_ARG + italic_O ( divide start_ARG italic_x end_ARG start_ARG ( roman_log italic_x ) start_POSTSUPERSCRIPT 2 end_POSTSUPERSCRIPT end_ARG ) .

3.3 An improved estimate for π(x)𝜋𝑥\pi(x)italic_π ( italic_x )

First, let us recall that, (3.1) in Theorem (3.1.1) gives us an order estimate for ψ(x)𝜓𝑥\psi(x)italic_ψ ( italic_x ) in terms of M(x)𝑀𝑥M(x)italic_M ( italic_x ), whereas, we have derived a unique representation of π(x)𝜋𝑥\pi(x)italic_π ( italic_x ) applying properties of ψ(x)𝜓𝑥\psi(x)italic_ψ ( italic_x ), and analytic properties of the Riemann Zeta Function ζ(s)𝜁𝑠\zeta(s)italic_ζ ( italic_s ), as mentioned in (3.9) in Theorem (3.2.2).

We substitute (3.1) into (3.9) to obtain,

π(x)=xnxM(xn)+O(x)logx+O(x(logx)2)𝜋𝑥𝑥subscript𝑛𝑥𝑀𝑥𝑛𝑂𝑥𝑥𝑂𝑥superscript𝑥2\displaystyle\pi(x)=\frac{x-\sum_{n\leq x}M\left(\frac{x}{n}\right)+O(\sqrt{x}% )}{\log x}+O\left(\frac{x}{(\log x)^{2}}\right)italic_π ( italic_x ) = divide start_ARG italic_x - ∑ start_POSTSUBSCRIPT italic_n ≤ italic_x end_POSTSUBSCRIPT italic_M ( divide start_ARG italic_x end_ARG start_ARG italic_n end_ARG ) + italic_O ( square-root start_ARG italic_x end_ARG ) end_ARG start_ARG roman_log italic_x end_ARG + italic_O ( divide start_ARG italic_x end_ARG start_ARG ( roman_log italic_x ) start_POSTSUPERSCRIPT 2 end_POSTSUPERSCRIPT end_ARG )
=xlogx1logxnxM(xn)+O(x)logx+O(x(logx)2)absent𝑥𝑥1𝑥subscript𝑛𝑥𝑀𝑥𝑛𝑂𝑥𝑥𝑂𝑥superscript𝑥2\displaystyle\hskip 50.0pt=\frac{x}{\log x}-\frac{1}{\log x}\sum_{n\leq x}M% \left(\frac{x}{n}\right)+\frac{O(\sqrt{x})}{\log x}+O\left(\frac{x}{(\log x)^{% 2}}\right)= divide start_ARG italic_x end_ARG start_ARG roman_log italic_x end_ARG - divide start_ARG 1 end_ARG start_ARG roman_log italic_x end_ARG ∑ start_POSTSUBSCRIPT italic_n ≤ italic_x end_POSTSUBSCRIPT italic_M ( divide start_ARG italic_x end_ARG start_ARG italic_n end_ARG ) + divide start_ARG italic_O ( square-root start_ARG italic_x end_ARG ) end_ARG start_ARG roman_log italic_x end_ARG + italic_O ( divide start_ARG italic_x end_ARG start_ARG ( roman_log italic_x ) start_POSTSUPERSCRIPT 2 end_POSTSUPERSCRIPT end_ARG )

Combining the error terms,

π(x)=xlogx1logxnxM(xn)+O(xlogx)+O(x(logx)2).𝜋𝑥𝑥𝑥1𝑥subscript𝑛𝑥𝑀𝑥𝑛𝑂𝑥𝑥𝑂𝑥superscript𝑥2\displaystyle\pi(x)=\frac{x}{\log x}-\frac{1}{\log x}\sum_{n\leq x}M\left(% \frac{x}{n}\right)+O\left(\frac{\sqrt{x}}{\log x}\right)+O\left(\frac{x}{(\log x% )^{2}}\right).italic_π ( italic_x ) = divide start_ARG italic_x end_ARG start_ARG roman_log italic_x end_ARG - divide start_ARG 1 end_ARG start_ARG roman_log italic_x end_ARG ∑ start_POSTSUBSCRIPT italic_n ≤ italic_x end_POSTSUBSCRIPT italic_M ( divide start_ARG italic_x end_ARG start_ARG italic_n end_ARG ) + italic_O ( divide start_ARG square-root start_ARG italic_x end_ARG end_ARG start_ARG roman_log italic_x end_ARG ) + italic_O ( divide start_ARG italic_x end_ARG start_ARG ( roman_log italic_x ) start_POSTSUPERSCRIPT 2 end_POSTSUPERSCRIPT end_ARG ) . (3.13)

Note that O(xlogx)𝑂𝑥𝑥O\left(\frac{\sqrt{x}}{\log x}\right)italic_O ( divide start_ARG square-root start_ARG italic_x end_ARG end_ARG start_ARG roman_log italic_x end_ARG ) is asymptotically smaller than O(xlog2x)𝑂𝑥superscript2𝑥O\left(\frac{x}{\log^{2}x}\right)italic_O ( divide start_ARG italic_x end_ARG start_ARG roman_log start_POSTSUPERSCRIPT 2 end_POSTSUPERSCRIPT italic_x end_ARG ), so the dominant error term is O(x(logx)2)𝑂𝑥superscript𝑥2O\left(\frac{x}{(\log x)^{2}}\right)italic_O ( divide start_ARG italic_x end_ARG start_ARG ( roman_log italic_x ) start_POSTSUPERSCRIPT 2 end_POSTSUPERSCRIPT end_ARG ). Therefore, we have the following improved estimate for π(x)𝜋𝑥\pi(x)italic_π ( italic_x ) as follows.

Theorem 3.3.1.
π(x)=xlogx1logxnxM(xn)+O(x(logx)2)𝜋𝑥𝑥𝑥1𝑥subscript𝑛𝑥𝑀𝑥𝑛𝑂𝑥superscript𝑥2\displaystyle\pi(x)=\frac{x}{\log x}-\frac{1}{\log x}\sum_{n\leq x}M\left(% \frac{x}{n}\right)+O\left(\frac{x}{(\log x)^{2}}\right)italic_π ( italic_x ) = divide start_ARG italic_x end_ARG start_ARG roman_log italic_x end_ARG - divide start_ARG 1 end_ARG start_ARG roman_log italic_x end_ARG ∑ start_POSTSUBSCRIPT italic_n ≤ italic_x end_POSTSUBSCRIPT italic_M ( divide start_ARG italic_x end_ARG start_ARG italic_n end_ARG ) + italic_O ( divide start_ARG italic_x end_ARG start_ARG ( roman_log italic_x ) start_POSTSUPERSCRIPT 2 end_POSTSUPERSCRIPT end_ARG ) (3.14)

4 Proving Ramanujan’s Inequality

In order to prove Ramanujan’s Inequality, our primary intention will be to investigate the sign of the function,

𝒢(x):=(π(x))2exlog(x)π(xe)assign𝒢𝑥superscript𝜋𝑥2𝑒𝑥𝑥𝜋𝑥𝑒\displaystyle\mathcal{G}(x):=(\pi(x))^{2}-\frac{ex}{\log(x)}\pi\left(\frac{x}{% e}\right)caligraphic_G ( italic_x ) := ( italic_π ( italic_x ) ) start_POSTSUPERSCRIPT 2 end_POSTSUPERSCRIPT - divide start_ARG italic_e italic_x end_ARG start_ARG roman_log ( italic_x ) end_ARG italic_π ( divide start_ARG italic_x end_ARG start_ARG italic_e end_ARG ) (4.1)

for large values of x𝑥xitalic_x using the relationship between π(x)𝜋𝑥\pi(x)italic_π ( italic_x ) and the Mertens function M(x)𝑀𝑥M(x)italic_M ( italic_x ). Given the complexity of the order relations an the extent of robust computations involving these functions as discussed in the previous section, we’ll have to analyze the expressions and error terms cautiously.

4.1 An order expression for 𝒢(x)𝒢𝑥\mathcal{G}(x)caligraphic_G ( italic_x )

A priori from (3.14) of Theorem (3.3.1), we have the following expressions for π(x)𝜋𝑥\pi(x)italic_π ( italic_x ) and π(x/e)𝜋𝑥𝑒\pi(x/e)italic_π ( italic_x / italic_e ) as,

π(x)=xlogx1logxnxM(xn)+O(x(logx)2)𝜋𝑥𝑥𝑥1𝑥subscript𝑛𝑥𝑀𝑥𝑛𝑂𝑥superscript𝑥2\displaystyle\pi(x)=\frac{x}{\log x}-\frac{1}{\log x}\sum_{n\leq x}M\left(% \frac{x}{n}\right)+O\left(\frac{x}{(\log x)^{2}}\right)italic_π ( italic_x ) = divide start_ARG italic_x end_ARG start_ARG roman_log italic_x end_ARG - divide start_ARG 1 end_ARG start_ARG roman_log italic_x end_ARG ∑ start_POSTSUBSCRIPT italic_n ≤ italic_x end_POSTSUBSCRIPT italic_M ( divide start_ARG italic_x end_ARG start_ARG italic_n end_ARG ) + italic_O ( divide start_ARG italic_x end_ARG start_ARG ( roman_log italic_x ) start_POSTSUPERSCRIPT 2 end_POSTSUPERSCRIPT end_ARG )

and,

π(x/e)=x/elog(x/e)1log(x/e)nx/eM(x/en)+O(x/e(log(x/e))2)𝜋𝑥𝑒𝑥𝑒𝑥𝑒1𝑥𝑒subscript𝑛𝑥𝑒𝑀𝑥𝑒𝑛𝑂𝑥𝑒superscript𝑥𝑒2\displaystyle\pi(x/e)=\frac{x/e}{\log(x/e)}-\frac{1}{\log(x/e)}\sum_{n\leq x/e% }M\left(\frac{x/e}{n}\right)+O\left(\frac{x/e}{(\log(x/e))^{2}}\right)italic_π ( italic_x / italic_e ) = divide start_ARG italic_x / italic_e end_ARG start_ARG roman_log ( italic_x / italic_e ) end_ARG - divide start_ARG 1 end_ARG start_ARG roman_log ( italic_x / italic_e ) end_ARG ∑ start_POSTSUBSCRIPT italic_n ≤ italic_x / italic_e end_POSTSUBSCRIPT italic_M ( divide start_ARG italic_x / italic_e end_ARG start_ARG italic_n end_ARG ) + italic_O ( divide start_ARG italic_x / italic_e end_ARG start_ARG ( roman_log ( italic_x / italic_e ) ) start_POSTSUPERSCRIPT 2 end_POSTSUPERSCRIPT end_ARG )
=x/elogx11logx1nx/eM(x/en)+O(x/e(logx1)2)absent𝑥𝑒𝑥11𝑥1subscript𝑛𝑥𝑒𝑀𝑥𝑒𝑛𝑂𝑥𝑒superscript𝑥12\displaystyle\hskip 50.0pt=\frac{x/e}{\log x-1}-\frac{1}{\log x-1}\sum_{n\leq x% /e}M\left(\frac{x/e}{n}\right)+O\left(\frac{x/e}{(\log x-1)^{2}}\right)= divide start_ARG italic_x / italic_e end_ARG start_ARG roman_log italic_x - 1 end_ARG - divide start_ARG 1 end_ARG start_ARG roman_log italic_x - 1 end_ARG ∑ start_POSTSUBSCRIPT italic_n ≤ italic_x / italic_e end_POSTSUBSCRIPT italic_M ( divide start_ARG italic_x / italic_e end_ARG start_ARG italic_n end_ARG ) + italic_O ( divide start_ARG italic_x / italic_e end_ARG start_ARG ( roman_log italic_x - 1 ) start_POSTSUPERSCRIPT 2 end_POSTSUPERSCRIPT end_ARG )

Subsequently,

exlogxπ(x/e)=exlogx(x/elogx11logx1nx/eM(x/en)+O(x/e(logx1)2))𝑒𝑥𝑥𝜋𝑥𝑒𝑒𝑥𝑥𝑥𝑒𝑥11𝑥1subscript𝑛𝑥𝑒𝑀𝑥𝑒𝑛𝑂𝑥𝑒superscript𝑥12\displaystyle\frac{ex}{\log x}\pi(x/e)=\frac{ex}{\log x}\left(\frac{x/e}{\log x% -1}-\frac{1}{\log x-1}\sum_{n\leq x/e}M\left(\frac{x/e}{n}\right)+O\left(\frac% {x/e}{(\log x-1)^{2}}\right)\right)divide start_ARG italic_e italic_x end_ARG start_ARG roman_log italic_x end_ARG italic_π ( italic_x / italic_e ) = divide start_ARG italic_e italic_x end_ARG start_ARG roman_log italic_x end_ARG ( divide start_ARG italic_x / italic_e end_ARG start_ARG roman_log italic_x - 1 end_ARG - divide start_ARG 1 end_ARG start_ARG roman_log italic_x - 1 end_ARG ∑ start_POSTSUBSCRIPT italic_n ≤ italic_x / italic_e end_POSTSUBSCRIPT italic_M ( divide start_ARG italic_x / italic_e end_ARG start_ARG italic_n end_ARG ) + italic_O ( divide start_ARG italic_x / italic_e end_ARG start_ARG ( roman_log italic_x - 1 ) start_POSTSUPERSCRIPT 2 end_POSTSUPERSCRIPT end_ARG ) )
=x2logx(logx1)exlogx(logx1)nx/eM(x/en)+O(x2logx(logx1)2)absentsuperscript𝑥2𝑥𝑥1𝑒𝑥𝑥𝑥1subscript𝑛𝑥𝑒𝑀𝑥𝑒𝑛𝑂superscript𝑥2𝑥superscript𝑥12\displaystyle\hskip 30.0pt=\frac{x^{2}}{\log x(\log x-1)}-\frac{ex}{\log x(% \log x-1)}\sum_{n\leq x/e}M\left(\frac{x/e}{n}\right)+O\left(\frac{x^{2}}{\log x% (\log x-1)^{2}}\right)= divide start_ARG italic_x start_POSTSUPERSCRIPT 2 end_POSTSUPERSCRIPT end_ARG start_ARG roman_log italic_x ( roman_log italic_x - 1 ) end_ARG - divide start_ARG italic_e italic_x end_ARG start_ARG roman_log italic_x ( roman_log italic_x - 1 ) end_ARG ∑ start_POSTSUBSCRIPT italic_n ≤ italic_x / italic_e end_POSTSUBSCRIPT italic_M ( divide start_ARG italic_x / italic_e end_ARG start_ARG italic_n end_ARG ) + italic_O ( divide start_ARG italic_x start_POSTSUPERSCRIPT 2 end_POSTSUPERSCRIPT end_ARG start_ARG roman_log italic_x ( roman_log italic_x - 1 ) start_POSTSUPERSCRIPT 2 end_POSTSUPERSCRIPT end_ARG ) (4.2)

Furthermore, squaring π(x)𝜋𝑥\pi(x)italic_π ( italic_x ) yields,

(π(x))2=(xlogx1logxnxM(xn)+O(x(logx)2))2superscript𝜋𝑥2superscript𝑥𝑥1𝑥subscript𝑛𝑥𝑀𝑥𝑛𝑂𝑥superscript𝑥22\displaystyle(\pi(x))^{2}=\left(\frac{x}{\log x}-\frac{1}{\log x}\sum_{n\leq x% }M\left(\frac{x}{n}\right)+O\left(\frac{x}{(\log x)^{2}}\right)\right)^{2}( italic_π ( italic_x ) ) start_POSTSUPERSCRIPT 2 end_POSTSUPERSCRIPT = ( divide start_ARG italic_x end_ARG start_ARG roman_log italic_x end_ARG - divide start_ARG 1 end_ARG start_ARG roman_log italic_x end_ARG ∑ start_POSTSUBSCRIPT italic_n ≤ italic_x end_POSTSUBSCRIPT italic_M ( divide start_ARG italic_x end_ARG start_ARG italic_n end_ARG ) + italic_O ( divide start_ARG italic_x end_ARG start_ARG ( roman_log italic_x ) start_POSTSUPERSCRIPT 2 end_POSTSUPERSCRIPT end_ARG ) ) start_POSTSUPERSCRIPT 2 end_POSTSUPERSCRIPT
=(xlogx)22(xlogx)(1logxnxM(xn))+(1logxnxM(xn))2+O(x2(logx)3)absentsuperscript𝑥𝑥22𝑥𝑥1𝑥subscript𝑛𝑥𝑀𝑥𝑛superscript1𝑥subscript𝑛𝑥𝑀𝑥𝑛2𝑂superscript𝑥2superscript𝑥3\displaystyle=\left(\frac{x}{\log x}\right)^{2}-2\left(\frac{x}{\log x}\right)% \left(\frac{1}{\log x}\sum_{n\leq x}M\left(\frac{x}{n}\right)\right)+\left(% \frac{1}{\log x}\sum_{n\leq x}M\left(\frac{x}{n}\right)\right)^{2}+O\left(% \frac{x^{2}}{(\log x)^{3}}\right)= ( divide start_ARG italic_x end_ARG start_ARG roman_log italic_x end_ARG ) start_POSTSUPERSCRIPT 2 end_POSTSUPERSCRIPT - 2 ( divide start_ARG italic_x end_ARG start_ARG roman_log italic_x end_ARG ) ( divide start_ARG 1 end_ARG start_ARG roman_log italic_x end_ARG ∑ start_POSTSUBSCRIPT italic_n ≤ italic_x end_POSTSUBSCRIPT italic_M ( divide start_ARG italic_x end_ARG start_ARG italic_n end_ARG ) ) + ( divide start_ARG 1 end_ARG start_ARG roman_log italic_x end_ARG ∑ start_POSTSUBSCRIPT italic_n ≤ italic_x end_POSTSUBSCRIPT italic_M ( divide start_ARG italic_x end_ARG start_ARG italic_n end_ARG ) ) start_POSTSUPERSCRIPT 2 end_POSTSUPERSCRIPT + italic_O ( divide start_ARG italic_x start_POSTSUPERSCRIPT 2 end_POSTSUPERSCRIPT end_ARG start_ARG ( roman_log italic_x ) start_POSTSUPERSCRIPT 3 end_POSTSUPERSCRIPT end_ARG )
=x2(logx)22x(logx)2nxM(xn)+1(logx)2(nxM(xn))2+O(x2(logx)3)absentsuperscript𝑥2superscript𝑥22𝑥superscript𝑥2subscript𝑛𝑥𝑀𝑥𝑛1superscript𝑥2superscriptsubscript𝑛𝑥𝑀𝑥𝑛2𝑂superscript𝑥2superscript𝑥3\displaystyle=\frac{x^{2}}{(\log x)^{2}}-\frac{2x}{(\log x)^{2}}\sum_{n\leq x}% M\left(\frac{x}{n}\right)+\frac{1}{(\log x)^{2}}\left(\sum_{n\leq x}M\left(% \frac{x}{n}\right)\right)^{2}+O\left(\frac{x^{2}}{(\log x)^{3}}\right)= divide start_ARG italic_x start_POSTSUPERSCRIPT 2 end_POSTSUPERSCRIPT end_ARG start_ARG ( roman_log italic_x ) start_POSTSUPERSCRIPT 2 end_POSTSUPERSCRIPT end_ARG - divide start_ARG 2 italic_x end_ARG start_ARG ( roman_log italic_x ) start_POSTSUPERSCRIPT 2 end_POSTSUPERSCRIPT end_ARG ∑ start_POSTSUBSCRIPT italic_n ≤ italic_x end_POSTSUBSCRIPT italic_M ( divide start_ARG italic_x end_ARG start_ARG italic_n end_ARG ) + divide start_ARG 1 end_ARG start_ARG ( roman_log italic_x ) start_POSTSUPERSCRIPT 2 end_POSTSUPERSCRIPT end_ARG ( ∑ start_POSTSUBSCRIPT italic_n ≤ italic_x end_POSTSUBSCRIPT italic_M ( divide start_ARG italic_x end_ARG start_ARG italic_n end_ARG ) ) start_POSTSUPERSCRIPT 2 end_POSTSUPERSCRIPT + italic_O ( divide start_ARG italic_x start_POSTSUPERSCRIPT 2 end_POSTSUPERSCRIPT end_ARG start_ARG ( roman_log italic_x ) start_POSTSUPERSCRIPT 3 end_POSTSUPERSCRIPT end_ARG ) (4.3)

Therefore, using (4.2) and (4.3), we deduce that,

𝒢(x)=(x2(logx)22x(logx)2nxM(xn)+1(logx)2(nxM(xn))2+O(x2(logx)3))𝒢𝑥superscript𝑥2superscript𝑥22𝑥superscript𝑥2subscript𝑛𝑥𝑀𝑥𝑛1superscript𝑥2superscriptsubscript𝑛𝑥𝑀𝑥𝑛2𝑂superscript𝑥2superscript𝑥3\displaystyle\mathcal{G}(x)=\left(\frac{x^{2}}{(\log x)^{2}}-\frac{2x}{(\log x% )^{2}}\sum_{n\leq x}M\left(\frac{x}{n}\right)+\frac{1}{(\log x)^{2}}\left(\sum% _{n\leq x}M\left(\frac{x}{n}\right)\right)^{2}+O\left(\frac{x^{2}}{(\log x)^{3% }}\right)\right)caligraphic_G ( italic_x ) = ( divide start_ARG italic_x start_POSTSUPERSCRIPT 2 end_POSTSUPERSCRIPT end_ARG start_ARG ( roman_log italic_x ) start_POSTSUPERSCRIPT 2 end_POSTSUPERSCRIPT end_ARG - divide start_ARG 2 italic_x end_ARG start_ARG ( roman_log italic_x ) start_POSTSUPERSCRIPT 2 end_POSTSUPERSCRIPT end_ARG ∑ start_POSTSUBSCRIPT italic_n ≤ italic_x end_POSTSUBSCRIPT italic_M ( divide start_ARG italic_x end_ARG start_ARG italic_n end_ARG ) + divide start_ARG 1 end_ARG start_ARG ( roman_log italic_x ) start_POSTSUPERSCRIPT 2 end_POSTSUPERSCRIPT end_ARG ( ∑ start_POSTSUBSCRIPT italic_n ≤ italic_x end_POSTSUBSCRIPT italic_M ( divide start_ARG italic_x end_ARG start_ARG italic_n end_ARG ) ) start_POSTSUPERSCRIPT 2 end_POSTSUPERSCRIPT + italic_O ( divide start_ARG italic_x start_POSTSUPERSCRIPT 2 end_POSTSUPERSCRIPT end_ARG start_ARG ( roman_log italic_x ) start_POSTSUPERSCRIPT 3 end_POSTSUPERSCRIPT end_ARG ) )
(x2logx(logx1)exlogx(logx1)nx/eM(x/en)+O(x2logx(logx1)2)).superscript𝑥2𝑥𝑥1𝑒𝑥𝑥𝑥1subscript𝑛𝑥𝑒𝑀𝑥𝑒𝑛𝑂superscript𝑥2𝑥superscript𝑥12\displaystyle\hskip 50.0pt-\left(\frac{x^{2}}{\log x(\log x-1)}-\frac{ex}{\log x% (\log x-1)}\sum_{n\leq x/e}M\left(\frac{x/e}{n}\right)+O\left(\frac{x^{2}}{% \log x(\log x-1)^{2}}\right)\right).- ( divide start_ARG italic_x start_POSTSUPERSCRIPT 2 end_POSTSUPERSCRIPT end_ARG start_ARG roman_log italic_x ( roman_log italic_x - 1 ) end_ARG - divide start_ARG italic_e italic_x end_ARG start_ARG roman_log italic_x ( roman_log italic_x - 1 ) end_ARG ∑ start_POSTSUBSCRIPT italic_n ≤ italic_x / italic_e end_POSTSUBSCRIPT italic_M ( divide start_ARG italic_x / italic_e end_ARG start_ARG italic_n end_ARG ) + italic_O ( divide start_ARG italic_x start_POSTSUPERSCRIPT 2 end_POSTSUPERSCRIPT end_ARG start_ARG roman_log italic_x ( roman_log italic_x - 1 ) start_POSTSUPERSCRIPT 2 end_POSTSUPERSCRIPT end_ARG ) ) .

Simplifying each term:

=x2(logx)2x2logx(logx1)2x(logx)2nxM(xn)+1(logx)2(nxM(xn))2absentsuperscript𝑥2superscript𝑥2superscript𝑥2𝑥𝑥12𝑥superscript𝑥2subscript𝑛𝑥𝑀𝑥𝑛1superscript𝑥2superscriptsubscript𝑛𝑥𝑀𝑥𝑛2\displaystyle=\frac{x^{2}}{(\log x)^{2}}-\frac{x^{2}}{\log x(\log x-1)}-\frac{% 2x}{(\log x)^{2}}\sum_{n\leq x}M\left(\frac{x}{n}\right)+\frac{1}{(\log x)^{2}% }\left(\sum_{n\leq x}M\left(\frac{x}{n}\right)\right)^{2}= divide start_ARG italic_x start_POSTSUPERSCRIPT 2 end_POSTSUPERSCRIPT end_ARG start_ARG ( roman_log italic_x ) start_POSTSUPERSCRIPT 2 end_POSTSUPERSCRIPT end_ARG - divide start_ARG italic_x start_POSTSUPERSCRIPT 2 end_POSTSUPERSCRIPT end_ARG start_ARG roman_log italic_x ( roman_log italic_x - 1 ) end_ARG - divide start_ARG 2 italic_x end_ARG start_ARG ( roman_log italic_x ) start_POSTSUPERSCRIPT 2 end_POSTSUPERSCRIPT end_ARG ∑ start_POSTSUBSCRIPT italic_n ≤ italic_x end_POSTSUBSCRIPT italic_M ( divide start_ARG italic_x end_ARG start_ARG italic_n end_ARG ) + divide start_ARG 1 end_ARG start_ARG ( roman_log italic_x ) start_POSTSUPERSCRIPT 2 end_POSTSUPERSCRIPT end_ARG ( ∑ start_POSTSUBSCRIPT italic_n ≤ italic_x end_POSTSUBSCRIPT italic_M ( divide start_ARG italic_x end_ARG start_ARG italic_n end_ARG ) ) start_POSTSUPERSCRIPT 2 end_POSTSUPERSCRIPT
+exlogx(logx1)nx/eM(x/en)+O(x2(logx)3).𝑒𝑥𝑥𝑥1subscript𝑛𝑥𝑒𝑀𝑥𝑒𝑛𝑂superscript𝑥2superscript𝑥3\displaystyle\hskip 200.0pt+\frac{ex}{\log x(\log x-1)}\sum_{n\leq x/e}M\left(% \frac{x/e}{n}\right)+O\left(\frac{x^{2}}{(\log x)^{3}}\right).+ divide start_ARG italic_e italic_x end_ARG start_ARG roman_log italic_x ( roman_log italic_x - 1 ) end_ARG ∑ start_POSTSUBSCRIPT italic_n ≤ italic_x / italic_e end_POSTSUBSCRIPT italic_M ( divide start_ARG italic_x / italic_e end_ARG start_ARG italic_n end_ARG ) + italic_O ( divide start_ARG italic_x start_POSTSUPERSCRIPT 2 end_POSTSUPERSCRIPT end_ARG start_ARG ( roman_log italic_x ) start_POSTSUPERSCRIPT 3 end_POSTSUPERSCRIPT end_ARG ) . (4.4)
=x2(logx)2(logx1)2x(logx)2nxM(xn)+1(logx)2(nxM(xn))2absentsuperscript𝑥2superscript𝑥2𝑥12𝑥superscript𝑥2subscript𝑛𝑥𝑀𝑥𝑛1superscript𝑥2superscriptsubscript𝑛𝑥𝑀𝑥𝑛2\displaystyle=\frac{x^{2}}{(\log x)^{2}(\log x-1)}-\frac{2x}{(\log x)^{2}}\sum% _{n\leq x}M\left(\frac{x}{n}\right)+\frac{1}{(\log x)^{2}}\left(\sum_{n\leq x}% M\left(\frac{x}{n}\right)\right)^{2}= divide start_ARG italic_x start_POSTSUPERSCRIPT 2 end_POSTSUPERSCRIPT end_ARG start_ARG ( roman_log italic_x ) start_POSTSUPERSCRIPT 2 end_POSTSUPERSCRIPT ( roman_log italic_x - 1 ) end_ARG - divide start_ARG 2 italic_x end_ARG start_ARG ( roman_log italic_x ) start_POSTSUPERSCRIPT 2 end_POSTSUPERSCRIPT end_ARG ∑ start_POSTSUBSCRIPT italic_n ≤ italic_x end_POSTSUBSCRIPT italic_M ( divide start_ARG italic_x end_ARG start_ARG italic_n end_ARG ) + divide start_ARG 1 end_ARG start_ARG ( roman_log italic_x ) start_POSTSUPERSCRIPT 2 end_POSTSUPERSCRIPT end_ARG ( ∑ start_POSTSUBSCRIPT italic_n ≤ italic_x end_POSTSUBSCRIPT italic_M ( divide start_ARG italic_x end_ARG start_ARG italic_n end_ARG ) ) start_POSTSUPERSCRIPT 2 end_POSTSUPERSCRIPT
+exlogx(logx1)nx/eM(x/en)+O(x2(logx)3)𝑒𝑥𝑥𝑥1subscript𝑛𝑥𝑒𝑀𝑥𝑒𝑛𝑂superscript𝑥2superscript𝑥3\displaystyle\hskip 150.0pt+\frac{ex}{\log x(\log x-1)}\sum_{n\leq x/e}M\left(% \frac{x/e}{n}\right)+O\left(\frac{x^{2}}{(\log x)^{3}}\right)+ divide start_ARG italic_e italic_x end_ARG start_ARG roman_log italic_x ( roman_log italic_x - 1 ) end_ARG ∑ start_POSTSUBSCRIPT italic_n ≤ italic_x / italic_e end_POSTSUBSCRIPT italic_M ( divide start_ARG italic_x / italic_e end_ARG start_ARG italic_n end_ARG ) + italic_O ( divide start_ARG italic_x start_POSTSUPERSCRIPT 2 end_POSTSUPERSCRIPT end_ARG start_ARG ( roman_log italic_x ) start_POSTSUPERSCRIPT 3 end_POSTSUPERSCRIPT end_ARG ) (4.5)

4.2 Asymptotic Behavior of indivudual Terms

A priori using involving the Mertens function M(x)𝑀𝑥M(x)italic_M ( italic_x ),

nxM(xn)=O(nxxn)=O(xnx1n)=O(xx)=O(x)subscript𝑛𝑥𝑀𝑥𝑛𝑂subscript𝑛𝑥𝑥𝑛𝑂𝑥subscript𝑛𝑥1𝑛𝑂𝑥𝑥𝑂𝑥\displaystyle\sum_{n\leq x}M\left(\frac{x}{n}\right)=O\left(\sum_{n\leq x}% \sqrt{\frac{x}{n}}\right)=O\left(\sqrt{x}\sum_{n\leq x}\frac{1}{\sqrt{n}}% \right)=O(\sqrt{x}\cdot\sqrt{x})=O(x)∑ start_POSTSUBSCRIPT italic_n ≤ italic_x end_POSTSUBSCRIPT italic_M ( divide start_ARG italic_x end_ARG start_ARG italic_n end_ARG ) = italic_O ( ∑ start_POSTSUBSCRIPT italic_n ≤ italic_x end_POSTSUBSCRIPT square-root start_ARG divide start_ARG italic_x end_ARG start_ARG italic_n end_ARG end_ARG ) = italic_O ( square-root start_ARG italic_x end_ARG ∑ start_POSTSUBSCRIPT italic_n ≤ italic_x end_POSTSUBSCRIPT divide start_ARG 1 end_ARG start_ARG square-root start_ARG italic_n end_ARG end_ARG ) = italic_O ( square-root start_ARG italic_x end_ARG ⋅ square-root start_ARG italic_x end_ARG ) = italic_O ( italic_x ) (4.6)

So, we can further approximate each term of (4.5) as follows,

1(logx)2(nxM(xn))2=O(x2(logx)2),1superscript𝑥2superscriptsubscript𝑛𝑥𝑀𝑥𝑛2𝑂superscript𝑥2superscript𝑥2\displaystyle\frac{1}{(\log x)^{2}}\left(\sum_{n\leq x}M\left(\frac{x}{n}% \right)\right)^{2}=O\left(\frac{x^{2}}{(\log x)^{2}}\right),divide start_ARG 1 end_ARG start_ARG ( roman_log italic_x ) start_POSTSUPERSCRIPT 2 end_POSTSUPERSCRIPT end_ARG ( ∑ start_POSTSUBSCRIPT italic_n ≤ italic_x end_POSTSUBSCRIPT italic_M ( divide start_ARG italic_x end_ARG start_ARG italic_n end_ARG ) ) start_POSTSUPERSCRIPT 2 end_POSTSUPERSCRIPT = italic_O ( divide start_ARG italic_x start_POSTSUPERSCRIPT 2 end_POSTSUPERSCRIPT end_ARG start_ARG ( roman_log italic_x ) start_POSTSUPERSCRIPT 2 end_POSTSUPERSCRIPT end_ARG ) ,
exlogx(logx1)nx/eM(x/en)=O(x2logx(logx1)).𝑒𝑥𝑥𝑥1subscript𝑛𝑥𝑒𝑀𝑥𝑒𝑛𝑂superscript𝑥2𝑥𝑥1\displaystyle\frac{ex}{\log x(\log x-1)}\sum_{n\leq x/e}M\left(\frac{x/e}{n}% \right)=O\left(\frac{x^{2}}{\log x(\log x-1)}\right).divide start_ARG italic_e italic_x end_ARG start_ARG roman_log italic_x ( roman_log italic_x - 1 ) end_ARG ∑ start_POSTSUBSCRIPT italic_n ≤ italic_x / italic_e end_POSTSUBSCRIPT italic_M ( divide start_ARG italic_x / italic_e end_ARG start_ARG italic_n end_ARG ) = italic_O ( divide start_ARG italic_x start_POSTSUPERSCRIPT 2 end_POSTSUPERSCRIPT end_ARG start_ARG roman_log italic_x ( roman_log italic_x - 1 ) end_ARG ) .

As a consequence,

𝒢(x)=2x(logx)2nxM(xn)+O(x2(logx)2)+O(x2logx(logx1))+O(x2(logx)3).𝒢𝑥2𝑥superscript𝑥2subscript𝑛𝑥𝑀𝑥𝑛𝑂superscript𝑥2superscript𝑥2𝑂superscript𝑥2𝑥𝑥1𝑂superscript𝑥2superscript𝑥3\displaystyle\mathcal{G}(x)=-\frac{2x}{(\log x)^{2}}\sum_{n\leq x}M\left(\frac% {x}{n}\right)+O\left(\frac{x^{2}}{(\log x)^{2}}\right)+O\left(\frac{x^{2}}{% \log x(\log x-1)}\right)+O\left(\frac{x^{2}}{(\log x)^{3}}\right).caligraphic_G ( italic_x ) = - divide start_ARG 2 italic_x end_ARG start_ARG ( roman_log italic_x ) start_POSTSUPERSCRIPT 2 end_POSTSUPERSCRIPT end_ARG ∑ start_POSTSUBSCRIPT italic_n ≤ italic_x end_POSTSUBSCRIPT italic_M ( divide start_ARG italic_x end_ARG start_ARG italic_n end_ARG ) + italic_O ( divide start_ARG italic_x start_POSTSUPERSCRIPT 2 end_POSTSUPERSCRIPT end_ARG start_ARG ( roman_log italic_x ) start_POSTSUPERSCRIPT 2 end_POSTSUPERSCRIPT end_ARG ) + italic_O ( divide start_ARG italic_x start_POSTSUPERSCRIPT 2 end_POSTSUPERSCRIPT end_ARG start_ARG roman_log italic_x ( roman_log italic_x - 1 ) end_ARG ) + italic_O ( divide start_ARG italic_x start_POSTSUPERSCRIPT 2 end_POSTSUPERSCRIPT end_ARG start_ARG ( roman_log italic_x ) start_POSTSUPERSCRIPT 3 end_POSTSUPERSCRIPT end_ARG ) .
=2x(logx)2nxM(xn)+O(x2(logx)2)absent2𝑥superscript𝑥2subscript𝑛𝑥𝑀𝑥𝑛𝑂superscript𝑥2superscript𝑥2\displaystyle\hskip 100.0pt=-\frac{2x}{(\log x)^{2}}\sum_{n\leq x}M\left(\frac% {x}{n}\right)+O\left(\frac{x^{2}}{(\log x)^{2}}\right)= - divide start_ARG 2 italic_x end_ARG start_ARG ( roman_log italic_x ) start_POSTSUPERSCRIPT 2 end_POSTSUPERSCRIPT end_ARG ∑ start_POSTSUBSCRIPT italic_n ≤ italic_x end_POSTSUBSCRIPT italic_M ( divide start_ARG italic_x end_ARG start_ARG italic_n end_ARG ) + italic_O ( divide start_ARG italic_x start_POSTSUPERSCRIPT 2 end_POSTSUPERSCRIPT end_ARG start_ARG ( roman_log italic_x ) start_POSTSUPERSCRIPT 2 end_POSTSUPERSCRIPT end_ARG ) (4.7)

For sufficiently large values of x𝑥xitalic_x. Therefore, we conclude that, 𝒢(x)<0𝒢𝑥0\mathcal{G}(x)<0caligraphic_G ( italic_x ) < 0, for large values of x𝑥xitalic_x, and this concludes our proof of the inequality.

later, we shall try to establish a better range of the values of x𝑥xitalic_x for which Ramanujan’s inequality does hold true.

5 A modified Bound for π(x)𝜋𝑥\pi(x)italic_π ( italic_x )

The derivations which we’ve made in the previous sections yielded several order estimates involving M(x)𝑀𝑥M(x)italic_M ( italic_x ), ψ(x)𝜓𝑥\psi(x)italic_ψ ( italic_x ) and especially π(x)𝜋𝑥\pi(x)italic_π ( italic_x ). In this section, we shall discuss how this method enables us to find more optimal bounds for π(x)𝜋𝑥\pi(x)italic_π ( italic_x ).

5.1 Upper Bound for π(x)𝜋𝑥\pi(x)italic_π ( italic_x )

A priori from Lemma (3.2.1), we have that for some positive constant α>0𝛼0\alpha>0italic_α > 0,

ψ(x)x+αx1/2(logx)2𝜓𝑥𝑥𝛼superscript𝑥12superscript𝑥2\displaystyle\psi(x)\leq x+\alpha x^{1/2}(\log x)^{2}italic_ψ ( italic_x ) ≤ italic_x + italic_α italic_x start_POSTSUPERSCRIPT 1 / 2 end_POSTSUPERSCRIPT ( roman_log italic_x ) start_POSTSUPERSCRIPT 2 end_POSTSUPERSCRIPT (5.1)

Substituting this into the the estimate (3.9) of Theorem (3.2.2) involving π(x)𝜋𝑥\pi(x)italic_π ( italic_x ),

π(x)x+αx1/2(logx)2logx+O(x(logx)2)=xlogx+αx1/2logx+O(x(logx)2).𝜋𝑥𝑥𝛼superscript𝑥12superscript𝑥2𝑥𝑂𝑥superscript𝑥2𝑥𝑥𝛼superscript𝑥12𝑥𝑂𝑥superscript𝑥2\displaystyle\pi(x)\leq\frac{x+\alpha x^{1/2}(\log x)^{2}}{\log x}+O\left(% \frac{x}{(\log x)^{2}}\right)=\frac{x}{\log x}+\alpha x^{1/2}\log x+O\left(% \frac{x}{(\log x)^{2}}\right).italic_π ( italic_x ) ≤ divide start_ARG italic_x + italic_α italic_x start_POSTSUPERSCRIPT 1 / 2 end_POSTSUPERSCRIPT ( roman_log italic_x ) start_POSTSUPERSCRIPT 2 end_POSTSUPERSCRIPT end_ARG start_ARG roman_log italic_x end_ARG + italic_O ( divide start_ARG italic_x end_ARG start_ARG ( roman_log italic_x ) start_POSTSUPERSCRIPT 2 end_POSTSUPERSCRIPT end_ARG ) = divide start_ARG italic_x end_ARG start_ARG roman_log italic_x end_ARG + italic_α italic_x start_POSTSUPERSCRIPT 1 / 2 end_POSTSUPERSCRIPT roman_log italic_x + italic_O ( divide start_ARG italic_x end_ARG start_ARG ( roman_log italic_x ) start_POSTSUPERSCRIPT 2 end_POSTSUPERSCRIPT end_ARG ) . (5.2)

5.2 Lower Bound for π(x)𝜋𝑥\pi(x)italic_π ( italic_x )

A priori using the estimate, M(x)=O(x1/2)𝑀𝑥𝑂superscript𝑥12M(x)=O(x^{1/2})italic_M ( italic_x ) = italic_O ( italic_x start_POSTSUPERSCRIPT 1 / 2 end_POSTSUPERSCRIPT ),

nxM(xn)=O(nx(xn)1/2)=O(x1/2nx1n1/2).subscript𝑛𝑥𝑀𝑥𝑛𝑂subscript𝑛𝑥superscript𝑥𝑛12𝑂superscript𝑥12subscript𝑛𝑥1superscript𝑛12\displaystyle\sum_{n\leq x}M\left(\frac{x}{n}\right)=O\left(\sum_{n\leq x}% \left(\frac{x}{n}\right)^{1/2}\right)=O\left(x^{1/2}\sum_{n\leq x}\frac{1}{n^{% 1/2}}\right).∑ start_POSTSUBSCRIPT italic_n ≤ italic_x end_POSTSUBSCRIPT italic_M ( divide start_ARG italic_x end_ARG start_ARG italic_n end_ARG ) = italic_O ( ∑ start_POSTSUBSCRIPT italic_n ≤ italic_x end_POSTSUBSCRIPT ( divide start_ARG italic_x end_ARG start_ARG italic_n end_ARG ) start_POSTSUPERSCRIPT 1 / 2 end_POSTSUPERSCRIPT ) = italic_O ( italic_x start_POSTSUPERSCRIPT 1 / 2 end_POSTSUPERSCRIPT ∑ start_POSTSUBSCRIPT italic_n ≤ italic_x end_POSTSUBSCRIPT divide start_ARG 1 end_ARG start_ARG italic_n start_POSTSUPERSCRIPT 1 / 2 end_POSTSUPERSCRIPT end_ARG ) .

Using the approximation, nx1n1/22xsimilar-tosubscript𝑛𝑥1superscript𝑛122𝑥\sum\limits_{n\leq x}\frac{1}{n^{1/2}}\sim 2\sqrt{x}∑ start_POSTSUBSCRIPT italic_n ≤ italic_x end_POSTSUBSCRIPT divide start_ARG 1 end_ARG start_ARG italic_n start_POSTSUPERSCRIPT 1 / 2 end_POSTSUPERSCRIPT end_ARG ∼ 2 square-root start_ARG italic_x end_ARG gives,

nxM(xn)=O(x).subscript𝑛𝑥𝑀𝑥𝑛𝑂𝑥\displaystyle\sum_{n\leq x}M\left(\frac{x}{n}\right)=O(x).∑ start_POSTSUBSCRIPT italic_n ≤ italic_x end_POSTSUBSCRIPT italic_M ( divide start_ARG italic_x end_ARG start_ARG italic_n end_ARG ) = italic_O ( italic_x ) .

Thus,

1logxnxM(xn)=O(xlogx).1𝑥subscript𝑛𝑥𝑀𝑥𝑛𝑂𝑥𝑥\displaystyle\frac{1}{\log x}\sum_{n\leq x}M\left(\frac{x}{n}\right)=O\left(% \frac{x}{\log x}\right).divide start_ARG 1 end_ARG start_ARG roman_log italic_x end_ARG ∑ start_POSTSUBSCRIPT italic_n ≤ italic_x end_POSTSUBSCRIPT italic_M ( divide start_ARG italic_x end_ARG start_ARG italic_n end_ARG ) = italic_O ( divide start_ARG italic_x end_ARG start_ARG roman_log italic_x end_ARG ) .

Therefore, applying the above derivation to (3.14) in Theorem (3.3.1), we assert that,

π(x)xlogxβxlogx+O(x(logx)2)=(1β)xlogx+O(x(logx)2).𝜋𝑥𝑥𝑥𝛽𝑥𝑥𝑂𝑥superscript𝑥21𝛽𝑥𝑥𝑂𝑥superscript𝑥2\displaystyle\pi(x)\geq\frac{x}{\log x}-\frac{\beta x}{\log x}+O\left(\frac{x}% {(\log x)^{2}}\right)=\left(1-\beta\right)\frac{x}{\log x}+O\left(\frac{x}{(% \log x)^{2}}\right).italic_π ( italic_x ) ≥ divide start_ARG italic_x end_ARG start_ARG roman_log italic_x end_ARG - divide start_ARG italic_β italic_x end_ARG start_ARG roman_log italic_x end_ARG + italic_O ( divide start_ARG italic_x end_ARG start_ARG ( roman_log italic_x ) start_POSTSUPERSCRIPT 2 end_POSTSUPERSCRIPT end_ARG ) = ( 1 - italic_β ) divide start_ARG italic_x end_ARG start_ARG roman_log italic_x end_ARG + italic_O ( divide start_ARG italic_x end_ARG start_ARG ( roman_log italic_x ) start_POSTSUPERSCRIPT 2 end_POSTSUPERSCRIPT end_ARG ) . (5.3)

for some positive constant β>0𝛽0\beta>0italic_β > 0. Hence, (5.2) and (5.3) allows us to state the following.

Theorem 5.2.1.

The following bounds on π(x)𝜋𝑥\pi(x)italic_π ( italic_x ) is valid for sufficiently large values of x𝑥xitalic_x :

(1β)xlogx+O(x(logx)2)π(x)xlogx+αx1/2logx+O(x(logx)2)1𝛽𝑥𝑥𝑂𝑥superscript𝑥2𝜋𝑥𝑥𝑥𝛼superscript𝑥12𝑥𝑂𝑥superscript𝑥2\displaystyle\left(1-\beta\right)\frac{x}{\log x}+O\left(\frac{x}{(\log x)^{2}% }\right)\leq\pi(x)\leq\frac{x}{\log x}+\alpha x^{1/2}\log x+O\left(\frac{x}{(% \log x)^{2}}\right)( 1 - italic_β ) divide start_ARG italic_x end_ARG start_ARG roman_log italic_x end_ARG + italic_O ( divide start_ARG italic_x end_ARG start_ARG ( roman_log italic_x ) start_POSTSUPERSCRIPT 2 end_POSTSUPERSCRIPT end_ARG ) ≤ italic_π ( italic_x ) ≤ divide start_ARG italic_x end_ARG start_ARG roman_log italic_x end_ARG + italic_α italic_x start_POSTSUPERSCRIPT 1 / 2 end_POSTSUPERSCRIPT roman_log italic_x + italic_O ( divide start_ARG italic_x end_ARG start_ARG ( roman_log italic_x ) start_POSTSUPERSCRIPT 2 end_POSTSUPERSCRIPT end_ARG ) (5.4)

for positive constants α,β>0𝛼𝛽0\alpha,\beta>0italic_α , italic_β > 0.

Remark 5.2.2.

The result (5.4) in Theorem (5.2.1) provides an alternative justification to the validity of the famous Prime Number Theorem. This can be observed from the fact that, π(x)𝜋𝑥\pi(x)italic_π ( italic_x ) is bounded on both sides by a constant multiple of xlogx𝑥𝑥\frac{x}{\log x}divide start_ARG italic_x end_ARG start_ARG roman_log italic_x end_ARG plus an error term, which can be minimized for large values of x𝑥xitalic_x.

6 An improved condition for Ramanujan’s Inequality

A priori adopting Sterneck’s deduction [5], as mentioned in section 2.42.42.42.4 of this text, we shall investigate the function 𝒢𝒢\mathcal{G}caligraphic_G for its sign for large values of x𝑥xitalic_x, with every intention of improving the claim made by Dudek and Platt [2, ref. Theorem 2].

6.1 Monotonicity of the function 𝒢𝒢\mathcal{G}caligraphic_G

Our aim in this section is to establish the following claim.

Proposition 6.1.1.

The function 𝒢(x)𝒢𝑥\mathcal{G}(x)caligraphic_G ( italic_x ) as defined in (4.1) is monotone decreasing for x201𝑥201x\geq 201italic_x ≥ 201.

Proof.

We intend on verifying that, 𝒢(x+ϵ)𝒢(x)<0𝒢𝑥italic-ϵ𝒢𝑥0\mathcal{G}(x+\epsilon)-\mathcal{G}(x)<0caligraphic_G ( italic_x + italic_ϵ ) - caligraphic_G ( italic_x ) < 0 for every large x𝑥xitalic_x and for every ϵ>0italic-ϵ0\epsilon>0italic_ϵ > 0 arbitraily chosen.

Observe from definition that the difference,

𝒢(x+ϵ)𝒢(x)=[(π(x+ϵ))2(π(x))2][e(x+ϵ)log(x+ϵ)π(x+ϵe)exlogxπ(x/e)]𝒢𝑥italic-ϵ𝒢𝑥delimited-[]superscript𝜋𝑥italic-ϵ2superscript𝜋𝑥2delimited-[]𝑒𝑥italic-ϵ𝑥italic-ϵ𝜋𝑥italic-ϵ𝑒𝑒𝑥𝑥𝜋𝑥𝑒\displaystyle\mathcal{G}(x+\epsilon)-\mathcal{G}(x)=\left[(\pi(x+\epsilon))^{2% }-(\pi(x))^{2}\right]-\left[\frac{e(x+\epsilon)}{\log(x+\epsilon)}\pi\left(% \frac{x+\epsilon}{e}\right)-\frac{ex}{\log x}\pi(x/e)\right]caligraphic_G ( italic_x + italic_ϵ ) - caligraphic_G ( italic_x ) = [ ( italic_π ( italic_x + italic_ϵ ) ) start_POSTSUPERSCRIPT 2 end_POSTSUPERSCRIPT - ( italic_π ( italic_x ) ) start_POSTSUPERSCRIPT 2 end_POSTSUPERSCRIPT ] - [ divide start_ARG italic_e ( italic_x + italic_ϵ ) end_ARG start_ARG roman_log ( italic_x + italic_ϵ ) end_ARG italic_π ( divide start_ARG italic_x + italic_ϵ end_ARG start_ARG italic_e end_ARG ) - divide start_ARG italic_e italic_x end_ARG start_ARG roman_log italic_x end_ARG italic_π ( italic_x / italic_e ) ] (6.1)

Now we evaluate,

(π(x+ϵ))2(π(x))2=(π(x+ϵ)π(x))(π(x+ϵ)+π(x))superscript𝜋𝑥italic-ϵ2superscript𝜋𝑥2𝜋𝑥italic-ϵ𝜋𝑥𝜋𝑥italic-ϵ𝜋𝑥\displaystyle(\pi(x+\epsilon))^{2}-(\pi(x))^{2}=\left(\pi(x+\epsilon)-\pi(x)% \right)\left(\pi(x+\epsilon)+\pi(x)\right)( italic_π ( italic_x + italic_ϵ ) ) start_POSTSUPERSCRIPT 2 end_POSTSUPERSCRIPT - ( italic_π ( italic_x ) ) start_POSTSUPERSCRIPT 2 end_POSTSUPERSCRIPT = ( italic_π ( italic_x + italic_ϵ ) - italic_π ( italic_x ) ) ( italic_π ( italic_x + italic_ϵ ) + italic_π ( italic_x ) ) (6.2)

Using the explicit forms (3.14):

π(x+ϵ)=xlogx+ϵlogxxϵ(logx)21logxnx+ϵM(x+ϵn)+O(x+ϵ(log(x+ϵ))2)𝜋𝑥italic-ϵ𝑥𝑥italic-ϵ𝑥𝑥italic-ϵsuperscript𝑥21𝑥subscript𝑛𝑥italic-ϵ𝑀𝑥italic-ϵ𝑛𝑂𝑥italic-ϵsuperscript𝑥italic-ϵ2\displaystyle\pi(x+\epsilon)=\frac{x}{\log x}+\frac{\epsilon}{\log x}-\frac{x% \epsilon}{(\log x)^{2}}-\frac{1}{\log x}\sum_{n\leq x+\epsilon}M\left(\frac{x+% \epsilon}{n}\right)+O\left(\frac{x+\epsilon}{(\log(x+\epsilon))^{2}}\right)italic_π ( italic_x + italic_ϵ ) = divide start_ARG italic_x end_ARG start_ARG roman_log italic_x end_ARG + divide start_ARG italic_ϵ end_ARG start_ARG roman_log italic_x end_ARG - divide start_ARG italic_x italic_ϵ end_ARG start_ARG ( roman_log italic_x ) start_POSTSUPERSCRIPT 2 end_POSTSUPERSCRIPT end_ARG - divide start_ARG 1 end_ARG start_ARG roman_log italic_x end_ARG ∑ start_POSTSUBSCRIPT italic_n ≤ italic_x + italic_ϵ end_POSTSUBSCRIPT italic_M ( divide start_ARG italic_x + italic_ϵ end_ARG start_ARG italic_n end_ARG ) + italic_O ( divide start_ARG italic_x + italic_ϵ end_ARG start_ARG ( roman_log ( italic_x + italic_ϵ ) ) start_POSTSUPERSCRIPT 2 end_POSTSUPERSCRIPT end_ARG )

Therefore,

π(x+ϵ)π(x)=ϵlogxxϵ(logx)21logxnx+ϵM(x+ϵn)+1logxnxM(xn)𝜋𝑥italic-ϵ𝜋𝑥italic-ϵ𝑥𝑥italic-ϵsuperscript𝑥21𝑥subscript𝑛𝑥italic-ϵ𝑀𝑥italic-ϵ𝑛1𝑥subscript𝑛𝑥𝑀𝑥𝑛\displaystyle\pi(x+\epsilon)-\pi(x)=\frac{\epsilon}{\log x}-\frac{x\epsilon}{(% \log x)^{2}}-\frac{1}{\log x}\sum_{n\leq x+\epsilon}M\left(\frac{x+\epsilon}{n% }\right)+\frac{1}{\log x}\sum_{n\leq x}M\left(\frac{x}{n}\right)italic_π ( italic_x + italic_ϵ ) - italic_π ( italic_x ) = divide start_ARG italic_ϵ end_ARG start_ARG roman_log italic_x end_ARG - divide start_ARG italic_x italic_ϵ end_ARG start_ARG ( roman_log italic_x ) start_POSTSUPERSCRIPT 2 end_POSTSUPERSCRIPT end_ARG - divide start_ARG 1 end_ARG start_ARG roman_log italic_x end_ARG ∑ start_POSTSUBSCRIPT italic_n ≤ italic_x + italic_ϵ end_POSTSUBSCRIPT italic_M ( divide start_ARG italic_x + italic_ϵ end_ARG start_ARG italic_n end_ARG ) + divide start_ARG 1 end_ARG start_ARG roman_log italic_x end_ARG ∑ start_POSTSUBSCRIPT italic_n ≤ italic_x end_POSTSUBSCRIPT italic_M ( divide start_ARG italic_x end_ARG start_ARG italic_n end_ARG )
+O(x+ϵ(log(x+ϵ))2x(logx)2)𝑂𝑥italic-ϵsuperscript𝑥italic-ϵ2𝑥superscript𝑥2\displaystyle\hskip 300.0pt+O\left(\frac{x+\epsilon}{(\log(x+\epsilon))^{2}}-% \frac{x}{(\log x)^{2}}\right)+ italic_O ( divide start_ARG italic_x + italic_ϵ end_ARG start_ARG ( roman_log ( italic_x + italic_ϵ ) ) start_POSTSUPERSCRIPT 2 end_POSTSUPERSCRIPT end_ARG - divide start_ARG italic_x end_ARG start_ARG ( roman_log italic_x ) start_POSTSUPERSCRIPT 2 end_POSTSUPERSCRIPT end_ARG ) (6.3)

For small ϵitalic-ϵ\epsilonitalic_ϵ, we use the linear approximation:

x+ϵlog(x+ϵ)xlogx+ϵlogxxϵ(logx)2𝑥italic-ϵ𝑥italic-ϵ𝑥𝑥italic-ϵ𝑥𝑥italic-ϵsuperscript𝑥2\displaystyle\frac{x+\epsilon}{\log(x+\epsilon)}\approx\frac{x}{\log x}+\frac{% \epsilon}{\log x}-\frac{x\epsilon}{(\log x)^{2}}divide start_ARG italic_x + italic_ϵ end_ARG start_ARG roman_log ( italic_x + italic_ϵ ) end_ARG ≈ divide start_ARG italic_x end_ARG start_ARG roman_log italic_x end_ARG + divide start_ARG italic_ϵ end_ARG start_ARG roman_log italic_x end_ARG - divide start_ARG italic_x italic_ϵ end_ARG start_ARG ( roman_log italic_x ) start_POSTSUPERSCRIPT 2 end_POSTSUPERSCRIPT end_ARG

And,

1log(x+ϵ)1logxϵx(logx)21𝑥italic-ϵ1𝑥italic-ϵ𝑥superscript𝑥2\displaystyle\frac{1}{\log(x+\epsilon)}\approx\frac{1}{\log x}-\frac{\epsilon}% {x(\log x)^{2}}divide start_ARG 1 end_ARG start_ARG roman_log ( italic_x + italic_ϵ ) end_ARG ≈ divide start_ARG 1 end_ARG start_ARG roman_log italic_x end_ARG - divide start_ARG italic_ϵ end_ARG start_ARG italic_x ( roman_log italic_x ) start_POSTSUPERSCRIPT 2 end_POSTSUPERSCRIPT end_ARG

Thus, we can compute further in (6.3) as follows,

π(x+ϵ)π(x)=ϵlogxxϵ(logx)21logxnx+ϵM(x+ϵn)+1logxnxM(xn)+O(xϵ(logx)2)𝜋𝑥italic-ϵ𝜋𝑥italic-ϵ𝑥𝑥italic-ϵsuperscript𝑥21𝑥subscript𝑛𝑥italic-ϵ𝑀𝑥italic-ϵ𝑛1𝑥subscript𝑛𝑥𝑀𝑥𝑛𝑂𝑥italic-ϵsuperscript𝑥2\displaystyle\pi(x+\epsilon)-\pi(x)=\frac{\epsilon}{\log x}-\frac{x\epsilon}{(% \log x)^{2}}-\frac{1}{\log x}\sum_{n\leq x+\epsilon}M\left(\frac{x+\epsilon}{n% }\right)+\frac{1}{\log x}\sum_{n\leq x}M\left(\frac{x}{n}\right)+O\left(\frac{% x\epsilon}{(\log x)^{2}}\right)italic_π ( italic_x + italic_ϵ ) - italic_π ( italic_x ) = divide start_ARG italic_ϵ end_ARG start_ARG roman_log italic_x end_ARG - divide start_ARG italic_x italic_ϵ end_ARG start_ARG ( roman_log italic_x ) start_POSTSUPERSCRIPT 2 end_POSTSUPERSCRIPT end_ARG - divide start_ARG 1 end_ARG start_ARG roman_log italic_x end_ARG ∑ start_POSTSUBSCRIPT italic_n ≤ italic_x + italic_ϵ end_POSTSUBSCRIPT italic_M ( divide start_ARG italic_x + italic_ϵ end_ARG start_ARG italic_n end_ARG ) + divide start_ARG 1 end_ARG start_ARG roman_log italic_x end_ARG ∑ start_POSTSUBSCRIPT italic_n ≤ italic_x end_POSTSUBSCRIPT italic_M ( divide start_ARG italic_x end_ARG start_ARG italic_n end_ARG ) + italic_O ( divide start_ARG italic_x italic_ϵ end_ARG start_ARG ( roman_log italic_x ) start_POSTSUPERSCRIPT 2 end_POSTSUPERSCRIPT end_ARG ) (6.4)

Similarly,

π(x+ϵ)+π(x)=2xlogx+ϵlogxxϵ(logx)21logxnx+ϵM(x+ϵn)𝜋𝑥italic-ϵ𝜋𝑥2𝑥𝑥italic-ϵ𝑥𝑥italic-ϵsuperscript𝑥21𝑥subscript𝑛𝑥italic-ϵ𝑀𝑥italic-ϵ𝑛\displaystyle\pi(x+\epsilon)+\pi(x)=2\frac{x}{\log x}+\frac{\epsilon}{\log x}-% \frac{x\epsilon}{(\log x)^{2}}-\frac{1}{\log x}\sum_{n\leq x+\epsilon}M\left(% \frac{x+\epsilon}{n}\right)italic_π ( italic_x + italic_ϵ ) + italic_π ( italic_x ) = 2 divide start_ARG italic_x end_ARG start_ARG roman_log italic_x end_ARG + divide start_ARG italic_ϵ end_ARG start_ARG roman_log italic_x end_ARG - divide start_ARG italic_x italic_ϵ end_ARG start_ARG ( roman_log italic_x ) start_POSTSUPERSCRIPT 2 end_POSTSUPERSCRIPT end_ARG - divide start_ARG 1 end_ARG start_ARG roman_log italic_x end_ARG ∑ start_POSTSUBSCRIPT italic_n ≤ italic_x + italic_ϵ end_POSTSUBSCRIPT italic_M ( divide start_ARG italic_x + italic_ϵ end_ARG start_ARG italic_n end_ARG )
+1logxnxM(xn)+O(xϵ(logx)2)1𝑥subscript𝑛𝑥𝑀𝑥𝑛𝑂𝑥italic-ϵsuperscript𝑥2\displaystyle\hskip 200.0pt+\frac{1}{\log x}\sum_{n\leq x}M\left(\frac{x}{n}% \right)+O\left(\frac{x\epsilon}{(\log x)^{2}}\right)+ divide start_ARG 1 end_ARG start_ARG roman_log italic_x end_ARG ∑ start_POSTSUBSCRIPT italic_n ≤ italic_x end_POSTSUBSCRIPT italic_M ( divide start_ARG italic_x end_ARG start_ARG italic_n end_ARG ) + italic_O ( divide start_ARG italic_x italic_ϵ end_ARG start_ARG ( roman_log italic_x ) start_POSTSUPERSCRIPT 2 end_POSTSUPERSCRIPT end_ARG ) (6.5)

Combining (6.4) and (6.5) yields,

(π(x+ϵ))2(π(x))2=(ϵlogxxϵ(logx)21logxnx+ϵM(x+ϵn)+1logxnxM(xn))superscript𝜋𝑥italic-ϵ2superscript𝜋𝑥2italic-ϵ𝑥𝑥italic-ϵsuperscript𝑥21𝑥subscript𝑛𝑥italic-ϵ𝑀𝑥italic-ϵ𝑛1𝑥subscript𝑛𝑥𝑀𝑥𝑛\displaystyle(\pi(x+\epsilon))^{2}-(\pi(x))^{2}=\left(\frac{\epsilon}{\log x}-% \frac{x\epsilon}{(\log x)^{2}}-\frac{1}{\log x}\sum_{n\leq x+\epsilon}M\left(% \frac{x+\epsilon}{n}\right)+\frac{1}{\log x}\sum_{n\leq x}M\left(\frac{x}{n}% \right)\right)( italic_π ( italic_x + italic_ϵ ) ) start_POSTSUPERSCRIPT 2 end_POSTSUPERSCRIPT - ( italic_π ( italic_x ) ) start_POSTSUPERSCRIPT 2 end_POSTSUPERSCRIPT = ( divide start_ARG italic_ϵ end_ARG start_ARG roman_log italic_x end_ARG - divide start_ARG italic_x italic_ϵ end_ARG start_ARG ( roman_log italic_x ) start_POSTSUPERSCRIPT 2 end_POSTSUPERSCRIPT end_ARG - divide start_ARG 1 end_ARG start_ARG roman_log italic_x end_ARG ∑ start_POSTSUBSCRIPT italic_n ≤ italic_x + italic_ϵ end_POSTSUBSCRIPT italic_M ( divide start_ARG italic_x + italic_ϵ end_ARG start_ARG italic_n end_ARG ) + divide start_ARG 1 end_ARG start_ARG roman_log italic_x end_ARG ∑ start_POSTSUBSCRIPT italic_n ≤ italic_x end_POSTSUBSCRIPT italic_M ( divide start_ARG italic_x end_ARG start_ARG italic_n end_ARG ) )
(2xlogx+ϵlogxxϵ(logx)2)+O(xϵ(logx)2)2𝑥𝑥italic-ϵ𝑥𝑥italic-ϵsuperscript𝑥2𝑂𝑥italic-ϵsuperscript𝑥2\displaystyle\hskip 200.0pt\left(2\frac{x}{\log x}+\frac{\epsilon}{\log x}-% \frac{x\epsilon}{(\log x)^{2}}\right)+O\left(\frac{x\epsilon}{(\log x)^{2}}\right)( 2 divide start_ARG italic_x end_ARG start_ARG roman_log italic_x end_ARG + divide start_ARG italic_ϵ end_ARG start_ARG roman_log italic_x end_ARG - divide start_ARG italic_x italic_ϵ end_ARG start_ARG ( roman_log italic_x ) start_POSTSUPERSCRIPT 2 end_POSTSUPERSCRIPT end_ARG ) + italic_O ( divide start_ARG italic_x italic_ϵ end_ARG start_ARG ( roman_log italic_x ) start_POSTSUPERSCRIPT 2 end_POSTSUPERSCRIPT end_ARG )
=2xϵ(logx)22x2ϵ(logx)32x(logx)2nx+ϵM(x+ϵn)+2x(logx)2nxM(xn)+O(xϵ(logx)2)absent2𝑥italic-ϵsuperscript𝑥22superscript𝑥2italic-ϵsuperscript𝑥32𝑥superscript𝑥2subscript𝑛𝑥italic-ϵ𝑀𝑥italic-ϵ𝑛2𝑥superscript𝑥2subscript𝑛𝑥𝑀𝑥𝑛𝑂𝑥italic-ϵsuperscript𝑥2\displaystyle=2\frac{x\epsilon}{(\log x)^{2}}-2\frac{x^{2}\epsilon}{(\log x)^{% 3}}-2\frac{x}{(\log x)^{2}}\sum_{n\leq x+\epsilon}M\left(\frac{x+\epsilon}{n}% \right)+2\frac{x}{(\log x)^{2}}\sum_{n\leq x}M\left(\frac{x}{n}\right)+O\left(% \frac{x\epsilon}{(\log x)^{2}}\right)= 2 divide start_ARG italic_x italic_ϵ end_ARG start_ARG ( roman_log italic_x ) start_POSTSUPERSCRIPT 2 end_POSTSUPERSCRIPT end_ARG - 2 divide start_ARG italic_x start_POSTSUPERSCRIPT 2 end_POSTSUPERSCRIPT italic_ϵ end_ARG start_ARG ( roman_log italic_x ) start_POSTSUPERSCRIPT 3 end_POSTSUPERSCRIPT end_ARG - 2 divide start_ARG italic_x end_ARG start_ARG ( roman_log italic_x ) start_POSTSUPERSCRIPT 2 end_POSTSUPERSCRIPT end_ARG ∑ start_POSTSUBSCRIPT italic_n ≤ italic_x + italic_ϵ end_POSTSUBSCRIPT italic_M ( divide start_ARG italic_x + italic_ϵ end_ARG start_ARG italic_n end_ARG ) + 2 divide start_ARG italic_x end_ARG start_ARG ( roman_log italic_x ) start_POSTSUPERSCRIPT 2 end_POSTSUPERSCRIPT end_ARG ∑ start_POSTSUBSCRIPT italic_n ≤ italic_x end_POSTSUBSCRIPT italic_M ( divide start_ARG italic_x end_ARG start_ARG italic_n end_ARG ) + italic_O ( divide start_ARG italic_x italic_ϵ end_ARG start_ARG ( roman_log italic_x ) start_POSTSUPERSCRIPT 2 end_POSTSUPERSCRIPT end_ARG ) (6.6)

As for the second term in (6.1),

e(x+ϵ)log(x+ϵ)π(x+ϵe)exlogxπ(x/e)e(x+ϵ)log(x+ϵ)(x+ϵelog(x+ϵ/e))exlogx(xelog(x/e))𝑒𝑥italic-ϵ𝑥italic-ϵ𝜋𝑥italic-ϵ𝑒𝑒𝑥𝑥𝜋𝑥𝑒𝑒𝑥italic-ϵ𝑥italic-ϵ𝑥italic-ϵ𝑒𝑥italic-ϵ𝑒𝑒𝑥𝑥𝑥𝑒𝑥𝑒\displaystyle\frac{e(x+\epsilon)}{\log(x+\epsilon)}\pi\left(\frac{x+\epsilon}{% e}\right)-\frac{ex}{\log x}\pi(x/e)\approx\frac{e(x+\epsilon)}{\log(x+\epsilon% )}\left(\frac{x+\epsilon}{e\log(x+\epsilon/e)}\right)-\frac{ex}{\log x}\left(% \frac{x}{e\log(x/e)}\right)divide start_ARG italic_e ( italic_x + italic_ϵ ) end_ARG start_ARG roman_log ( italic_x + italic_ϵ ) end_ARG italic_π ( divide start_ARG italic_x + italic_ϵ end_ARG start_ARG italic_e end_ARG ) - divide start_ARG italic_e italic_x end_ARG start_ARG roman_log italic_x end_ARG italic_π ( italic_x / italic_e ) ≈ divide start_ARG italic_e ( italic_x + italic_ϵ ) end_ARG start_ARG roman_log ( italic_x + italic_ϵ ) end_ARG ( divide start_ARG italic_x + italic_ϵ end_ARG start_ARG italic_e roman_log ( italic_x + italic_ϵ / italic_e ) end_ARG ) - divide start_ARG italic_e italic_x end_ARG start_ARG roman_log italic_x end_ARG ( divide start_ARG italic_x end_ARG start_ARG italic_e roman_log ( italic_x / italic_e ) end_ARG )

[Using the earlier approximations for π(x+ϵe)𝜋𝑥italic-ϵ𝑒\pi\left(\frac{x+\epsilon}{e}\right)italic_π ( divide start_ARG italic_x + italic_ϵ end_ARG start_ARG italic_e end_ARG ) and π(xe)𝜋𝑥𝑒\pi\left(\frac{x}{e}\right)italic_π ( divide start_ARG italic_x end_ARG start_ARG italic_e end_ARG )]

=(x+ϵ)2(log(x+ϵ))2x2(logx)2+O(xϵ(logx)3)=(x+ϵ)2x2(logx)2+O(xϵ(logx)3)absentsuperscript𝑥italic-ϵ2superscript𝑥italic-ϵ2superscript𝑥2superscript𝑥2𝑂𝑥italic-ϵsuperscript𝑥3superscript𝑥italic-ϵ2superscript𝑥2superscript𝑥2𝑂𝑥italic-ϵsuperscript𝑥3\displaystyle=\frac{(x+\epsilon)^{2}}{(\log(x+\epsilon))^{2}}-\frac{x^{2}}{(% \log x)^{2}}+O\left(\frac{x\epsilon}{(\log x)^{3}}\right)=\frac{(x+\epsilon)^{% 2}-x^{2}}{(\log x)^{2}}+O\left(\frac{x\epsilon}{(\log x)^{3}}\right)= divide start_ARG ( italic_x + italic_ϵ ) start_POSTSUPERSCRIPT 2 end_POSTSUPERSCRIPT end_ARG start_ARG ( roman_log ( italic_x + italic_ϵ ) ) start_POSTSUPERSCRIPT 2 end_POSTSUPERSCRIPT end_ARG - divide start_ARG italic_x start_POSTSUPERSCRIPT 2 end_POSTSUPERSCRIPT end_ARG start_ARG ( roman_log italic_x ) start_POSTSUPERSCRIPT 2 end_POSTSUPERSCRIPT end_ARG + italic_O ( divide start_ARG italic_x italic_ϵ end_ARG start_ARG ( roman_log italic_x ) start_POSTSUPERSCRIPT 3 end_POSTSUPERSCRIPT end_ARG ) = divide start_ARG ( italic_x + italic_ϵ ) start_POSTSUPERSCRIPT 2 end_POSTSUPERSCRIPT - italic_x start_POSTSUPERSCRIPT 2 end_POSTSUPERSCRIPT end_ARG start_ARG ( roman_log italic_x ) start_POSTSUPERSCRIPT 2 end_POSTSUPERSCRIPT end_ARG + italic_O ( divide start_ARG italic_x italic_ϵ end_ARG start_ARG ( roman_log italic_x ) start_POSTSUPERSCRIPT 3 end_POSTSUPERSCRIPT end_ARG )
=2xϵ+ϵ2(logx)2+O(xϵ(logx)3)absent2𝑥italic-ϵsuperscriptitalic-ϵ2superscript𝑥2𝑂𝑥italic-ϵsuperscript𝑥3\displaystyle\hskip 200.0pt=\frac{2x\epsilon+\epsilon^{2}}{(\log x)^{2}}+O% \left(\frac{x\epsilon}{(\log x)^{3}}\right)= divide start_ARG 2 italic_x italic_ϵ + italic_ϵ start_POSTSUPERSCRIPT 2 end_POSTSUPERSCRIPT end_ARG start_ARG ( roman_log italic_x ) start_POSTSUPERSCRIPT 2 end_POSTSUPERSCRIPT end_ARG + italic_O ( divide start_ARG italic_x italic_ϵ end_ARG start_ARG ( roman_log italic_x ) start_POSTSUPERSCRIPT 3 end_POSTSUPERSCRIPT end_ARG ) (6.7)

Hence, substituting (6.6) and (6.7) in (6.1) and simplifying,

𝒢(x+ϵ)𝒢(x)=(2xϵ(logx)22x2ϵ(logx)32x(logx)2nx+ϵM(x+ϵn)+2x(logx)2nxM(xn))𝒢𝑥italic-ϵ𝒢𝑥2𝑥italic-ϵsuperscript𝑥22superscript𝑥2italic-ϵsuperscript𝑥32𝑥superscript𝑥2subscript𝑛𝑥italic-ϵ𝑀𝑥italic-ϵ𝑛2𝑥superscript𝑥2subscript𝑛𝑥𝑀𝑥𝑛\displaystyle\mathcal{G}(x+\epsilon)-\mathcal{G}(x)=\left(2\frac{x\epsilon}{(% \log x)^{2}}-2\frac{x^{2}\epsilon}{(\log x)^{3}}-2\frac{x}{(\log x)^{2}}\sum_{% n\leq x+\epsilon}M\left(\frac{x+\epsilon}{n}\right)+2\frac{x}{(\log x)^{2}}% \sum_{n\leq x}M\left(\frac{x}{n}\right)\right)caligraphic_G ( italic_x + italic_ϵ ) - caligraphic_G ( italic_x ) = ( 2 divide start_ARG italic_x italic_ϵ end_ARG start_ARG ( roman_log italic_x ) start_POSTSUPERSCRIPT 2 end_POSTSUPERSCRIPT end_ARG - 2 divide start_ARG italic_x start_POSTSUPERSCRIPT 2 end_POSTSUPERSCRIPT italic_ϵ end_ARG start_ARG ( roman_log italic_x ) start_POSTSUPERSCRIPT 3 end_POSTSUPERSCRIPT end_ARG - 2 divide start_ARG italic_x end_ARG start_ARG ( roman_log italic_x ) start_POSTSUPERSCRIPT 2 end_POSTSUPERSCRIPT end_ARG ∑ start_POSTSUBSCRIPT italic_n ≤ italic_x + italic_ϵ end_POSTSUBSCRIPT italic_M ( divide start_ARG italic_x + italic_ϵ end_ARG start_ARG italic_n end_ARG ) + 2 divide start_ARG italic_x end_ARG start_ARG ( roman_log italic_x ) start_POSTSUPERSCRIPT 2 end_POSTSUPERSCRIPT end_ARG ∑ start_POSTSUBSCRIPT italic_n ≤ italic_x end_POSTSUBSCRIPT italic_M ( divide start_ARG italic_x end_ARG start_ARG italic_n end_ARG ) )
(2xϵ+ϵ2(logx)2)+O(xϵ(logx)3)2𝑥italic-ϵsuperscriptitalic-ϵ2superscript𝑥2𝑂𝑥italic-ϵsuperscript𝑥3\displaystyle\hskip 300.0pt-\left(\frac{2x\epsilon+\epsilon^{2}}{(\log x)^{2}}% \right)+O\left(\frac{x\epsilon}{(\log x)^{3}}\right)- ( divide start_ARG 2 italic_x italic_ϵ + italic_ϵ start_POSTSUPERSCRIPT 2 end_POSTSUPERSCRIPT end_ARG start_ARG ( roman_log italic_x ) start_POSTSUPERSCRIPT 2 end_POSTSUPERSCRIPT end_ARG ) + italic_O ( divide start_ARG italic_x italic_ϵ end_ARG start_ARG ( roman_log italic_x ) start_POSTSUPERSCRIPT 3 end_POSTSUPERSCRIPT end_ARG )
=2x(logx)2(nx+ϵM(x+ϵn)nxM(xn))ϵ2(logx)2+O(xϵ(logx)3)absent2𝑥superscript𝑥2subscript𝑛𝑥italic-ϵ𝑀𝑥italic-ϵ𝑛subscript𝑛𝑥𝑀𝑥𝑛superscriptitalic-ϵ2superscript𝑥2𝑂𝑥italic-ϵsuperscript𝑥3\displaystyle=-2\frac{x}{(\log x)^{2}}\left(\sum_{n\leq x+\epsilon}M\left(% \frac{x+\epsilon}{n}\right)-\sum_{n\leq x}M\left(\frac{x}{n}\right)\right)-% \frac{\epsilon^{2}}{(\log x)^{2}}+O\left(\frac{x\epsilon}{(\log x)^{3}}\right)= - 2 divide start_ARG italic_x end_ARG start_ARG ( roman_log italic_x ) start_POSTSUPERSCRIPT 2 end_POSTSUPERSCRIPT end_ARG ( ∑ start_POSTSUBSCRIPT italic_n ≤ italic_x + italic_ϵ end_POSTSUBSCRIPT italic_M ( divide start_ARG italic_x + italic_ϵ end_ARG start_ARG italic_n end_ARG ) - ∑ start_POSTSUBSCRIPT italic_n ≤ italic_x end_POSTSUBSCRIPT italic_M ( divide start_ARG italic_x end_ARG start_ARG italic_n end_ARG ) ) - divide start_ARG italic_ϵ start_POSTSUPERSCRIPT 2 end_POSTSUPERSCRIPT end_ARG start_ARG ( roman_log italic_x ) start_POSTSUPERSCRIPT 2 end_POSTSUPERSCRIPT end_ARG + italic_O ( divide start_ARG italic_x italic_ϵ end_ARG start_ARG ( roman_log italic_x ) start_POSTSUPERSCRIPT 3 end_POSTSUPERSCRIPT end_ARG ) (6.8)

We utilize Sterneck’s conjecture,

|M(x)|<12x, for x201.formulae-sequence𝑀𝑥12𝑥 for 𝑥201\displaystyle|M(x)|<\frac{1}{2}\sqrt{x},\hskip 10.0pt\mbox{ for }x\geq 201.| italic_M ( italic_x ) | < divide start_ARG 1 end_ARG start_ARG 2 end_ARG square-root start_ARG italic_x end_ARG , for italic_x ≥ 201 .

to bound the difference as follows,

2x(logx)2(nx+ϵM(x+ϵn)nxM(xn))2x(logx)2(12x+ϵ12x)2𝑥superscript𝑥2subscript𝑛𝑥italic-ϵ𝑀𝑥italic-ϵ𝑛subscript𝑛𝑥𝑀𝑥𝑛2𝑥superscript𝑥212𝑥italic-ϵ12𝑥\displaystyle-2\frac{x}{(\log x)^{2}}\left(\sum_{n\leq x+\epsilon}M\left(\frac% {x+\epsilon}{n}\right)-\sum_{n\leq x}M\left(\frac{x}{n}\right)\right)\leq-2% \frac{x}{(\log x)^{2}}\left(\frac{1}{2}\sqrt{x+\epsilon}-\frac{1}{2}\sqrt{x}\right)- 2 divide start_ARG italic_x end_ARG start_ARG ( roman_log italic_x ) start_POSTSUPERSCRIPT 2 end_POSTSUPERSCRIPT end_ARG ( ∑ start_POSTSUBSCRIPT italic_n ≤ italic_x + italic_ϵ end_POSTSUBSCRIPT italic_M ( divide start_ARG italic_x + italic_ϵ end_ARG start_ARG italic_n end_ARG ) - ∑ start_POSTSUBSCRIPT italic_n ≤ italic_x end_POSTSUBSCRIPT italic_M ( divide start_ARG italic_x end_ARG start_ARG italic_n end_ARG ) ) ≤ - 2 divide start_ARG italic_x end_ARG start_ARG ( roman_log italic_x ) start_POSTSUPERSCRIPT 2 end_POSTSUPERSCRIPT end_ARG ( divide start_ARG 1 end_ARG start_ARG 2 end_ARG square-root start_ARG italic_x + italic_ϵ end_ARG - divide start_ARG 1 end_ARG start_ARG 2 end_ARG square-root start_ARG italic_x end_ARG ) (6.9)

Subsequently, ϵ>0italic-ϵ0\epsilon>0italic_ϵ > 0 enables us to conclude from (6.8) and (6.9),

𝒢(x+ϵ)𝒢(x)2x(logx)2(12(x+ϵx))ϵ2(logx)2+O(xϵ(logx)3)𝒢𝑥italic-ϵ𝒢𝑥2𝑥superscript𝑥212𝑥italic-ϵ𝑥superscriptitalic-ϵ2superscript𝑥2𝑂𝑥italic-ϵsuperscript𝑥3\displaystyle\mathcal{G}(x+\epsilon)-\mathcal{G}(x)\leq-2\frac{x}{(\log x)^{2}% }\left(\frac{1}{2}(\sqrt{x+\epsilon}-\sqrt{x})\right)-\frac{\epsilon^{2}}{(% \log x)^{2}}+O\left(\frac{x\epsilon}{(\log x)^{3}}\right)caligraphic_G ( italic_x + italic_ϵ ) - caligraphic_G ( italic_x ) ≤ - 2 divide start_ARG italic_x end_ARG start_ARG ( roman_log italic_x ) start_POSTSUPERSCRIPT 2 end_POSTSUPERSCRIPT end_ARG ( divide start_ARG 1 end_ARG start_ARG 2 end_ARG ( square-root start_ARG italic_x + italic_ϵ end_ARG - square-root start_ARG italic_x end_ARG ) ) - divide start_ARG italic_ϵ start_POSTSUPERSCRIPT 2 end_POSTSUPERSCRIPT end_ARG start_ARG ( roman_log italic_x ) start_POSTSUPERSCRIPT 2 end_POSTSUPERSCRIPT end_ARG + italic_O ( divide start_ARG italic_x italic_ϵ end_ARG start_ARG ( roman_log italic_x ) start_POSTSUPERSCRIPT 3 end_POSTSUPERSCRIPT end_ARG )
=x(logx)2(x+ϵx)ϵ2(logx)2+O(xϵ(logx)3)absent𝑥superscript𝑥2𝑥italic-ϵ𝑥superscriptitalic-ϵ2superscript𝑥2𝑂𝑥italic-ϵsuperscript𝑥3\displaystyle\hskip 50.0pt=-\frac{x}{(\log x)^{2}}(\sqrt{x+\epsilon}-\sqrt{x})% -\frac{\epsilon^{2}}{(\log x)^{2}}+O\left(\frac{x\epsilon}{(\log x)^{3}}\right)= - divide start_ARG italic_x end_ARG start_ARG ( roman_log italic_x ) start_POSTSUPERSCRIPT 2 end_POSTSUPERSCRIPT end_ARG ( square-root start_ARG italic_x + italic_ϵ end_ARG - square-root start_ARG italic_x end_ARG ) - divide start_ARG italic_ϵ start_POSTSUPERSCRIPT 2 end_POSTSUPERSCRIPT end_ARG start_ARG ( roman_log italic_x ) start_POSTSUPERSCRIPT 2 end_POSTSUPERSCRIPT end_ARG + italic_O ( divide start_ARG italic_x italic_ϵ end_ARG start_ARG ( roman_log italic_x ) start_POSTSUPERSCRIPT 3 end_POSTSUPERSCRIPT end_ARG ) (6.10)

Since x+ϵx>0𝑥italic-ϵ𝑥0\sqrt{x+\epsilon}-\sqrt{x}>0square-root start_ARG italic_x + italic_ϵ end_ARG - square-root start_ARG italic_x end_ARG > 0 for any ϵ>0italic-ϵ0\epsilon>0italic_ϵ > 0 and the error term O(xϵ(logx)3)𝑂𝑥italic-ϵsuperscript𝑥3O\left(\frac{x\epsilon}{(\log x)^{3}}\right)italic_O ( divide start_ARG italic_x italic_ϵ end_ARG start_ARG ( roman_log italic_x ) start_POSTSUPERSCRIPT 3 end_POSTSUPERSCRIPT end_ARG ) is smaller, it implies,

𝒢(x+ϵ)𝒢(x)<0 for every x+ϵ>x,ϵ>0𝒢𝑥italic-ϵ𝒢𝑥expectation0 for every 𝑥italic-ϵ𝑥italic-ϵ0\displaystyle\mathcal{G}(x+\epsilon)-\mathcal{G}(x)<0\text{ for every }x+% \epsilon>x,\epsilon>0caligraphic_G ( italic_x + italic_ϵ ) - caligraphic_G ( italic_x ) < 0 for every italic_x + italic_ϵ > italic_x , italic_ϵ > 0

Thus, 𝒢(x)𝒢𝑥\mathcal{G}(x)caligraphic_G ( italic_x ) is monotone decreasing for large x𝑥xitalic_x such that, x201𝑥201x\geq 201italic_x ≥ 201.

Remark 6.1.2.

Note that, Sterneck’s Conjecture only establishes the fact that,

|M(n)|<12n, for n>200.formulae-sequence𝑀𝑛12𝑛 for 𝑛200\displaystyle|M(n)|<\frac{1}{2}\sqrt{n},\hskip 10.0pt\mbox{ for }n>200.| italic_M ( italic_n ) | < divide start_ARG 1 end_ARG start_ARG 2 end_ARG square-root start_ARG italic_n end_ARG , for italic_n > 200 .

where, n𝑛n\in\mathbb{N}italic_n ∈ blackboard_N. But in this case, since, we’re dealing with x𝑥x\in\mathbb{R}italic_x ∈ blackboard_R, hence, we’ve modified the lower bound for x𝑥xitalic_x accordingly.

6.2 A better range for the values of x𝑥xitalic_x

Just to recall, Proposition (6.1.1) comments on the monotonicity of 𝒢𝒢\mathcal{G}caligraphic_G for x𝑥xitalic_x within a certain interval. Thus for every sufficiently large x201𝑥201x\geq 201italic_x ≥ 201, we must have,

𝒢(x)<𝒢(logxe2)<0𝒢𝑥𝒢𝑥superscript𝑒20\displaystyle\mathcal{G}(x)<\mathcal{G}\left(\frac{\log x}{e^{2}}\right)<0caligraphic_G ( italic_x ) < caligraphic_G ( divide start_ARG roman_log italic_x end_ARG start_ARG italic_e start_POSTSUPERSCRIPT 2 end_POSTSUPERSCRIPT end_ARG ) < 0

provided, logxe2201𝑥superscript𝑒2201\frac{\log x}{e^{2}}\geq 201divide start_ARG roman_log italic_x end_ARG start_ARG italic_e start_POSTSUPERSCRIPT 2 end_POSTSUPERSCRIPT end_ARG ≥ 201, i.e., logx1485.2002758851𝑥1485.2002758851\log x\geq 1485.2002758851roman_log italic_x ≥ 1485.2002758851. Therefore, we have our following improved bound on x𝑥xitalic_x in order to satisfy (1.1).

Theorem 6.2.1.

The Ramanujan’s Inequality (1.1) is unconditionally true for every xexp(1486)𝑥1486x\geq\exp(1486)italic_x ≥ roman_exp ( 1486 ).

Acknowledgments

I’ll always be grateful to Prof. Adrian W. Dudek ( Adjunct Associate Professor, Department of Mathematics and Physics, University of Queensland, Australia ) for inspiring me to work on this problem and pursue research in this topic. His leading publications in this area helped me immensely in detailed understanding of the essential concepts.

References

  • [1] Ramanujan Aiyangar, Srinivasa, and Bruce C Berndt, Ramanujan’s Notebooks: Part IV, New York: Springer-Verlag, 1994.
  • [2] Adrian W. Dudek, David J. Platt, On Solving a Curious Inequality of Ramanujan, Experimental Mathematics, 24:3, 289-294, DOI: 10.1080/10586458.2014.990118, 2015.
  • [3] E. C. Titchmarsh, The Theory of the Riemann Zeta-function, Oxford University Press, 1951. Second edition revised by D. R. Heath-Brown, published by Oxford University Press, 1986.
  • [4] F. Mertens, U¨¨𝑈\ddot{U}over¨ start_ARG italic_U end_ARGber eine zahlentheoretische Funktion, Sitzungsber. Akad. Wiss. Wien 106(IIa) (1897) 761-830 .
  • [5] R. D. von Sterneck, Die zahlentheoretische Funktion σ(n)𝜎𝑛\sigma(n)italic_σ ( italic_n ) bis zur Grenze 5000000500000050000005000000, Sitzungsber. Akad. Wiss. Wien 121(IIa) (1912) 1083-1096 .
  • [6] De, Subham, ”On the Order Estimates for Specific Functions of ζ(s)𝜁𝑠\zeta(s)italic_ζ ( italic_s ) and its Contribution towards the Analytic Proof of The Prime Number Theorem.”, arXiv preprint arXiv:2308.16303 (2023).
  • [7] L. Schoenfeld, Sharper bounds for the Chebyshev Functions θ(x)𝜃𝑥\theta(x)italic_θ ( italic_x ) and ψ(x)𝜓𝑥\psi(x)italic_ψ ( italic_x ). II, Mathematics of Computation, vol. 30, no. 134, pp. 337-360; DOI: 10.2307/2005976.
  • [8] A. E. Ingham, The distribution of prime numbers, Cambridge University Press, 1932. Reprinted by Stechert-Hafner, 1964, and (with a foreword by R. C. Vaughan) by Cambridge University Press, 1990.
  • [9] R. A. MacLeod, A new estimate for the sum M(x)=nxμ(n)𝑀𝑥subscript𝑛𝑥𝜇𝑛M(x)=\sum\limits_{n\leq x}\mu(n)italic_M ( italic_x ) = ∑ start_POSTSUBSCRIPT italic_n ≤ italic_x end_POSTSUBSCRIPT italic_μ ( italic_n ), Acta Arith. 13 (1967) 49-59. Erratum, ibid. 16 (1969) 99-100.
  • [10] F. Dress, Majorations de la fonction sommatoire de la fonction de Mo¨¨𝑜\ddot{o}over¨ start_ARG italic_o end_ARGbius, Bull. Soc. Math. Fr., Suppl., Mem. 49-50 (1977) 47-52.
  • [11] H. G. Diamond, K. S. McCurley, Constructive elementary estimates for M(x)𝑀𝑥M(x)italic_M ( italic_x ), In M. I. Knopp, editor, Analytic Number Theory, Lecture Notes in Mathematics 899, pp. 239-253, Springer, 1982.
  • [12] N. Costa Pereira, Elementary estimate for the Chebyshev function ψ(x)𝜓𝑥\psi(x)italic_ψ ( italic_x ) and the Mo¨¨𝑜\ddot{o}over¨ start_ARG italic_o end_ARGbius function M(x)𝑀𝑥M(x)italic_M ( italic_x ), Acta Arith. 52 (1989) 307-337.
  • [13] F. Dress, M. El Marraki, Fonction sommatoire de la fonction de Mobius. 2, Majorations asymptotiques elementaires, Exp. Math. 2 (1993) 99-112.
  • [14] E. Landau, Handbuch der Lehre von der Verteilung der Primzahlen, Vol. 2 (of 2), Teubner, 1909. Reprinted by Chelsea, 1953.
  • [15] De, Subham, ”On the Order Estimate of the Mertens Function and its Relation to the Zeros of the Riemann Zeta Function”, Preprints 2023, 2023090723. https://doi.org/10.20944/preprints202309.0723.v1
  • [16] T. Kotnik, J. van de Lune, On the order of the Mertens function, experimental mathematics, Vol. 13 (2004), pp. 473-481.
  • [17] T. M. Apostol, Introduction to Analytic Number Theory, Springer, 1976.
  • [18] G.J.O. Jameson, The Prime Number Theorem, Cambridge ; New York : Cambridge University Press, London Mathematical Society student texts, Vol. 53, 2003.
  • [19] Olver, F. W. J., Asymptotics and Special Functions, Academic Press, New York, 1974.
  • [20] De, Subham, On the proof of the Prime Number Theorem using Order Estimates for the Chebyshev Theta Function, International Journal of Science and Research (IJSR), Vol. 12 Issue 11, Nov. 2023, pp. 1677-1691.
  • [21] L Ahlfors, Complex Analysis, McGraw-Hill Education , 3rdsuperscript3𝑟𝑑3^{rd}3 start_POSTSUPERSCRIPT italic_r italic_d end_POSTSUPERSCRIPT edition , Jan. 1, 1979 .
  • [22] Mehdi Hassani, “On an Inequality of Ramanujan Concerning the Prime Counting Function”, Ramanujan Journal 28 (2012), 435–442.
  • [23] Ramanujan, S, “Collected Papers”, Chelsea, New York, 1962.
  • [24] Hardy, G. H., A formula of Ramanujan in the theory of primes, 1. London Math. Soc. 12 (1937), 94-98.
  • [25] Hardy, G. H., Collected Papers, vol. II, Clarendon Press, Oxford, 1967.
  • [26] Axler, Christian, On Ramanujan’s prime counting inequality,arXiv preprint arXiv:2207.02486 (2022).